Examveda English

  • Uploaded by: Dhiman Nath
  • 0
  • 0
  • January 2021
  • PDF

This document was uploaded by user and they confirmed that they have the permission to share it. If you are author or own the copyright of this book, please report to us by using this DMCA report form. Report DMCA


Overview

Download & View Examveda English as PDF for free.

More details

  • Words: 85,444
  • Pages: 243
Loading documents preview...
Contents Topic

Voice Narration (Direct and Indirect) Change of Speech Spelling Check Synonyms Antonyms One Word Substitution One Word Substitutes Idioms and Phrases Verbal Analogies Selecting Words Sentence Completion Completing Statements Common Error Detection Spotting Errors Sentence Correction Sentence Improvement Sentence Formation Ordering of Words Ordering of Sentences

1|Page

Page 2 6 14 17 26 29 32 38 44 59 61 75 79 85 125 196 205 214 216 224

Voice

How to change the Voice of a sentence - Tips and Tricks Active and Passive Voice: A sentence is said to be in Active Voice when the subject does something. Similarly, a sentence is said to be in Passive Voice when something is done to the subject. Active Voice : Ram killed Ravan. Passive Voice : Ravan was killed by Ram. In the above sentence Ram is the subject and Ravan is the object. In the active voice sentence Ram i.e., the subject has killed the object Ravan. In the second sentence Ravan is the subject but he allowed himself to be killed by subject Ram. Here the subject is Passive. Therefore, it is called Passive voice. Here it must be noted that: I becomes me in Passive voice. We becomes us You remains you He becomes him She becomes her It remains it They becomes them. I) Present Tense: a) Simple Present: 1. I watch movies every week. - Active Voice Movies are watched by me every week. - Passive Voice 2. We watch movies every week. Movies are watched by us every week. 3. You watch movies every week. Movies are watched by you every week. 4. He watches movies every week. Movies are watched by him every week. 5. She watches movies every week. Movies are watched by her every week. 6. It watches movies every week. Movies are watched by it every week. 7. They watch movies every week. Movies are watched by them every week. b) Present Continuous : 1. I am watching a movie now. A movie is being watched by me now. 2. We are watching a movie now. A movie is being watched by us now. 3. You are watching a movie now. A movie is being watched by you now. 2|Page

4. He is watching a movie now. A movie is being watched by him now. 5. She is watching a movie now. A movie is being watched by her now. 6. It is watching a movie now. A movie is being watched by it now. 7. They are watching a movie now. A movie is being watched by them now. c) Present Perfect: 1. I have watched a movie today. A movie has been watched by me today. 2. We have watched a movie today. A movie has been watched by us today. 3. You have watched a movie today. A movie has been watched by you today. 4. He has watched a movie today. A movie has been watched by him today. 5. She has watched a movie today. A movie has been watched by her today. 6. It has watched a movie today. A movie has been watched by it today. 7. They have watched a movie today. A movie has been watched by them today. II) Past Tense a) Simple Past: 1. I watched a movie yesterday. A movie was watched by me yesterday. 2. We watched a movie yesterday. A movie was watched by us yesterday. 3. You watched a movie yesterday. A movie was watched by you yesterday. 4. He watched a movie yesterday. A movie was watched by him yesterday. 5. She watched a movie yesterday. A movie was watched by you yesterday b) Past Continuous: 1. I was watching a movie yesterday. A movie was being watched by me yesterday. 2. We were watching a movie yesterday. A movie was being watched by me yesterday. c) Past Perfect: 1. I had watched a movie three years ago. A movie had been watched by me three years ago. 3|Page

2. We had watched a movie three years ago. A movie had been watched by us three years ago. III) Future Tense a) Simple Future: 1. I will watch a movie tomorrow. A movie will be watched by me tomorrow. 2. We will watch a movie tomorrow. A movie will be watched by us tomorrow. b) Future Perfect: 1. You will have watched a movie by 6 pm tomorrow. A movie will have been watched by you by 6 pm tomorrow. 2. He will have watched a movie by 6 pm tomorrow. A movie will have been watched by him by 6 pm tomorrow. Voice Changing Rules in Bird's Eye View

4|Page

1. Our task had been completed before sunset. A. We completed our task before sunset. B. We have completed our task before sunset. C. We complete our task before sunset. D. We had completed our task before sunset. Answer: Option D 2. The boy laughed at the beggar. A. The beggar was laughed by the boy. B. The beggar was being laughed by the boy. C. The beggar was being laughed at by the boy. D. The beggar was laughed at by the boy. Answer: Option D 3. The boys were playing Cricket. A. Cricket had been played by the boys. B. Cricket has been played by the boys. C. Cricket was played by the boys. D. Cricket was being played by the boys. Answer: Option D 4. They drew a circle in the morning. A. A circle was being drawn by them in the morning. B. A circle was drawn by them in the morning. C. In the morning a circle have been drawn by them. D. A circle has been drawing since morning. Answer: Option B 5. They will demolish the entire block. A. The entire block is being demolished. B. The block may be demolished entirely. C. The entire block will have to be demolished by the D. The entire block will be demolished. Answer: Option D

5|Page

Narration (Direct and Indirect) In our speech, we often speak to the other person of something that was said to us by somebody. In other words, we often report a speech whether ours or someone else's. We do this in two ways. We either report the speech exactly as we had heard or said it without making any change. This is called Direct Speech. Example: The girl said to her mother, "My plate is empty." Or We may change the sentence that we had heard or said without changing its meaning and then report it. This is called Indirect Speech. Example: The girl said to her mother that her plate was empty. In the first example, the first part of the sentence which is before the comma is referred to as reporting verb and the part which is within inverted commas is called the reported speech. Note: While transforming from direct into indirect, we have made several changes in the sentence above: 1. We have removed the comma in the indirect sentence and put that in its place. 2. We have removed the inverted commas of the reported speech. 3. We have changed the my of the reported speech into her. 4. We have not used any capital letter in between the sentence unlike in the direct form where the reported speech always begins with a capital letter. Now, in order to bring these changes while converting from direct into indirect or vice-versa, there are several important but simple rules that need to be observed. They are : Changes in Tense and Verb While changing from direct to indirect we have to make different changes regarding tenses, verbs or helping verb. A short list is given below to have a look on those changes: Direct Speech Present Indefinite Present Continuous Present Perfect Present Perfect Continuous Past Indefinite Past Continuous Past Perfect Past Perfect Continuous

6|Page

Indirect Speech Past Indefinite Past Continuous Past Perfect Past Perfect Continuous Past Perfect Past Perfect Continuous No change No change

Important Rules a. If the reporting verb, i.e. the main verb in the first part of the sentence before comma is in the present or in the future tense, the tense of the verbs in the reported speech will not change. Example: Direct : Mummy says, "I shall write a letter." Indirect : Mummy says that she will write a letter. b. If the reporting verb is in the past tense, the tense of the verbs in the reported speech will be changed into their corresponding past tense. Example: Direct : The boy said to his friend. "I write everyday." (Present Indefinite) Indirect : The boy said to his friend that he wrote everyday. (Past Indefinite) Direct : The boy said to the teacher, "I am going there every day." (Present Continuous) Indirect : The boy said to the teacher that he was going there every day. (Past Continuous) c. If the reporting verb is in the past tense, the simple past in the reported speech may become the past perfect in the Indirect. Example: Direct : He said, "The horse died in the night." (Simple Past) Indirect : He said that the horse had died in the night. (Past Perfect) Direct : He said, "The man came at six." (Simple Past) Indirect : He said that the man had come at six. (Past Perfect) d. If the reporting verb is in the past tense, the past continuous in the reported speech changes into the past perfect continuous. Example: Direct : He said, "The man was coming." (Past Continuous) Indirect : He said that the man had been coming. (Past Perfect Continuous) Direct : He said, "Rain was falling yesterday." (Past Continuous) Indirect : He said that rain had been falling the previous day. (Past Perfect Continuous) 7|Page

Change of person The persons of the pronouns and of the verbs in the reported speech undergo changes when converted into indirect form. Thus: a. First person pronouns in the direct reporter speech change according to the subject of the reporting verb. Direct : He says, "I am going to Delhi." Indirect : He says that he is going to Delhi. Direct : Mummy says, "I will have to go." Indirect : Mummy says that she will have to go. b. Second person's pronoun changes according to the noun or pronoun coming after the reporting verbs. Second person changes to third, if there nothing mentioned of the second person in the reporting verb part. Direct

Indirect

You

He

Your

His/Her

You (Plural)

They

Your (Plural)

Their

You (Object)

Him/Her

You (Object)

Them

Yours (object)

Theirs

But if there you (second person) is mentioned in the direct speech, you in the indirect speech remain unchanged. Direct : Ram said to you, "You are a good boy." Indirect : Ram said to you that you were a good boy.

8|Page

Direct and Indirect Speech 1. Kiran asked me, "Did you see the Cricket match on television last night?" A. Kiran asked me whether I saw the Cricket match on television the earlier night. B. Kiran asked me whether I had seen the Cricket match on television the earlier night. C. Kiran asked me did I see the Cricket match on television the last night. D. Kiran asked me whether I had seen the Cricket match on television the last night. Answer: Option D 2. David said to Anna, "Mona will leave for her native place tomorrow." A. David told Anna that Mona will leave for her native place tomorrow. B. David told Anna that Mona left for her native place the next day. C. David told Anna that Mona would be leaving for her native place tomorrow. D. David told Anna that Mona would leave for her native place the next day.

4. He said to her, "What a cold day!" A. He told her that it was a cold day. B. He exclaimed that it was a cold day. C. He exclaimed sorrowfully that it was a cold day. D. He exclaimed that it was a very cold day. Answer: Option D 5. The tailor said to him, "Will you have the suit ready by tomorrow evening?" A. The tailor asked him that he will have the suit ready by the next evening. B. The tailor asked him that he would had the suit ready by the next evening. C. The tailor asked him if he would have the suit ready by the next evening. D. The tailor asked him if he will like to the suit ready by the next evening. Answer: Option C

6. He said to interviewer, "Could you please repeat the question?" A. He requested the interviewer if he 3. I said to him, "Why are you working so could please repeat the question. B. He requested the interviewer to please hard?" repeat the question. A. I asked him why he was working so C. He requested the interviewer to repeat hard. the question. B. I asked him why was he working so D. He requested the interviewer if he hard. C. I asked him why had he been working could repeat the question. so hard. D. I asked him why he had been working Answer: Option C so hard. Answer: Option D

Answer: Option A 9|Page

7. He said. "Be quite and listen to my words. A. He urged them to be quite and listen to his words. B. He urged them and said be quite and listen to his words. C. He urged they should be quite and listen to his words. D. He said you should be quite and listen to his words. Answer: Option A 8. He said to me, I have often told you not to play with fire. A. He said that he has often been telling me not to play with fire. B. He told me that he had often told me not to play with fire. C. He reminded me that he often said to me not to play with fire. D. He said to me that he often told me not to play with fire. Answer: Option B

10. Pawan said to me, "If I hear any news, I'll phone you." A. Pawan told me that if he heard any news, he will phone me. B. Pawan told me that if he will hear my news, he will phone me. C. Pawan told me that if he had heard any news, he would phone me. D. Pawan told me that if he heard any news, he would phone me. Answer: Option D 11. He said,"I cannot help you at present because I am myself in difficulty." A. He said that I cannot help you at present because I myself in difficulty. B. He said that he could not help me at present because he was himself in difficulty. C. He told that he could not help you at present because he was himself in difficulty. D. He asked that he could not help you at present because he was himself in difficulty. Answer: Option B

9. The Captain said to his men, "Stand at ease." A. The Captain urged his men to stand at ease. B. The Captain wanted his men to stand at ease. C. The Captain told his men that they should stand at ease. D. The Captain commanded his men to stand at ease.

12. He told her, "I want to meet your father": A. He told her that I want to meet your father. B. He told her that he wanted to meet her father. C. He told her that he wanted to meet your father. D. He told her that she wanted to meet her father.

Answer: Option D Answer: Option B

10 | P a g e

13. He says, ''I don't want to play any more." A. He says that he doesn't want to play any more. B. He says that I don't want to play any more. C. He says that I didn't want to play any more. D. He says that he didn't want to play any more. Answer: Option A 14. The government has announced, "Taxes will be raised": A. The government has announced that taxes would be raised. B. The government has announced that taxes would raised. C. The government has announced that taxes will be raised. D. The government has announced taxes will be raised. Answer: Option C 15. Mohan said, "We shall go to see the Taj in the moonlit night": A. Mohan said that we shall go to see the Taj in the moonlit night. B. Mohan told that we shall go to see the Taj in moonlit night. C. Mohan told that we should go to see the Taj in the moonlit night. D. Mohan said that they should go to see the Taj in moonlit night. Answer: Option D

16. The teacher said to Ram,"Congratulations ! Wish you success in life." A. The teacher congratulated Ram and said wish you success in life. B. The teacher wished congratulations and success in life to Ram. C. The teacher wished congratulations to Ram and wished him success in life. D. The teacher congratulated Ram and wished him success in life. Answer: Option D 17. The poor examine said,"O God, take pity on me." A. The poor examine prayed God to take pity on him. B. The poor examine, involving God, implored him to take pity on him. C. The poor examine exclaimed that God take pity on him. D. The poor examine asked God to take pity on him. Answer: Option A 18. "Where will you be tomorrow," I said, "in case I have to ring you ?" A. I asked where you will be the next day in case I will ring him. B. I asked where he would be the next day in case I had to ring him. C. I said to him where he will be in case I have to ring him. D. I enquired about his where about the next day in case I would have to ring up. Answer: Option B

11 | P a g e

19. Seeta said to me, " Can you give me your pen ?" A. Seeta asked me can I give her my pen. B. Seeta asked me if I can give me your pen. C. Seeta asked me if I could give her my pen. D. Seeta asked me if i could give her my pen. Answer: Option C 20. The father warned his son that he should be beware of him. A. The father warned his son, "beware of him !" B. The father warned his son, "Watch that chap !" C. The father warned his son, "Be careful about him." D. The father warned his son, "Don't fall into the trap."

23. "What about going for swim," he said, "It's quite fine now." A. He asked me what about going for a swim as it was quite fine then. B. He proposed going for swim as it was quite fine. C. He suggested going for a swim as it was quite fine. D. He advised me to go for a swim as it was quite fine. Answer: Option A

24. "You can't bathe in this sea," he said to me, "it's very rough." A. He said that I can't bathe in this sea because it's very rough. B. He said that you couldn't bathe in the sea if Answer: Option A it was very rough. C. He said that I couldn't bathe in that sea as it 21. Manna asked Rohan, "Have you sat in a was very rough. trolley bus before ?" D. He said that you can't bathe in this sea A. Manna asked Rohan had he sat in a trolley since it was very rough. bus earlier. B. Manna asked Rohan had he sat in a trolley Answer: Option C bus before. C. Manna asked Rohan if he sat on a trolley bus before. D. Manna asked Rohan if he has ever sat in a 25. Vissu said, "We passed by a beautiful lake when we went on a trip to Goa." trolley bus. A. Vissu said that they passed by a beautiful lake when they had gone on a trip to Goa. Answer: Option A B. Vissu said that they has passed by a 22. Farhan asked Geeta, "Could you lend me a beautiful lake when they went on a trip to Goa. hundred rupees until tomorrow ?" C. Vissu said that they had passed by a A. Farhan asked Geeta whether she could lend him a hundred rupees until tomorrow. beautiful lake when they had gone on a trip to Goa. B. Farhan asked Geeta whether she could lend him a hundred rupees until the next day. D. Vissu said they passed by a beautiful lake when they went on a trip to Goa. C. Farhan asked Geeta whether she could lend me a hundred rupees untill the next day. Answer: Option C D. Farhan asked whether Geeta could lend me a hundred rupees until the next day. Answer: Option B 12 | P a g e

26. He said to me, "I expect you to attend the function." A. He told me that he had expected me to attend the function. B. He told me that he expected me to attended the function. C. He told me that he expected me to have attended the function. D. He told me that he expected me to attend the function.

29. Suresh asked, " How long will it take to travel from Germany to South Africa ?" A. Suresh asked how long it will take to travel from Germany to South Africa. B. Suresh asked how long would it take to travel from Germany to South Africa. C. Suresh asked how long it would take to travel from Germany to South Africa. D. Suresh was asking how long must it take to travel from Germany to South Africa.

Answer: Option A

Answer: Option B

27. He said, "Why didn't you send your application to me ?" A. He enquired why I had not sent my application to him. B. He enquired why I did not send my application to him. C. He enquired why had I not sent my application to him. D. He enquired why did I not send my application to him.

30. "What did you see at the South Pole ?" Ashok asked Anil. A. Ashok asked Anil if he saw anything at the South Pole. B. Ashok asked Anil What he had seen at the South Pole. C. Ashok asked Anil What did he see at the South Pole. D. Ashok asked Anil that he saw anything at the South Pole.

Answer: Option A Answer: Option B 28. Dinesh asked, "Are you going to the party tomorrow, Rani ?" A. Dinesh asked whether Rani was going to the party the next day. B. Dinesh asked Rani whether you are going to the party the next day. C. Dinesh asked Rani whether she was going to the party the next day. D. Dinesh asked Rani are you going to the party tomorrow. Answer: Option C

31. My father said to us, "We should study and get a good division in the examination." A. My father said us that we should study and got a good division in the examination. B. My father told us that we should studied and got a good division in the examination. C. My father told us that we should study and get a good division in the examination. D. My father told to us that we should study and get a good division in the examination. Answer: Option B

13 | P a g e

Change of Speech 1. "If you don't keep quiet I shall shoot you", he said to her in a calm voice. A. He warned her to shoot if she didn B. He said calmly that I shall shoot you if you don C. He warned her calmly that he would shoot her if she didn D. Calmly he warned her that be quiet or else he will have to shoot her. Answer: Option C 2. I told him that he was not working hard. A. I said to him, "You are not working hard." B. I told to him, "You are not working hard." C. I said, "You are not working hard." D. I said to him, "He is not working hard." Answer: Option A 3. His father ordered him to go to his room and study. A. His father said, "Go to your room and study." B. His father said to him, "Go and study in your room." C. His father shouted, "Go right now to your study room" D. His father said firmly, "Go and study in your room." Answer: Option A

4. He said to his father, "Please increase my pocket-money." A. He told his father, "Please increase the pocket-money" B. He pleaded his father to please increase my pocket money. C. He requested his father to increase his pocket-money. D. He asked his father to increase his pocket-money. Answer: Option C 5. She said that her brother was getting married. A. She said, "Her brother is getting married." B. She told, "Her brother is getting married." C. She said, "My brother is getting married." D. She said, "My brother was getting married." Answer: Option C 6. The boy said, "Who dare call you a thief?" A. The boy enquired who dared call him a thief. B. The boy asked who called him a thief. C. The boy told that who dared call him a thief. D. The boy wondered who dared call a thief. Answer: Option A

14 | P a g e

7. She exclaimed with sorrow that was a very miserable plight. A. She said with sorrow, "What a pity it is." B. She said, "What a mystery it is." C. She said, "What a miserable sight it is." D. She said, "What a miserable plight it is." Answer: Option D 8. Dhruv said that he was sick and tired of working for that company. A. Dhruv said, "I am sick and tired of working for this company." B. Dhruv said, "He was tired of that company." C. Dhruv said to me, "I am sick and tired of working for this company." D. Dhruv said, "I will be tired of working for that company."

11. He exclaimed with joy that India had won the Sahara Cup. A. He said, "India has won the Sahara Cup" B. He said, "India won the Sahara Cup" C. He said, "How! India will win the Sahara Cup" D. He said, "Hurrah! India has won the Sahara Cup" Answer: Option D

Answer: Option A

12. The little girl said to her mother, "Did the sun rise in the East?" A. The little girl said to her mother that the sun rose in the East. B. The little girl asked her mother if the sun rose in the East. C. The little girl said to her mother if the sun rises in the East. D. The little girl asked her mother if the sun is in the East. Answer: Option B

9. "Are you alone, my son?" asked a soft voice close behind me. A. A soft voice asked that what I was doing there alone. B. A soft voice said to me are you alone son. C. A soft voice from my back asked If I was alone. D. A soft voice behind me asked If I was alone.

13. The man said, "No, I refused to confers guilt." A. The man emphatically refused to confers guilt. B. The man refused to confers his guilt. C. The man told that he did not confers guilt. D. The man was stubborn enough to confers guilt. Answer: Option A

Answer: Option D 10. She said to him, "Why don't you go today?" A. She asked him why he did not go that day. B. She said to him why he don C. She asked him not to go that day. D. She asked him why he did not go today. Answer: Option A 15 | P a g e

14. Nita ordered her servant to bring her cup of tea. A. Nita told her servant, "Bring a cup of tea." B. Nita said, "Bring me a cup of tea." C. Nita said to her servant, "Bring me a cup of tea." D. Nita told her servant, "Bring her that cup of tea." Answer: Option C

15. My cousin said, "My room-mate had snored throughout the night." A. my cousin said that her room-mate snored throughout the night. B. my cousin told me that her room-mate snored throughout the night. C. my cousin complained to me that her room-mate is snoring throughout the night. D. my cousin felt that her room-mate may be snoring throughout the night. Answer: Option A 16. "Please don't go away", she said. A. She said to please her and not go away. B. She told me to go away. C. She begged me not to go away. D. She begged that I not go away. Answer: Option C 17. She said to her friend, "I know where is everyone" A. She told that she knew where was everyone. B. She told her friend that she knew where was everyone. C. She told her friend that she knew where is everyone. D. She told her friend that she knows where was everyone. Answer: Option B

16 | P a g e

18. "Please don't go away", she said. A. She said to please her and not go away. B. She told me to go away. C. She begged me not to go away. D. She begged that I not go away. Answer: Option C 19. "Please don't go away", she said. A. She said to please her and not go away. B. She told me to go away. C. She begged me not to go away. D. She begged that I not go away. Answer: Option C

Spelling Check: Section 1 1. Find the correctly spelt word. A. Adulation B. Adlation C. Aduletion D. Addulation Answer: Option A

6. Find the correctly spelt word. A. Agglomeration B. Aglomeration C. Agglomaration D. Aglomaration Answer: Option A

11. Find the correctly spelt word. A. Cacophone B. Cacophoney C. Cacophoni D. Cacophony Answer: Option D

2. Find the correctly spelt word. A. Adulterate B. Adeldurate C. Adulterat D. Adultarate Answer: Option A

7. Find the correctly spelt word. A. Agrandize B. Aggrandize C. Aggranndice D. Aggradise Answer: Option B

12. Find the correctly spelt word. A. Capricious B. Cappricious C. Caprisious D. Carisuous Answer: Option A

3. Find the correctly spelt word. A. Adventitious B. Adventitous C. Adventitus D. Adventituous Answer: Option A

8. Find the correctly spelt word. A. Aberant B. Abbarant C. Aberrant D. Abberant Answer: Option C

13. Find the correctly spelt word. A. Centrefuge B. Centrifuse C. Centifuse D. Centrifuge Answer: Option D

4. Find the correctly spelt word. A. Adverisity B. Advercety C. Adversity D. Advercity Answer: Option C

9. Find the correctly spelt word. A. Abeyense B. Abayance C. Abeyence D. Abeyance Answer: Option D

14. Find the correctly spelt word. A. Chauvinist B. Chaubinist C. Chauviniste D. Chaubenist Answer: Option A

5. Find the correctly spelt word. A. Affedevit B. Afidevit C. Affidevit D. Affidavit Answer: Option D

10. Find the correctly spelt word. A. Abstemius B. Abstemaus C. Abstemious D. Abstemous Answer: Option C

15. Find the correctly spelt word. A. Compendioum B. Compendium C. Compandium D. Commppendium Answer: Option B

17 | P a g e

16. Find correctly spelt word. A. Connillatory B. Concilletry C. Conciliatory D. Concilletry Answer: Option C 17. Find the correctly spelt word. A. Deference B. Defferance C. Defference D. Defference Answer: Option A 18. Find the correctly spelt word. A. Delineate B. Deleneat C. Dileneate D. Deleneate Answer: Option A 19. Find the correctly spelt word. A. Deppricate B. Dapricate C. Depricate D. Deprecate Answer: Option D 20. Find the correctly spelt word. A. Desiccate B. Desicate C. Descicate D. Deccicate Answer: Option A

18 | P a g e

21. Find the correctly spelt word. A. Dicotomy B. Dicotemy C. Dichotomy D. Dechotomy Answer: Option C

26. Find the correctly spelt word: A. enemyty B. enemity C. enmity D. enmety Answer: Option C

22. Find the correctly spelt word. A. Embelis B. Embelesh C. Embellish D. Embelish Answer: Option C

27. Find the correctly spelt word: A. irelevant B. irrelavent C. irelevent D. irrelevant Answer: Option D

23. Find the correctly spelt word: A. livelihood B. livelyhood C. livlihood D. livelyhud Answer: Option A

28. Find the correctly spelt word. A. Benificial B. Beneficil C. Beneficial D. Benifical Answer: Option C

24. Find the correctly spelt word: A. foyere B. foayer C. foyer D. fouyer Answer: Option C

29. Find the correctly spelt word. A. Uniquely B. Internaly C. Coldlly D. Fortunatelly Answer: Option A

25. Find the correctly spelt word: A. lassivious B. lacivoius C. lascivious D. lasivious Answer: Option C

30. Find the correctly spelt word. A. Apprehension B. Apprehension C. Apprihension D. Aprihension Answer: Option B

31. Find the correctly spelt word. A. Affactionately B. Affectionately C. Afectionately D. Affectionnality Answer: Option B 32. Find the correctly spelt word. A. Averisious B. Avericious C. Avaricious D. Avarisious Answer: Option C 33. Find the correctly spelt word. A. Enterpreneur B. Entreorenure C. Entrepreneur D. Enterprenure Answer: Option C 34. Find the correctly spelt word. A. Promiscuous B. Promescuous C. Promiscuos D. Promescuous Answer: Option A

36. Find correctly spelt word. A. Bureacracy B. Bereaucracy C. Buereacracy D. Bureaucracy Answer: Option D 37. Find correctly spelt word. A. Millionair B. Millionnaire C. Milionaire D. Millionaire Answer: Option D 38. Find the correctly spelt word. A. Sychological B. Psychological C. Psykological D. Sykological Answer: Option B 39. Find the correctly spelt words. A. Efficient B. Treatmeant C. Beterment D. Employd Answer: Option A

40. Find the correctly spelt words. 35. Find correctly spelt word. A. Foreign A. Onomatopoeia B. Foreine B. Onomotopoei C. Fariegn C. Onomatopoia D. Forein D. Onamotipoei Answer: Option A Answer: Option A

19 | P a g e

41. Find the correctly spelt words. A. Ommineous B. Omineous C. Ominous D. Omenous Answer: Option C 42. Find the correctly spelt words. A. Pessenger B. Passenger C. Pasanger D. Pesanger Answer: Option B 43. Find the correctly spelt words. A. Benefitted B. Benifited C. Benefited D. Benefeted Answer: Option C 44. Find the correctly spelt words. A. Treachrous B. Trecherous C. Trechearous D. Treacherous Answer: Option D 45. Find the correctly spelt words. A. Forcast B. Forecaste C. Forcaust D. Forecast Answer: Option D

46. Find the correctly spelt words. A. Rigerous B. Rigourous C. Regerous D. Rigorous Answer: Option D

51. Find the correctly spelt words. A. Excessive B. Exccessive C. Exxcesive D. Excesive Answer: Option A

56. Find the correctly spelt words. A. Sepulchral B. Sepilchrle C. Sepalchrul D. Sepalchrl Answer: Option A

47. Find the correctly spelt words. A. Palete B. Palet C. Palate D. Pelate Answer: Option C

52. Find the correctly spelt words. A. Indipensable B. Indipenseble C. Indispansible D. Indispensable Answer: Option D

57. Find the correctly spelt words. A. Acommodation B. Accomodaton C. Accommodation D. Acomodation Answer: Option C

48. Find the correctly spelt words. A. Bouquete B. Bouquette C. Bouquet D. Boqquet Answer: Option C

53. Find the correctly spelt words. A. Humorous B. Ganerous C. Pupolous D. Maretorious Answer: Option A

58. Find the correctly spelt words. A. Faithfuly B. Sincerely C. Truely D. Affectionatly Answer: Option B

49. Find the correctly spelt words. A. Vetarinary B. Veteninary C. Vetinary D. Veterinary Answer: Option D

54. Find the correctly spelt words. A. Itinarery B. Itinerary C. Itenary D. Itinarary Answer: Option B

59. Find the correctly spelt words. A. Klaptomania B. Klepptomania C. Kleptemania D. Kleptomania Answer: Option D

50. Find the correctly spelt words. A. Chancelary B. Chancellery C. Chancelery D. Chancellary Answer: Option B

55. Find the correctly spelt words. A. Survaillance B. Surveillance C. Survellance D. Surveilance Answer: Option B

60. Find the correctly spelt words. A. Schedulle B. Schedeule C. Schdule D. Schedule Answer: Option D

20 | P a g e

61. Find the correctly spelt words. A. Skillful B. Skillfull C. Skilfull D. Skilpull Answer: Option A

65. Find the correctly spelt words. A. Dammage B. Damaige C. Dammege D. Damage Answer: Option D

69. Find the correctly spelt words. A. Inoculation B. Innoculation C. Inocculation D. Inocullation Answer: Option A

62. Find the correctly spelt words. A. Judicious B. Cancious C. Dilicous D. Gracous Answer: Option A

66. Find the correctly spelt words. A. Accomplish B. Acomplush C. Ackmplesh D. Accompalish Answer: Option A

70. Find the correctly spelt words. A. Velnerable B. Vulnarable C. Vulnerable D. Valnerable Answer: Option C

63. Find the correctly spelt words. A. Gaurantee B. Guarantee C. Garuntee D. Guaruntee Answer: Option B

67. Find the correctly spelt words. A. Puerrile B. Puerrille C. Purrile D. Puerile Answer: Option D

64. Find the correctly spelt words. A. Friming B. Burnning C. Running D. Fryng Answer: Option C

68. Find the correctly spelt words. A. Satelite B. Sattelite C. Satellite D. Sattellite Answer: Option C

21 | P a g e

Spelling Check: Section 2 1. Find the correctly spelt words. A. Simpal B. Bannar C. Pattren D. Modern Answer: Option D

6. Find the correctly spelt words. A. Hindrance B. Hinderrance C. Hindrence D. Hinderence Answer: Option A

11. Find the correctly spelt words. A. Swelte B. Filpant C. Licentious D. Puessile Answer: Option C

2. Find the correctly spelt words. A. Scripher B. Scripture C. Skripture D. Scriptur Answer: Option B

7. Find the correctly spelt words. A. Parallelled B. Parralleled C. Paralleled D. Paraleled Answer: Option C

12. Find the correctly spelt words. A. Grief B. Breif C. Recieve D. Diceive Answer: Option A

3. Find the correctly spelt words. A. Comitte B. Commitee C. Committee D. Comiittee Answer: Option C

8. Find the correctly spelt words. A. Lckadaisicle B. Lackdaisical C. Lackadisical D. Lackadaisical Answer: Option D

13. Find the correctly spelt words. A. Furnituer B. Exampel C. Medicine D. Sampal Answer: Option C

4. Find the correctly spelt words. A. Exaggerate B. Exeggrate C. Exagerate D. Exadgerate Answer: Option A

9. Find the correctly spelt words. A. Equanimity B. Equannimity C. Equanimmity D. Equinimity Answer: Option A

14. Find the correctly spelt words. A. Eflorescence B. Efllorescence C. Efflorescence D. Efflorascence Answer: Option C

5. Find the correctly spelt words. A. Asspersion B. Voluptuous C. Voguei D. Equestrain Answer: Option B

10. Find the correctly spelt words. A. Occured B. Occurad C. Ocurred D. Occurred Answer: Option D

15. Find the correctly spelt words. A. Exterminatte B. Inexpliccable C. Offspring D. Reffere Answer: Option C

22 | P a g e

16. Find the correctly spelt words. A. Occasion B. Occassion C. Ocasion D. Ocassion Answer: Option A 17. Find the correctly spelt words. A. Entrepreneur B. Entrapreneur C. Entrepraneur D. Enterprenuer Answer: Option A 18. Find out that word, which spelling is WRONG A. Appraise B. Commend C. Mentanence D. Behavior E. All correct Answer: Option C 19. Find out that word, which spelling is WRONG A. Passion B. Fashion C. Ration D. Tution E. All correct Answer: Option D 20. Find out that word, which spelling is WRONG A. Amature B. Manual C. Nephew D. Athletic E. All correct Answer: Option A

23 | P a g e

21. Find out that word, which spelling is WRONG A. Inflamable B. Musician C. Righteousness D. Negotiate E. All correct Answer: Option A

26. Find out that word, which spelling is WRONG A. Refuse B. Repute C. Despute D. Confuse E. All correct Answer: Option C

22. Find out that word, which spelling is WRONG A. Geography B. History C. Chemistry D. Commerce E. All correct Answer: Option E

27. Find out that word, which spelling is WRONG A. Urge B. Merge C. Perge D. Surge E. All correct Answer: Option C

23. Find out that word, which spelling is WRONG A. Immature B. Imminent C. Ilicit D. Imperative E. All correct Answer: Option C

28. Find out that word, which spelling is WRONG A. Boundary B. Exhibit C. Depresion D. Demonstration E. All correct Answer: Option C

24. Find out that word, which spelling is WRONG A. Quarreled B. Rebellious C. Commission D. Mirraculous E. All correct Answer: Option D 25. Find out that word, which spelling is WRONG A. Lenient B. Nationalism C. Overhaul D. Transfered E. All correct Answer: Option D

29. Find out that word, which spelling is WRONG A. Logical B. Ludicrucous C. Lovely D. Lonesome E. All correct Answer: Option B 30. Find out that word, which spelling is WRONG A. Periphery B. Advurtise C. Courteous D. Indefinite E. All correct Answer: Option B

31. Find out that word, which spelling is WRONG A. Dismiss B. Dispel C. Disservice D. Discribe E. All correct Answer: Option D

36. Find out that word, which spelling is WRONG A. Benificial B. Regular C. Despise D. Deprave E. All correct Answer: Option A

41. Find out that word, which spelling is WRONG A. Transmit B. Attribute C. Constitute D. Investegate E. All correct Answer: Option D

32. Find out that word, which spelling is WRONG A. Period B. Saggest C. Famous D. Reference E. All correct Answer: Option B

37. Find out that word, which spelling is WRONG A. Psychologist B. Psychaitrist C. Physiologist D. Psychoanalyst E. All correct Answer: Option B

42. Find out that word, which spelling is WRONG A. Peaceful B. Skillful C. Beautyful D. Hopeful E. All correct Answer: Option C

33. Find out that word, which spelling is WRONG A. Formulate B. Formidable C. Forman D. Format E. All correct Answer: Option C

38. Find out that word, which spelling is WRONG A. Conserve B. Reserve C. Diserve D. Preserve E. All correct Answer: Option C

43. Find out that word, which spelling is WRONG A. Numerous B. Nucleus C. Nuisence D. Numismatics E. All correct Answer: Option C

34. Find out that word, which spelling is WRONG A. Aristocracy B. Prophecy C. Beaureacuracy D. Democracy E. All correct Answer: Option C

39. Find out that word, which spelling is WRONG A. Filled B. Fulfiled C. Expelled D. Skilled E. All correct Answer: Option B

44. Find out that word, which spelling is WRONG A. Choclate B. Woolen C. Parliament D. Biscuit E. All correct Answer: Option A

35. Find out that word, which spelling is WRONG A. Artificial B. Aggrevate C. Forefront D. Negligence E. All correct Answer: Option B

40. Find out that word, which spelling is WRONG A. Seperation B. Desertion C. Rejoice D. Serenity E. All correct Answer: Option A

45. Find out that word, which spelling is WRONG A. Grammer B. Hammer C. Manner D. Stammer E. All correct Answer: Option A

24 | P a g e

46. Find out that word, which spelling is WRONG A. Deploy B. Emply C. Supply D. Reply E. All correct Answer: Option B

50. Find out that word, which spelling is WRONG A. Narrator B. Overseer C. Pretence D. Licence E. All correct Answer: Option E

54. Find out that word, which spelling is WRONG A. Permission B. Ambition C. Admision D. Submission E. All correct Answer: Option C

47. Find out that word, which spelling is WRONG A. Approach B. Compartment C. Restaurant D. Municipality E. All correct Answer: Option E

51. Find out that word, which spelling is WRONG A. Burglar B. Designation C. Controversy D. Ratificasion E. All correct Answer: Option D

55. Find out that word, which spelling is WRONG A. Collision B. Superstition C. Conversation D. Humilation E. All correct Answer: Option D

48. Find out that word, which spelling is WRONG A. Irritate B. Turmoil C. Ignorent D. Terrible E. All correct Answer: Option C

52. Find out that word, which spelling is WRONG A. Leisure B. Nuisance C. Opineon D. Mystery E. All correct Answer: Option C

56. Find out that word, which spelling is WRONG A. Gentalman B. Criticise C. Valuable D. Continuous E. All correct Answer: Option A

49. Find out that word, which spelling is WRONG A. Intellectual B. Opportunity C. Efficiency D. Responsibility E. All correct Answer: Option E

53. Find out that word, which spelling is WRONG A. Differed B. Suffered C. Offered D. Reffered E. All correct Answer: Option D

57. Find out that word, which spelling is WRONG A. Punctuation B. Fashion C. Pention D. Ration E. All correct Answer: Option C

25 | P a g e

Synonyms 1. Fostering : A. Safeguarding B. Neglecting C. Ignoring D. Nurturing Answer: Option D

6. Cease: A. Begin B. Stop C. Create D. Dull Answer: Option B

11. Recuperate : A. Rehabilitate B. Recover C. Reimburse D. Relocate Answer: Option B

2. Propel: A. Drive B. Jettison C. Burst D. Acclimatize E. Modify Answer: Option A

7. Pious: A. Religious B. Sympathetic C. Afraid D. Faithful Answer: Option A

12. Augury: A. Prophecy B. Astronomy C. Intention D. Future Answer: Option D

8. Cancel: A. Abolish B. Approve C. Allow D. Break Answer: Option A

13. Delectable: A. Nobel B. Enjoyable C. Delicious D. Sumptuous Answer: Option C

9. Abandon : A. Forsake 4. Stumbling Block: B. Keep A. Argument C. Cherish B. Frustration D. Enlarge C. Advantage Answer: Option A D. Hurdle E. Fallout 10. Germinate : Answer: Option D A. Decay B. Breed 5. Defer : C. Produce A. Indifferent D. Sprout B. Defy Answer: Option B C. Differ D. Postpone Answer: Option D

14. Dielectric : A. Argumentative B. Instructive C. Constructive D. Destructive Answer: Option A

3. Massive: A. Lump Sum B. Strong C. Little D. Gaping E. Huge Answer: Option E

26 | P a g e

15. Paltry : A. Insignificant B. Unfair C. Average D. Slovenly Answer: Option A

16. Disparage : A. Scatter B. Discriminate C. Belittle D. Waste E. Parable Answer: Option C 17. Alacrity : A. Briskness B. Fear C. Frankness D. Alarm E. None of These Answer: Option A 18. Busy : A. Active B. Occupied C. Preoccupied D. Diligent E. None of These Answer: Option B 19. Bitterness : A. Sourness B. Hoarseness C. Acrimony D. Aspersion E. None of These Answer: Option C

20. Sterile : A. Dry B. Barren C. Childless D. Arid E. None of These Answer: Option B 21. Altercation : A. Choice B. Charge C. Heated Dispute D. Distribution E. None of These Answer: Option C 22. Prohibit : A. Prescribe B. Forfeit C. Forbid D. Provide Answer: Option C 23. Stupid : A. Rude B. Dull C. Illiterate D. Obstinate Answer: Option B 24. Synopsis : A. Index B. Mixture C. Summary D. Puzzle Answer: Option C

27 | P a g e

25. Graphic : A. Moving B. Timid C. Obvious D. Instructive Answer: Option C

31. Diligent : A. Fool B. Unhappy C. Hardworking D. Disappointment Answer: Option C

37. Fugitive : A. Escaping B. Enduring C. Vain D. Weak Answer: Option A

26. Showy : A. Rowdy B. Hungry C. Greedy D. Gaudy Answer: Option D

32. Predominant : A. Lovable B. Important C. Vague D. Assurance Answer: Option B

38. Exponent : A. Meddler B. Cancel C. Advocate D. Horrid Answer: Option C

27. Annihilate : A. Destroy B. Entrain C. Forward D. Testify Answer: Option A

33. Utility : A. Benefit B. Usefulness C. Profitability D. Advantage Answer: Option B

39. Goad : A. Soothe B. Save C. Irritate D. Worship Answer: Option C

28. Pacify : A. Gather B. Allot C. Calm D. Remit Answer: Option C

34. Compassion : A. Kindness B. Similarity C. Contrast D. Cruelty Answer: Option A

40. Foment : A. Extirpate B. Provoke C. Isolation D. Abrasion Answer: Option B

29. Liberal : A. Broad-Minded B. Generous C. Visionary D. Imaginative Answer: Option B

35. Contentment : A. Craftiness B. Satisfaction C. Idleness D. Craving Answer: Option B

41. Imperceptible : A. Mean B. Obvious C. Humble D. Subtle Answer: Option D

30. Security : A. Boldness B. Safety C. Fearlessness D. Confidence Answer: Option B

36. Fiction : A. Imaginary B. Tamed C. Novel D. Wildness Answer: Option C

42. See : A. Talk B. Observe C. Stand D. Look Answer: Option B

43. Installed : A. Prepared B. Settled C. Started D. Manufactured E. Established Answer: Option E 44. Contain : A. Prevent B. Limit C. Hold D. Express E. Control Answer: Option C 45. Eventually : A. Suddenly B. Ultimately C. Quickly D. Abruptly E. Initially Answer: Option B

47. Turn : A. Shift B. Curve C. Move D. Rotate E. Fabricate Answer: Option D 48. Awakened : A. Enlightened B. Realized C. Shook D. Briefed Answer: Option A 49. Diversion : A. Amusement B. Distortion C. Deviation D. Change Answer: Option A 50. Pass : A. Qualify B. Die C. Cross D. Elongated Answer: Option C

46. Essential : A. Desirable B. Advisable C. Beneficial D. Unavoidable Answer: Option D 51. Wretched: A. Poor B. Foolish C. Insane D. Strained E. Transfer Answer: Option A

28 | P a g e

52. Duplicity : A. Repetition B. Artlessness C. Deceit D. Cleverness Answer: Option C

57. Cater : A. Advance B. Develop C. Provide D. Discover Answer: Option C

53. Vanguard : A. Race Driver B. Officer C. Flag Bearer D. Pioneer Answer: Option C

58. Redressed : A. Addressed B. Equalised C. Restored D. Redone E. Rearranged Answer: Option C

54. Fortitude : A. Similarity B. Courage C. Protection D. Safety Answer: Option B 55. Inadvertent : A. Ignorant B. Unexpected C. Unintentional D. Undisturbed Answer: Option C 56. Fidelity : A. Faithfulness B. Resourcefulness C. Strength D. Weakness Answer: Option A

59. Haggle: A. Postpone B. Accept C. Bargain D. Reject Answer: Option C 60. Abstruse: A. Awful B. Irrelevant C. Shallow D. Profound Answer: Option B 61. Yob: A. Aggressive B. Intelligent C. Annoyed D. Humble Answer: Option A

Antonyms 1. Foremost : A. Hindmost B. Unimportant C. Disposed D. Mature E. Premature Answer: Option B 2. Protects : A. Defends B. Deprives C. Deserts D. Devises E. Secures Answer: Option C 3. Beautiful : A. Wonderful B. Graceful C. Ugly D. Handsome E. Marvelous Answer: Option C 4. Terrible : A. Soothing B. Frightening C. Scaring D. Delectable E. Horrible Answer: Option D 5. Widely : A. Spaciously B. Succinctly C. Broad Based D. Limitedly E. Narrowly Answer: Option E 29 | P a g e

6. Last : A. End B. Grow C. Diminish D. Finish Answer: Option B 7. Mighty : A. Forcible B. Forceful C. Weak D. Tough E. Haughty Answer: Option C 8. Glory : A. Splendour B. Notoriety C. Fame D. Reputation E. Debasement Answer: Option E 9. Development : A. Retardation B. Retrogression C. Transgression D. Progress E. Impunity Answer: Option B 10. Modern : A. Up-Date B. Recent C. Cast D. Ancient Answer: Option D

11. Prosperous : A. Adverse B. Advanced C. Retarded D. Impecunious E. Affluent Answer: Option D

16. Haunts : A. Permeates B. Includes C. Repels D. Attracts E. Invigorates Answer: Option C

12. Impediments : A. Handicaps B. Obstacles C. Obstructions D. Hindrances E. Promoters Answer: Option E

17. Underhand : A. Hidden B. Surreptitious C. Obscure D. Exposed Answer: Option D

13. Divided : A. United B. Dignified C. Separated D. Isolated Answer: Option A 14. Differences : A. Resemblances B. Discrimination C. Dissimilarities D. Agreement Answer: Option A 15. Superiority : A. Seniority B. Juniority C. Inferiority D. Urbanity E. Posteriority Answer: Option C

18. Testify : A. Falsify B. Evidence C. Prove D. Confide E. Witness Answer: Option A 19. Extensive : A. Expensive B. Compact C. Concrete D. Abstract E. Narrow Answer: Option E 20. Discovered : A. Exposed B. Obscured C. Concealed D. Contended Answer: Option B

21. Abstain : A. Hoard B. Tolerate C. Forbear D. Begin Answer: Option D

27. Rigid : A. Merciful B. Generous C. Lenient D. Tolerant Answer: Option C

33. Consolidate : A. Weaken B. Expand C. Destroy D. Build Answer: Option A

39. Bright : A. Narrow B. Dull C. Coward D. Sharp Answer: Option B

22. Profusion : A. Aspersion B. Scarcity C. Aversion D. Confusion Answer: Option B

28. Benign : A. Malevolent B. Soft C. Friendly D. Unwise Answer: Option A

34. Trivial : A. Serious B. Intricate C. Calm D. Dainty Answer: Option A

40. Obeying : A. Ordering B. Following C. Refusing D. Contradicting E. Protesting Answer: Option A

23. Obscure : A. Implicit B. Obnoxious C. Explicit D. Pedantic Answer: Option C

29. Genuine : A. Unnatural B. Synthetic C. Artificial D. Sham Answer: Option D

35. Acquit : A. Retreat B. Convict C. Conceal D. Deprive Answer: Option B

24. Repulsive : A. Alluring B. Refulgent C. Effulgent D. Meek Answer: Option A

30. Singular : A. Dull B. Ancient C. Common D. Social Answer: Option C

36. Always : A. Disallow B. Never C. Daily D. Descend Answer: Option B

25. Ambiguous : A. Auxiliary B. Responsible C. Salvageable D. Clear Answer: Option D

31. Frugal : A. Extravagant B. Rich C. Miserable D. Happy Answer: Option A

37. Ancient : A. Repulsion B. Modern C. Disappear D. Departure Answer: Option B

26. Attractive : A. Repulsive B. Loathsome C. Abominable D. Repugnant E. None Of These Answer: Option A

32. Permit : A. Give B. Forbid C. Allow D. Preserve Answer: Option B

38. Bitter : A. Blunt B. Bold C. Timid D. Sweet Answer: Option D

30 | P a g e

41. Asked : A. Ordered B. Ignored C. Requested D. Directed E. Refused Answer: Option B 42. Commissioned : A. Started B. Closed C. Finished D. Terminated E. Unlocked Answer: Option B 43. Covering : A. Excluding B. Avoiding C. Exhibiting D. Hiding E. Protecting Answer: Option C

44. Amicable : A. Hostile B. Friendly C. Haughty D. Unpleasant Answer: Option A

47. Mitigate : A. Abate B. Aggravate C. Allay D. Alleviate Answer: Option B

50. Querulous: A. Disconnected B. Timorous C. Disputed D. Contented Answer: Option D

45. Aversion : A. Avoidable B. Awareness C. Hatred D. Liking Answer: Option D

48. Cacophonous: A. Loud B. Melodious C. Sonorous D. Harsh Answer: Option B

51. Scurrilous: A. Decent B. Ribald C. Abusive D. Mitigate Answer: Option A

46. Taciturn : A. Judge B. Silent C. Talkative D. Immense Answer: Option C

49. Gloomy: A. Discouraging B. Disguising C. Bright D. Tragic Answer: Option C

52. Ephemeral: A. Passing B. Sensuous C. Permanent D. Distasteful Answer: Option C

31 | P a g e

One Word Substitution One who is out to subvert a government

Anarchist

One who is recovering from illness

Convalescent

One who is all powerful

Omnipotent

One who is present everywhere

Omnipresent

One who knows everything

Omniscient

One who is easily deceived

Gullible

One who works for free

Volunteer

One who does not make mistakes

Infallible

One who can do anything for money

Mercenary

One who has no money

Pauper

One who changes sides

Turncoat

One who loves books

Bibliophile

One who can speak two languages

Bilingual

One who loves mankind

Philanthropist

One who hates mankind

Misanthrope

One who looks on the bright side of things

Optimist

One who looks on the dark side of things

Pessimist

One who doubts the existence of god

Agnostic

One who pretends to be what he is not

Hypocrite

One incapable of being tired

Indefatigable

One who helps others Good

Samaritan

One who copies from other writers

Plagiarist

One who hates women

Misogynist

One who knows many languages

Polyglot

One who is fond of sensuous pleasures

Epicure

One who thinks only of himself

Egoist

One who thinks only of welfare of women

Feminist

One who is indifferent to pleasure or pain

Stoic

32 | P a g e

One who is quite like a woman

Effeminate

One who has strange habits

Eccentric

One who speaks less

Reticent

One who goes on foot

Pedestrian

One who believes in fate

Fatalist

One who dies without a Will

Intestate

One who always thinks himself to be ill

Valetudinarian

A Government by the people

Democracy

A Government by a king or queen

Monarchy

A Government by the officials

Bureaucracy

A Government by the rich

Plutocracy

A Government by the few

Oligarchy

A Government by the Nobles

Aristocracy

A Government by one

Autocracy

Rule by the mob

Mobocracy

That through which light can pass

Transparent

That through which light cannot pass

Opaque

That through which light can partly pass

Translucent

A sentence whose meaning is unclear

Ambiguous

A place where orphans live

Orphanage

That which cannot be described

Indescribable

That which cannot be imitated

Inimitable

That which cannot be avoided

Inevitable

A position for which no salary is paid

Honorary

That which cannot be defended

Indefensible

Practice of having several wives

Polygamy

Practice of having several husbands

Polyandry

Practice of having one wife or husband

Monogamy

Practice of having two wives or husbands

Bigamy

That which is not likely to happen

Improbable

33 | P a g e

People living at the same time

Contemporaries

A book published after the death of its author

Posthumous

A book written by an unknown author

Anonymous

A life history written by oneself

Autobiography

A life history written by somebody else

Biography

People who work together

Colleagues

One who eats too much

Glutton

That which cannot be satisfied

Insatiable

One who questions everything

Cynic

A flesh eating animal

Carnivorous

A grass eating animal

Herbivorous

One who lives in a foreign country

Immigrant

To transfer one's authority to another

Delegate

One who is a newcomer

Neophyte

That which is lawful

Legal

That which is against law

Illegal

One who is unmarried

Celibate

A game in which no one wins

Draw

A study of man

Anthropology

A study of races

Ethnology

A study of the body

Physiology

A study of animals

Zoology

A study of birds

Ornithology

A study of ancient things

Archaeology

A study of derivation of words

Etymology

Murder of a human being

Homicide

Murder of a father

Patricide

Murder of a mother

Matricide

Murder of an brother

Fratricide

34 | P a g e

One Word Substitution 1. A person who renounces the world and practices selfdiscipline in order to attain salvation: A. Sceptic B. Ascetic C. Devotee D. Antiquarian Answer: Option B 2. One who abandons his religious faith: A. Apostate B. Prostate C. Profane D. Agnostic Answer: Option A 3. A hater of knowledge and learning: A. Bibliophile B. Philologist C. Misogynist D. Misologist Answer: Option D

5. Person who does not believe in the existence of God: A. Theist B. Heretic C. Atheist D. Fanatic Answer: Option C 6. A lady's umbrella is: A. Parasol B. Granary C. Epitaph D. Aviary Answer: Option A 7. Story of old time gods or heroes is: A. Lyric B. Epic C. Legend D. Romance Answer: Option C

9. One who believes in the power of fate: A. Fatalist B. Optimist C. Pessimist D. Parsimonious Answer: Option A

13. A person who is indifferent to pains and pleasure of life: A. Stoic B. Sadist C. Psychiatrist D. Aristocrat Answer: Option A

10. A person who loves everybody: A. Cosmopolitan B. Fratricide C. Atheist D. Aristocrat Answer: Option A

14. An act of speaking one's thoughts aloud when by oneself or regardless of any hearers, especially by a character in a play: A. Sinecure B. Soliloquy C. Pessimist D. Philanthropist Answer: Option B

11. One who cannot easily pleased: A. Cosmopolitan B. Frightening C. Fastidious D. Feminist Answer: Option C

15. A disease which spreads by contact: A. Incurable B. Infectious C. Contagious 12. The murder of D. Fatal 4. Commencement 8. A sad song: Answer: Option C Brother: of words with the A. Ditty A. Homicide same letter: 16. A man who B. Knell B. Regicide A. Pun rarely speaks the C. Dirge C. Fratricide B. Alliteration truth: D. Lay D. Suicide C. Transferred A. Crook Answer: Option C Answer: Option C epithet B. Liar D. Oxymoron C. Scoundrel Answer: Option B D. Hypocrite Answer: Option D

35 | P a g e

17. A remedy for all diseases: A. Narcotics B. Antiseptic C. Panacea D. Lyric Answer: Option C

22. A person who forsakes religion: A. Apostle B. Apotheosis C. Renegade D. Charlatan Answer: Option C

18. International destruction of racial groups: A. Matricide B. Regicide C. Genocide D. Homicide Answer: Option C

23. One who studies insect life: A. Geologist B. Zoologist C. Entomologist D. Botanist Answer: Option C

19. Custom of having many wives: A. Misogamy B. Bigamy C. Polygamy D. Monogamy Answer: Option C

24. A doctor who treats skin diseases: A. Ophthalmologist B. Dermatologist C. Paediatrician D. Cardiologist Answer: Option B

20. A pioneer of a reform movement: A. Apostle B. Apothecary C. Apotheosis D. Renegade Answer: Option A

25. A lady who remains unmarried: A. Spinster B. Artist C. Bachelor D. Misanthrope Answer: Option A

21. A person who pretends to have more knowledge or skill than he rally has: A. Crook B. Apotheosis C. Renegade D. Charlatan Answer: Option D

26. One who has good taste for food: A. Gourmet B. Curator C. Parasite D. Stoic Answer: Option A

36 | P a g e

27. One who knows two languages: A. Bigot B. Bigamy C. Bilingual D. Brittle Answer: Option C

32. One who able to use the right and left hands equally well: A. Sinister B. Ambidextrous C. Ambivalent D. Amateur 28. Government by Answer: Option B the representatives of the people: 33. Government by A. Autocracy rich: B. Democracy A. Oligarchy C. Socialism B. Aristocracy D. Anarchy Answer: Option B C. Pantisocracy D. Plutocracy 29. A disease that Answer: Option D spreads over a large area: 34. An independent A. Academic person or body B. Epidemic officially appointed C. Incorrigible to settle a dispute: D. Invincible Answer: Option B A. Arbiter B. Mediator 30. That which can C. Agent never be believed: provocateur D. Arbitrator A. Irrevocable B. Inevitable Answer: Option D C. Incredible D. Irritable 35. Murder of man: Answer: Option C A. Regicide B. Fratricide 31. That which cannot be avoided: C. Homicide D. Genocide A. Inevitable Answer: Option C B. Irreparable C. Incomparable D. Indisputable Answer: Option A

36. Use of force or threats to get someone to agree to something: A. Coercion B. Conviction C. Confession D. Cajolement Answer: Option A 37. Animal that feeds on plants: A. Carnivorous B. Herbivorous C. Insectivorous D. Graminivorous Answer: Option B 38. Group of stars: A. Orbit B. Constellation C. Solar system D. Comet Answer: Option B

40. To examine one's own thoughts and feelings: A. Meditation B. Retrospection C. Reflection D. Introspection Answer: Option D

44. A narrow piece of land connecting two large masses of land : A. Peninsula B. Isthmus C. Continent D. Gulf Answer: Option B 41. One indifferent 45. One who to art and literature collects coins as is : hobby : A. Aromatic A. Philatelist B. Critic B. Ornithologist C. Philistine C. Statistician D. Scholar D. Numismatist Answer: Option C Answer: Option D

42. Open to injury or Criticism : A. Invincible B. Vulnerable C. Naive D. Sensitive 39. Lasting only for Answer: Option B a very short time: A. Transparent 43. Government by B. Temporal a small group of all C. Temporary powerful persons : D. Temperate A. Oligarchy Answer: Option C B. Monarchy C. Democracy D. Anarchy Answer: Option A

37 | P a g e

One Word Substitutes: Section 1 1. Extreme old age when a man behaves like a fool A. Imbecility B. Senility C. Dotage D. Superannuation Answer: Option C

6. State in which the few govern the many A. Monarchy B. Oligarchy C. Plutocracy D. Autocracy Answer: Option B

7. A style in which a 2. That which cannot writer makes a be corrected display of his A. Unintelligible knowledge B. Indelible A. Pedantic C. Illegible B. Verbose D. Incorrigible C. Pompous Answer: Option D D. Ornate Answer: Option A 3. The study of ancient societies 8. List of the business A. Anthropology or subjects to be B. Archaeology considered at a C. History meeting D. Ethnology A. Schedule Answer: Option B B. Timetable C. Agenda 4. A person of good D. Plan understanding Answer: Option C knowledge and reasoning power 9. Leave or remove A. Expert from a place B. Intellectual considered C. Snob dangerous D. Literate A. Evade Answer: Option B B. Evacuate C. Avoid 5. A person who D. Exterminate insists on something Answer: Option B A. Disciplinarian B. Stickler C. Instantaneous D. Boaster Answer: Option B 38 | P a g e

10. A prima facie case is such A. As it seems at first sight B. As it is made to seem at first sight C. As it turns out to be at the end D. As it seems to the court after a number of hearings Answer: Option A 11. A person pretending to be somebody he is not A. Magician B. Rogue C. Liar D. Imposter Answer: Option D

14. One who possesses many talents A. Versatile B. Nubile C. Exceptional D. Gifted Answer: Option A 15. Words inscribed on tomb A. Epitome B. Epistle C. Epilogue D. Epitaph Answer: Option D

16. One who eats everything A. Omnivorous B. Omniscient C. Irrestible 12. A person who D. Insolvent knows many foreign Answer: Option A languages A. Linguist 17. Malafide case is B. Grammarian one C. Polyglot A. Which is D. Bilingual undertaken in a good Answer: Option A faith B. Which is 13. One who has undertaken in a bad little faith in human faith sincerity and C. Which is goodness undertaken after a A. Egoist long delay B. Fatalist D. Which is not C. Stoic undertaken at all D. Cynic Answer: Option B Answer: Option D

18. The custom or practice of having more than one husband at same time A. Polygyny B. Polyphony C. Polyandry D. Polychromy Answer: Option C

22. A drawing on transparent paper A. Red print B. Blue print C. Negative D. Transparency Answer: Option D

23. One who is not easily pleased by 19. Tending to move anything away from the centre A. Maiden or axis B. Mediaeval A. Centrifugal C. Precarious B. Centripetal D. Fastidious C. Axiomatic Answer: Option D D. Awry Answer: Option A 24. A remedy for all diseases 20. Teetotaller means A. Stoic A. One who abstains B. Marvel from theft C. Panacea B. One who abstains D. Recompense from meat Answer: Option C C. One who abstains from taking wine 25. One who is fond D. One who abstains of fighting from malice A. Bellicose Answer: Option C B. Aggressive C. Belligerent 21. A person D. Militant interested in Answer: Option A collecting, studying and selling of old 26. A small shop that things sells fashionable A. Antiquarian clothes, cosmetics, B. Junk-dealer etc. C. Crank A. Store D. Archealogist B. Stall Answer: Option A C. Boutique D. Booth Answer: Option C

39 | P a g e

27. That which cannot be read A. Negligible B. Illegible C. Ineligible D. Incorrigible Answer: Option B

32. One who cannot be corrected A. Incurable B. Incorrigible C. Hardened D. Invulnerable Answer: Option B

28. In a state of tension or anxiety or suspense A. Off balance B. Depressed C. Diffused D. On tenterhooks Answer: Option D

33. Be the embodiment or perfect example of A. Characterise B. Idol C. Personify D. Signify Answer: Option C

29. That which cannot be seen A. Insensible B. Intangible C. Invisible D. Unseen Answer: Option C

34. A person not sure of the existence of god A. Cynic B. Agnostic C. Atheist D. Theist Answer: Option B

30. To slap with a flat object A. Chop B. Hew C. Gnaw D. Swat Answer: Option D 31. Habitually silent or talking little A. Servile B. Unequivocal C. Taciturn D. Synoptic Answer: Option C

35. A paper written by hand A. Handicraft B. Manuscript C. Handiwork D. Thesis Answer: Option B 36. The act of violating the sanctity of the church is A. Blashphemy B. Heresy C. Sacrilege D. Desecration Answer: Option C

37. Something that can be heard A. Auditory B. Audio-visual C. Audible D. Audition Answer: Option C

43. The raison d'etre of a controversy is A. The enthusiasm with which it is kept alive B. The fitness with which participants handle it 38. A name adopted C. The reason or by an author in his justification of its writings existence A. Nickname D. The unending B. Pseudonym hostility the parties C. Nomenclature concerned have D. Title towards each other Answer: Option B Answer: Option C 39. Study of birds 44. A place where A. Orology B. Optology bees are kept in called C. Ophthalmology A. An apiary D. Ornithology B. A mole Answer: Option D C. A hive D. A sanctury 40. A place that Answer: Option A provides refuge A. Asylum 45. A religious B. Sanatorium discourse C. Shelter A. Preach D. Orphanage B. Stanza Answer: Option A C. Sanctorum 41. A child born after D. Sermon Answer: Option D death of his father A. Posthumous 46. Parts of a country B. Orphan behind the coast or a C. Bastard river's banks D. Progenitor Answer: Option A A. Isthmus B. Archipelago 42. The absence of C. Hinterland law and order D. Swamps A. Rebellion Answer: Option C B. Anarchy C. Mutiny D. Revolt Answer: Option B 40 | P a g e

47. Study of the evolution of man as an animal A. Archaeology B. Anthropology C. Chronology D. Ethnology Answer: Option B

52. A person who brings goods illegally into the country A. Importer B. Exporter C. Fraud D. Smuggler Answer: Option D

48. A person who speaks many languages A. Linguist B. Monolingual C. Polyglot D. Bilingual Answer: Option C

53. To take secretly in small quantities A. Robbery B. Pilferage C. Theft D. Defalcation Answer: Option B

49. One who does not believe in existence of god A. Egoist B. Atheist C. Stoic D. Naive Answer: Option B

54. To accustom oneself to a foreign climate A. Adapt B. Adopt C. Accustom D. Acclimatise Answer: Option D

50. A disease of mind causing an uncontrollable desire to steal A. Schizophrenia B. Claustrophobia C. Kleptomania D. Magolomania Answer: Option C

55. One who knows everything A. Literate B. Scholar C. Omnipotent D. Omniscient Answer: Option D

56. Detailed plan of journey 51. One who A. Travelogue sacrifices his life for a B. Travelkit cause C. Schedule A. Patriot D. Itinerary B. Martyr Answer: Option D C. Revolutionary D. Soldier Answer: Option B

57. Giving undue favours to one's own kith and kin A. Nepotism B. Favouritism C. Wordliness D. Corruption Answer: Option A

61. Incapable of being seen through A. Ductile B. Opaque C. Obsolete D. Potable Answer: Option B

65. A building for storing threshed grain A. Hangar B. Dockyard C. Store D. Granary Answer: Option D

69. An expression of mild disapproval A. Warning B. Denigration C. Impertinence D. Reproof Answer: Option D

62. One who does 58. Hater of learning not care for literature and knowledge or art A. Misologist A. Primitive B. Bibliophile B. Illiterate C. Misogynist C. Philistine D. Misanthropist D. Barbarian Answer: Option A Answer: Option C

66. Policeman riding on motorcycles as guards to a VIP A. Outriders B. Servants C. Commandos D. Attendants Answer: Option A

59. A person interested in reading books and nothing else A. Book-keepr B. Scholar C. Book-worm D. Student Answer: Option C

67. One who is determined to exact full vengeance for wrongs done to him A. Virulent B. Vindictive C. Usurer D. Vindicator Answer: Option B

70. One absorbed in his own thoughts and feelings rather than in things outside A. Scholar B. Recluse C. Introvert D. Intellectual Answer: Option C

63. A large sleepingroom with many beds A. Bedroom B. Dormitory C. Hostel D. Basement Answer: Option B

64. Continuing fight 60. A place where between parties, monks live as a families, clans, etc. secluded community A. Enmity A. Cathedral B. Feud B. Diocese C. Quarrel C. Convent D. Skirmish D. Monastery Answer: Option B Answer: Option D

41 | P a g e

68. Murder of a king A. Infanticide B. Matricide C. Genocide D. Regicide Answer: Option D

One Word Substitutes: Section 2 1. One who dabbles in fine arts for the love of it and not for monetary gains A. Connoisseur B. Amateur C. Professional D. Dilettante Answer: Option B 2. A school boy who cuts classes frequently is a A. Defeatist B. Sycophant C. Truant D. Martinet Answer: Option C 3. Ready to believe A. Credulous B. Credible C. Creditable D. Incredible Answer: Option A 4. Medical study of skin and its diseases A. Dermatology B. Endocrinology C. Gynealogy D. Orthopaedics Answer: Option A 5. A person who tries to deceive people by claiming to be able to do wonderful things A. Trickster B. Imposter C. Magician D. Mountebank Answer: Option A 42 | P a g e

6. A dramatic performance A. Mask B. Mosque C. Masque D. Mascot Answer: Option C

11. One who believes that all things and events in life are predetermined is a A. Fatalist B. Puritan C. Egoist D. Tyrant 7. One who does not Answer: Option A marry, especially as a religious obligation 12. Something that A. Bachelor relates to everyone in B. Celibate the world C. Vigin A. General D. Recluse B. Common Answer: Option B C. Usual D. Universal 8. That which is Answer: Option D perceptible by touch is 13. To walk with A. Contagious slow or regular Steps B. Contingent is to C. Tenacious A. Limp D. Tangible B. Stride Answer: Option D C. Pace D. Advance 9. Very pleasing to Answer: Option C eat A. Appetising 14. A style full of B. Palatable words C. Tantalising A. Verbose D. Sumptuous B. Pedantic Answer: Option B C. Rhetorical D. Abundant 10. The part of Answer: Option A government which is concerned with 15. Murder of a making of rules brother A. Court A. Patricide B. Tribunal B. Regicide C. Bar C. Homicide D. Legislature D. Fratricide Answer: Option D Answer: Option D

16. Having superior or intellectual interests and tastes A. Elite B. Highbrow C. Sophisticated D. Fastidious Answer: Option B 17. To cause troops, etc. to spread out in readiness for battle A. Disperse B. Deploy C. Collocate D. Align Answer: Option B 18. A voice loud enough to be heard A. Audible B. Applaudable C. Laudable D. Oral Answer: Option A 19. A light sailing boat built specially for racing A. Canoe B. Yacht C. Frigate D. Dinghy Answer: Option B 20. One who is in charge of museum A. Curator B. Supervisor C. Caretaker D. Warden Answer: Option A

21. A government by the nobles A. Aristocracy B. Democracy C. Autocracy D. Bureaucracy Answer: Option A 22. One who is honourably discharged from service A. Retired B. Emeritus C. Relieved D. Emancipated Answer: Option B

24. The policy of extending a country's empire and influence A. Communism B. Internationalism C. Capitalism D. Imperialism Answer: Option D

27. A fixed orbit in space in relation to earth A. Geological B. Geo-synchronous C. Geo-centric D. Geo-stationary Answer: Option D

25. Of outstanding significance A. Monumental B. Rational C. Ominous D. Evident Answer: Option A

28. That which cannot be believed A. Incredible B. Incredulous C. Implausible D. Unreliable Answer: Option A

26. Code of 23. Present opposing diplomatic etiquette arguments or and precedence evidence A. Statesmanship A. Criticise B. Formalism B. Rebuff C. Hierarchy C. Reprimand D. Protocol D. Rebut Answer: Option D Answer: Option D

43 | P a g e

29. To issue a thunderous verbal attack A. Languish B. Animate C. Fulminate D. Invigorate Answer: Option C

Idioms & Phrases: Section 1 1. To make clean breast of A. To gain prominence B. To praise oneself C. To confess without of reserve D. To destroy before it blooms E. None of these Answer: Option C

6. To cry wolf A. To listen eagerly B. To give false alarm C. To turn pale D. To keep off starvation E. None of these Answer: Option B

2. To keeps one's temper A. To become hungry B. To be in good mood C. To preserve ones energy D. To be aloof from E. None of these Answer: Option B

7. To end in smoke A. To make completely understand B. To ruin oneself C. To excite great applause D. To overcome someone E. None of these Answer: Option B

3. To catch a tartar A. To trap wanted criminal with great difficulty B. To catch a dangerous person C. To meet with disaster D. To deal with a person who is more than one's match E. None of these Answer: Option B

8. To be above board A. To have a good height B. To be honest in any business deal C. They have no debts D. To try to be beautiful E. None of these Answer: Option B

4. To drive home A. To find one's roots B. To return to place of rest C. Back to original position D. To emphasize E. None of these Answer: Option D 5. To have an axe to grind A. A private end to serve B. To fail to arouse interest C. To have no result D. To work for both sides E. None of these Answer: Option A 44 | P a g e

9. To put one's hand to plough A. To take up agricultural farming B. To take a difficult task C. To get entangled into unnecessary things D. Take interest in technical work E. None of these Answer: Option B 10. To pick holes A. To find some reason to quarrel B. To destroy something C. To criticize someone D. To cut some part of an item E. None of these Answer: Option C

11. To leave someone in the lurch A. To come to compromise with someone B. Constant source of annoyance to someone C. To put someone at ease D. To desert someone in his difficulties E. None of these Answer: Option D 12. To play second fiddle A. To be happy, cheerful and healthy B. To reduce importance of one's senior C. To support the role and view of another person D. To do back seat driving E. None of these Answer: Option C 13. To beg the question A. To refer to B. To take for granted C. To raise objections D. To be discussed E. None of these Answer: Option B 14. A black sheep A. An unlucky person B. A lucky person C. An ugly person D. A partner who takes no share of the profits E. None of these Answer: Option E 15. A man of straw A. A man of no substance B. A very active person C. A worthy fellow D. An unreasonable person E. None of these Answer: Option A

45 | P a g e

16. To smell a rat A. To see signs of plague epidemic B. To get bad small of a bad dead rat C. To suspect foul dealings D. To be in a bad mood E. None of these Answer: Option C 17. To hit the nail right on the head A. To do the right thing B. To destroy one's reputation C. To announce one's fixed views D. To teach someone a lesson E. None of these Answer: Option A 18. To set one's face against A. To oppose with determination B. To judge by appearance C. To get out of difficulty D. To look at one steadily E. None of these Answer: Option A 19. Sobhraj could be easily arrested because the police were tipped off in an advance. A. Toppled over B. Bribed C. Given advance information D. Threatened Answer: Option C 20. I met him after a long time, but he gave me the cold shoulder. A. scolded me B. insulted me C. abused me D. ignored me Answer: Option D

21. He passed himself off as a noble man. A. Was regarded as B. Pretended to be C. Was thought to be D. Was looked upon Answer: Option B

26. He was undecided. He let the grass grow under his feet. A. loitered around B. stayed out C. sat unmoving D. moved away Answer: Option A

22. This matter has been hanging fire for the last many months and must therefore be decided one way or the other. A. going on slowly B. hotly debated C. stuck up D. ignored Answer: Option A

27. Although he has failed in the written examination, he is using backstairs influence to get the job. A. Political influence B. Backing influence C. Deserving and proper influence D. Secret and unfair influence Answer: Option D

23. In the armed forces, it is considered a great privilege to die in harness. A. die on a horse back B. die in the battlefield C. die while still working D. die with honour Answer: Option C

28. Companies producing goods play to the gallery to boost their sales. A. advertise B. cater to the public taste C. attempt to appeal to popular taste D. depend upon the public for approval Answer: Option C

24. The cricket match proved to be a big draw. A. a keen contest B. a huge attraction C. a lovely spectacle D. a game without any result Answer: Option B

29. Since he knew what would happen, he should be left to stew in his own juice. A. Make a stew B. Boil C. Suffer in his own juice D. Suffer for his own act Answer: Option D

25. When he heard that he had once again not been selected he lost heart. A. became desperate B. felt sad C. became angry D. became discouraged Answer: Option D

46 | P a g e

30. The project did not appear to hold out bright prospects. A. highlight B. show C. offer D. promise Answer: Option C

31. I am afraid he is burning the candle at both ends and ruining his life. A. wasting his money B. becoming overgenerous C. overtaxing his energies D. losing his objectives Answer: Option C

36. The case was held over due to the great opposition to it. A. stopped B. postponed C. dropped D. cancelled Answer: Option B

32. The university will have to shelve its plans for expansion in view of present situation A. cancel B. discuss C. reconsider D. postpone Answer: Option D

37. In the organized society of today no individual or nation can plough a lonely furrow. A. remain unaffected B. do without the help of others C. survive in isolation D. remain non-aligned Answer: Option B

33. Do no trust a man who blows his own trumpet A. flatters B. praises others C. admonishes others D. praises himself Answer: Option D

38. It was he who put a spoke in my wheel. A. tried to cause an accident B. helped in the execution of the plan C. thwarted in the execution of the plan D. destroyed the plan Answer: Option C

34. He is out and out a reactionary. A. no more B. thoroughly C. in favour of D. deadly against Answer: Option B

39. It is no longer easy to strike gold in Shakespeare's research since much work has already been done on him. A. Hit a golden spot B. Come across gold C. Come across the word "gold" D. Uncover or find a valuable line of argument or information Answer: Option D

35. I did not mind what he was saying, he was only through his hat. A. talking nonsense B. talking ignorantly C. talking irresponsibly D. talking insultingly Answer: Option A

47 | P a g e

40. Sanjay is very different about passing the Civil Services Examination this year. A. Dead sure of B. Very hopeful of C. Lacking self-confidence about D. Reasonably certain of Answer: Option C

41. Why must you look a gift horse in the mouth? A. welcome a gift B. find fault with a gift C. be fastidious D. examine a gift carefully Answer: Option B

46. He is an interesting speaker but tends to go off at a tangent. A. change the subject immediately B. forget things in between C. go on at great length D. become boisterous Answer: Option A

42. Dowry is a burning question of the day. A. a widely debated issue B. a dying issue C. a relevant problem D. an irrelevant issue Answer: Option A

47. He sold his house for a song. A. at a reasonable price B. at a discount C. very cheaply D. at a premium Answer: Option C

43. Leaders should not only make speeches they should also be prepared to to bell the cat. A. To take lead in danger. B. To tie bell to a cat's neck C. To be alert of the enemy D. To make noise Answer: Option A 44. The popularity of the yesterday's superstar is on the wane. A. at its peak B. at rock bottom C. growing more D. growing less Answer: Option D 45. He is leaving the country for a good. A. for better prospects B. for a good cause C. for ever D. for others good Answer: Option C

48 | P a g e

48. Despite the trust bestowed on the minister he turned out to be a snake in the grass during the revolution. A. a secret enemy B. a treacherous person C. an unforeseen danger D. an unexpected misfortune Answer: Option A 49. Women should be paid the same as men when they do the same job, for, surely what is sauce for the goose is sauce for the gander. A. What is thought suitable pay for a man should also be for a woman B. Goose and the gender eat the same sauce C. Both goose and gander should be equally treated D. The principle of equal treatment should be implemented Answer: Option A

50. The party stalwarts have advised the President to take it lying down for a while. A. to be cautious B. to be on the defensive C. to take rest D. to show no reaction Answer: Option D 51. The thief took to his heels when he saw a policeman on the beat. A. had some pain in his heels B. ran away from the scene C. confronted the policeman D. could not decide what to do Answer: Option B

55. The teacher warned the student once for all that no mischief shall be tolerated in the class. A. authoritatively B. finally C. angrily D. coldly Answer: Option B 56. I hope it will not put you out if I am late. A. harm you B. irritate you C. please you D. worry out Answer: Option B

52. He struck several bad patches before he made good. A. came across bad soil B. Had a bad time C. Went through many illness D. Had many professional difficulties Answer: Option D

57. Govind has left his country for good. A. for better opportunities B. on an important mission C. forever D. to return soon Answer: Option C

53. The accounts of the murder made her flesh creep. A. Made her sad B. Surprised her C. Made her cry bitterly D. Fill her with horror Answer: Option D

58. Sanjay was the real power behind the throne and all politicians were aware of this. A. The person who had the real control and power B. The acknowledged leader C. The person who controlled the monarch D. The person who advised the queen Answer: Option A

54. How long will the people put up with the increasing economic hardships? A. welcome B. take easily C. remain satisfied with D. tolerate Answer: Option D

49 | P a g e

59. His friends let him down. A. failed B. comforted C. succeeded D. stood by Answer: Option A

60. I felt like a fish out of water among all those business tycoons. A. troubled B. stupid C. uncomfortable D. inferior Answer: Option C 61. Some people now wonder whether we just pay lip-service or genuinely subscribe to democracy. A. pay oral tribute B. attach no value C. remain indifferent D. show only outward respect Answer: Option D 62. Don't thrust your nose into my affairs. A. Advise me about B. Be in opposition C. Deal with D. Meddle officiously in Answer: Option D 63. He visits the doctor off and on. A. everyday B. regularly C. never at all D. occasionally Answer: Option D 64. The recent film "Secular India" has tried to keep the pot of Muslim women's Bill boiling A. Earning enough to keep body and soul together B. Keep a controversy alive C. Boil the bill D. Boil something in a pot Answer: Option B 50 | P a g e

65. The robber took to his heels when the police arrived. A. opened fire B. hid himself C. ran off D. surrendered Answer: Option C 66. You cannot have your cake and eat it too. A. Enjoy forever B. Have it both ways C. Run away from responsibility D. Absolve yourself of guilt Answer: Option B 67. The rebels held out for about a month A. waited B. retreated C. bargained D. resisted Answer: Option D 68. The new economic policy is likely to run into rough weather. A. create problems B. encounter difficulties C. confuse matters D. makes things difficult Answer: Option B 69. The old father brought home the moral of unity by asking each of his sons to break the bundle of sticks A. emphasized B. voiced C. declared D. suggested Answer: Option A 70. If you give John all your money, you are likely to burn your fingers. A. be unhappy B. be happy C. suffer D. be ill Answer: Option C

Idioms & Phrases: Section 2 1. I am sure they will fight tooth and nail for their rights. A. with all their might B. without any other weapon C. resorting to violence D. very cowardly Answer: Option A 2. We were in hurry. The road being zigzag we had to cut off a corner to reach in time. A. To cut a portion of the road B. To take a short cut C. To go fast D. To take an alternative route Answer: Option B 3. Discipline is on the wane in schools and colleges these days. A. declining B. increasing C. spreading D. spiraling Answer: Option A 4. In spite of the efforts of all peace loving people, world peace is still a far cry. A. an impracticable idea B. an abstract idea C. out of reach D. a long way off Answer: Option D

51 | P a g e

5. I felt that it was a tall order to expect Monisha to go home alone at twelve in the night. A. Difficult B. Too much C. Customary D. Simple Answer: Option B 6. I cannot get along with a man who plays fast and loose. A. behaves in an unreliable and insincere way B. has a loose tongue C. lives a life of ease and luxury D. does not know how to behave himself Answer: Option A 7. There is no love lost between two neighbours. A. close friendship B. cool indifference C. intense dislike D. a love hate relationship Answer: Option C 8. The question of abolition of private property is still a moot point. A. undecided B. uncertain C. unknown D. not clear Answer: Option A

9. This regular absenteeism is a bad business since on work is being completed. A. Bad for business B. Non-business like C. An unfortunate event D. Creating ill-will Answer: Option C 10. When he tells stories about himself, he is inclined to draw the long bow. A. understate B. get emotional C. exaggerate D. get excited Answer: Option C 11. Vikram said that he has got a problem to square up with the manager. A. consider B. discuss C. settle D. workout Answer: Option C 12. Although both the parents are running the show for the last ten years but their business is now on its last legs. A. About to fructify B. About to perish C. About to produce results D. About to take off Answer: Option B 13. The time is running out, you must look sharp. A. be careful B. make haste C. be punctual D. be sensible Answer: Option B 52 | P a g e

14. He is in the habit of throwing dust in his superior's eyes A. To put sand into eyes B. To make blind C. o deceive D. o show false things Answer: Option C 15. In modern democratic societies lynch law seems to have become the spheres of life. A. law of the mob B. law of the underworld C. law of the constitution D. law of the parliament Answer: Option A 16. He was in high spirits when I met him in the restaurant. A. He was in a drunken state B. He was very cheerful C. He talked incoherently D. He was deeply engrossed in thoughts Answer: Option B 17. In his youth, he was practically rolling in money. A. Spending more than his income B. Borrowing money liberally C. Very rich D. Wasting a lot of money Answer: Option C 18. Aya had a chequered career. Since I first knew him as an office assistant in the insurance company. A. Had a variety of jobs and experiences B. A career which helped him make lot of money C. A career where he signed a lot of cheques D. Did odd jobs Answer: Option A

19. The sight of the accident made my flesh creep. A. draw my attention B. confused me C. frightened me D. worried me E. None Answer: Option C

24. The autographed bat from the famous cricketer Sunil Gavaskar is worth a jew's eye. A. Not a worthy possession B. unnecessary C. A costly items D. A possession of high value Answer: Option D

20. There is no hard and fast rule regarding this subject. A. rule that is difficult B. rule that is fat changing C. rule that cannot be broken or modified D. rule that can be broken or modified Answer: Option C

25. The speaker gave a bird's eye view of the political conditions in the country. A. a personal view B. a general view C. a biased view D. a detailed presentation Answer: Option B

21. He went to his friend's house in the evening as was his wont. A. as usual B. as he wanted C. as his want was D. as he wanted that day Answer: Option A 22. Why do you wish to tread on the toes? A. To give offence to them B. To follow them grudgingly C. To treat them indifferently D. To be kicked by them Answer: Option A 23. He intends setting up as a lawyer in the adjoining district. A. To establish himself B. To migrate C. To join D. To settle Answer: Option A 53 | P a g e

26. He resigned the post of his own accord. A. which he liked B. according to his convenience C. voluntarily and willingly D. according to his judgment Answer: Option C 27. As a politician he is used to being in the limelight all the time. A. giving speeches B. the object of admiration C. the centre of attraction D. an object of public notice Answer: Option C 28. I ran out of money on my European tour. A. exhausted my stock of B. did not have enough C. lost D. carried a lot Answer: Option A

29. Madhuri might scream blue murder, but I feel Deepali should get the promotion since she is better qualified for the job. A. Someone has been murdered with some blue liquid B. Someone is being murdered and has become blue C. Suffer from persecution complex D. Make a great deal of noise and object vehemently Answer: Option D 30. Having sold off his factory, he is now a gentleman at large. A. Has no serious occupation B. Is living comfortably C. Is respected by everybody D. Is held in high esteem Answer: Option A 31. Though he has lot of money, yet all his plans are built upon sand. A. established on insecure foundations B. based on inexperience C. resting on cheap material D. resting on immature ideas Answer: Option D 32. His plan was so complicated that it floored his listeners. A. entertained B. puzzled C. annoyed D. encouraged Answer: Option B 33. There has been bad blood between the two communities even before shouting. A. Impure blood B. Ill feeling C. bloody fights D. Quarrels Answer: Option B

54 | P a g e

34. The curious neighbours were disappointed as the young couple's quarrel was just a storm in a tea cup. A. violent quarrel B. fuss about a trifle C. brittle situation D. quarrel about tea cups Answer: Option B 35. Mohan always keeps himself to himself. A. Is too busy B. Is selfish C. Is unsociable D. Does not take sides Answer: Option C 36. While the ladies continued their small talk in the drawing room, I felt bored. A. whispering B. backbiting C. gossip D. light conversation Answer: Option D 37. My car broke down on way to the railway station. A. stopped B. met with an accident C. ran out of petrol D. failed to work Answer: Option A 38. My father strained every nerve to enable me to get settled in life. A. worked very hard B. spent a huge amount C. tried all tricks D. bribed several persons Answer: Option A

39. He is always picking holes in every project. A. creating problems in B. finding fault with C. suggesting improvement in D. asking irrelevant questions on Answer: Option B

44. Don't lose patience, things will improve by and by A. soon B. finally C. gradually D. unexpectedly Answer: Option C

40. The die is cast and now let us hope for the best. A. project is over B. decision is made C. death is inevitable D. cloth has been dyed Answer: Option B

45. Do not imagine that Dharmendra is really sorry that his wife died. Those are only crocodile tears. A. Pretended sorrow B. Tears a crocodile C. A weeping crocodile D. Mild regret Answer: Option A

41. Pt. Nehru was born with a silver spoon in his mouth. A. born in a middle class family B. born in a wealthy family C. born in a royal family D. born in a family of nationalists Answer: Option B 42. The arrival of the mother-in-law in the family proved a rift in the lute. A. caused unnecessary worries B. brought about disharmony C. caused a pleasant atmosphere D. brought about a disciplined atmosphere Answer: Option B 43. The prince did not take after the king. A. run after B. follow C. precede D. resemble Answer: Option D

55 | P a g e

46. The P.M has pulled up Orissa partymen. A. elevated B. rewarded C. punished D. reprimanded Answer: Option D 47. He did me a good turn by recommending me for the post of Vice Principal. A. returned my kindness B. did an act of kindness C. improved my prospects D. became suddenly good Answer: Option B 48. He believes in the policy of making hay while the sun shines. A. giving bribes to get his work done B. seeking advice from one and all C. helping those who help him D. making the best use of a favourable situation Answer: Option D

49. His friends advised him to be fair and square in his dealings. A. Careful B. Considerate C. Polite D. Upright Answer: Option D

55. I requested him to put in a word for me. A. introduce B. assist C. support D. recommend Answer: Option D

50. It is high time that India did something about the population problem. A. already late B. appropriate time C. desired occasion D. auspicious moment Answer: Option A

56. The dacoit murdered the man in cold blood. A. coldly B. boldly C. ruthlessly D. deliberately Answer: Option C

51. He is always standing up for the weak and oppressed. A. Boosting the claims of B. Championing the cause of C. Seeking help of others for D. Moving about with Answer: Option B 52. We should give a wide berth to bad characters. A. give publicity to B. publicly condemn C. keep away from D. not sympathize with Answer: Option C 53. Turban is in vogue in some communities. A. in fashion B. out of use C. vaguely used D. never used Answer: Option A 54. The old man was cut to the quick when his rich son refused to recognize him. A. surprised B. hurt intensely C. annoyed D. irritated Answer: Option B

56 | P a g e

57. He has built a big business empire by his sharp practices. A. extreme hard work B. keen business skills C. dishonest dealings D. sharp intelligence Answer: Option C 58. The secretary and the treasurer are hand in glove with each other. A. very good friends B. constantly fighting C. associates in some action D. suspicious of each other Answer: Option A 59. He never liked the idea of keeping his wife under his thumb and so he let her do what she liked. A. Pressed down B. Unduly under control C. Below his thumb D. Under tyrannical conditions. Answer: Option B 60. It is time that professors came down from their ivory towers and studied the real needs of the students. A. Detachment and seclusion B. A tower made of ivory C. Prison D. Dream lands Answer: Option A

61. You have to be a cool customer and be patient if you want to get the best buys. A. Be calm and not be excitable B. Have a cool head C. Be uncommunicative D. Be choosy Answer: Option A

66. Harassed by repeated acts of injustice, he decided to put his foot down. A. not to yield B. resign C. to accept the proposal unconditionally D. withdraw Answer: Option A

62. There was no opposition to the new policy by the rank and file of the Government. A. the official machinery B. the ordinary members C. the majority D. the cabinet ministers Answer: Option B

67. The class could not keep a straight face on hearing the strange pronunciation of the new teacher. A. remain silent B. remain serious C. remain mute D. remain disturbed Answer: Option B

63. The clerk wiped the nose of his employer by submitting a false bill and was dismissed from his job. A. Cleaned the nose B. Cheated C. Abused D. Slapped Answer: Option B

68. The parliamentary inquiry into the Bofors deal did not bring to light any startling facts. A. Prove B. Probe C. Highlight D. Disclose Answer: Option D

64. I have a bone to pick with you in this matter. A. Am in agreement B. Am angry C. Am indebted D. Will join hands Answer: Option B 65. The new C.M stuck his neck out today and promised 10kgs. free wheat a month for all rural families. A. took an oath B. took a risk C. extended help D. caused embarrassment Answer: Option B 57 | P a g e

69. His speech went down well with the majority of the audience. A. found acceptance with B. was attentively listened to by C. was appreciated by D. was applauded by Answer: Option C 70. Rohit has bitten off more than he chew. A. Is trying to do much B. Is very greedy C. Is always hungry D. Has little regard for others Answer: Option A

Idioms & Phrases: Section 3 1. The detective left no stone unturned to trace the culprit. A. took no pains B. did very irrelevant things C. resorted to illegitimate practices D. used all available means Answer: Option D

5. He went back on his promise to vote for me A. withdrew B. forgot C. reinforced D. supported Answer: Option A

2. The authorities took him to task for his negligence. A. gave him additional work B. suspended his assignment C. reprimanded him D. forced him to resign Answer: Option C

6. The old beggar ran amuck and began to throw stones at the passerby. A. became desperate B. ran about wildly C. become annoyed D. felt disgusted Answer: Option B

3. In spite of the immense pressure exerted by the militants, the Government has decided not to give in. A. accede B. yield C. oblige D. confirm Answer: Option B

7. I cannot conceive of a time when I was without a refrigerator A. Imagine B. Give birth C. Understand D. Depend Answer: Option A

4. Their business is now on its last legs. A. About to fructify B. About to perish C. About to produce results D. About to take off Answer: Option B

58 | P a g e

Verbal Analogies 1. Select the pair which has the same relationship. DIVA:OPERA A. producer:theatre B. director:drama C. conductor:bus D. thespian:play Answer: Option D

5. Select the pair which has the same relationship. LIGHT:BLIND A. speech:dumb B. language:deaf C. tongue:sound D. voice:vibration Answer: Option A

9. Select the pair which has the same relationship. INDIGENT:WEALTHY A. angry:rich B. native:affluent C. gauche:graceful D. scholarly:erudite Answer: Option C

2. Select the pair which has the same relationship. GRAIN:SALT A. shard:pottery B. shred:wood C. blades:grass D. chip:glass Answer: Option D

6. Select the pair which has the same relationship. WAN:COLOUR A. corpulent:weight B. insipid:flavour C. pallid:complexion D. enigmatic:puzzle Answer: Option B

10. Select the pair which has the same relationship. DISTANCE:MILE A. liquid:litre B. bushel:corn C. weight:scale D. fame:television Answer: Option A

3. Select the pair which has the same relationship. THRUST:SPEAR A. mangle:iron B. scabbard:sword C. bow:arrow D. fence:epee Answer: Option D

7. Select the pair which has the same relationship. PORK:PIG A. rooster:chicken B. mutton:sheep C. steer:beef D. lobster:crustacean Answer: Option B

11. Select the pair which has the same relationship. TEN:DECIMAL A. seven:septet B. four:quartet C. two:binary D. five:quince Answer: Option C

4. Select the pair which has the same relationship. PAIN:SEDATIVE A. comfort:stimulant B. grief:consolation C. trance:narcotic D. ache:extraction Answer: Option B

8. Select the pair which has the same relationship. AFTER:BEFORE A. first:second B. present:past C. contemporary:historic D. successor:predecessor Answer: Option D

12. Select the pair which has the same relationship. MUNDANE:SPIRITUAL A. common:ghostly B. worldly:unworldly C. routine:novel D. secular:clerical Answer: Option B

59 | P a g e

13. Select the pair which has the same relationship. ARMY:LOGISTICS A. business:strategy B. soldier:students C. war:logic D. team:individual Answer: Option A

16. Select the pair which has the same relationship. HOPE:ASPIRES A. love:elevates B. film:flam C. fib:lie D. fake:ordinary Answer: Option C

14. Select the pair which has the same relationship. GRAVITY:PULL A. iron:metal B. north pole:directions C. magnetism:attraction D. dust:desert Answer: Option C

17. Select the pair which has the same relationship. CORPOREAL:SPIRITUAL A. mesa:plateau B. moron:savant C. foreigner:immigrant D. pedagogue:teacher Answer: Option B

15. Select the pair which has the same relationship. FILTER:WATER A. curtail:activity B. expunge:book C. edit:text D. censor:play Answer: Option D

18. Select the pair which has the same relationship. SYMPHONY:COMPOSER A. Leonardo:music B. Fresco:painter C. colours:pallet D. art:appreciation Answer: Option B

60 | P a g e

19. Select the pair which has the same relationship. HOPE:ASPIRES A. love:elevates B. film:flam C. fib:lie D. fake:ordinary Answer: Option C

Selecting Words: Section 1 1. Fate smiles ...... those who untiringly grapple with stark realities of life. A. with B. over C. on D. round Answer: Option C

6. Success in this examination depends ...... hard work alone. A. at B. over C. for D. on Answer: Option D

11. Rohan and Rohit are twin brothers, but they do not look ...... A. unique B. different C. likely D. alike Answer: Option D

2. The miser gazed ...... at the pile of gold coins in front of him. A. avidly B. admiringly C. thoughtfully D. earnestly Answer: Option A

7. My uncle decided to take ...... and my sister to the market. A. I B. mine C. me D. myself Answer: Option C

12. To err is ...... to forgive divine. A. beastly B. human C. inhuman D. natural Answer: Option B

13. The ruling party will 8. If you smuggle goods have to put its own house into the country, they may ...... order. 3. Catching the earlier train be ...... by the customs A. in B. on will give us the ...... to do authority. C. to D. into some shopping. A. possessed Answer: Option A A. chance B. punished B. luck C. confiscated 14. ...... of old paintings is a C. possibility D. fined job for experts. D. occasion Answer: Option C A. Resurrection Answer: Option A B. Retrieval 9. Man does not live by ...... C. Restoration 4. I saw a ...... of cows in alone. D. Resumption the field. A. food B. bread Answer: Option C A. group B. herd C. meals D. diet C. swarm D. flock Answer: Option B 15. During Diwali the Answer: Option B shops are ...... of people. 10. Piyush behaves A. busy B. full 5. The grapes are now ...... strangely at times and, C. crowded D. bubbling enough to be picked. therefore, nobody gets ...... Answer: Option B A. ready with him. B. mature A. about B. through 16. The paths of glory lead C. ripe C. along D. up ...... to the grave. D. advanced Answer: Option C A. straight B. but Answer: Option C C. in D. directly Answer: Option B 61 | P a g e

17. The telephone ...... several times before I answered it. A. was ringing B. has rung C. had rung D. would ring Answer: Option C

22. Some regions of our country still remain ...... to the average man. A. inaccessible B. impossible C. impermeable D. impenetrable Answer: Option A

23. It ...... that Prashant will 18. He passed the not be selected for the post examination in the first A. feels B. looks class because he ...... C. believes D. seems A. was hard working for it Answer: Option D B. worked hardly for it C. had worked hard for it 24. In Bush, Saddam was D. was working hard for it up ...... more than his Answer: Option C match. A. for B. into 19. Jawaharlal spent his C. against D. to Answer: Option C childhood ...... Anand Bhawan. 25. I haven't seen you ...... a A. at B. in week. C. on D. across A. within B. since Answer: Option A C. for D. from Answer: Option C 20. If negotiations are to prove fruitful, there must 26. I listened, but I had no not only be sincerity on idea what he was ...... each side, but there must about. also be ...... in the sincerity A. saying of the other side. B. talking A. faith C. telling B. belief D. discussing C. substance Answer: Option B D. certainty Answer: Option A 27. The car in which the minister was traveling ...... 21. I hate sitting ...... him as with an accident. B. drove he always smells of garlic. A. hit C. crashed D. met A. besides B. along Answer: Option D C. at D. beside Answer: Option D 62 | P a g e

28. The non cooperative attitude of the members can only ...... the image of the society. A. spoil B. improve C. degrade D. defame Answer: Option A 29. Sonika is quite intelligent but rather ...... A. idealistic B. generous C. lazy D. optimistic Answer: Option C 30. In a little-publicised deal, Pepsi, Cola has ...... the entire soft drink market in Afghanistan. A. occupied B. conquered C. swallowed D. captured Answer: Option D 31. He ...... in wearing the old fashioned coat in spite of his wife's disapproval. A. insists B. persists C. desists D. resists Answer: Option B 32. She ...... a brief appearance at the end of party. A. put on B. put in C. put across D. put up Answer: Option B

33. Life is to death as pleasure is to ...... A. poverty B. suffering C. anguish D. pain Answer: Option D 34. This, partly, explains how the Mehta family has been able to ...... its lavish lifestyle in recent times, despite the fact that all its assets have been ...... A. keep, removed B. afford, attached C. develop, liquidated D. keep up, destroyed Answer: Option D

38. His father-in-law ...... him up in business. A. put B. made C. set D. built Answer: Option C

42. A crescendo of metallic thuds arose from the market, where the ironsmiths were ...... the pieces of metals. A. flattening B. striking C. hammering 39. ...... works of reference D. thrashing are valuable as Answer: Option C Encyclopedia, Brittanica. A. A few 43. I haven't eaten an apple B. Few ...... a long while. C. The few A. from B. since D. Fewer C. for D. until Answer: Option B Answer: Option C

40. This book is quite 35. The machine is difficult similar ...... to build ...... easy to A. with the "Treasure maintain. Island" A. but B. and B. of that film we saw at C. for D. if school Answer: Option A C. to the one I read last week 36. If you work beyond D. than a story told by our your capacity, you will teacher naturally feel ...... Answer: Option C A. drowsy B. tired 41. If our friends are not C. confident able to take us in their car, D. giddy we must make ...... Answer: Option B arrangements to go to the airport. 37. If you persists in telling A. alternative lies to me I shall sue you B. another ...... slander. C. alternate A. to B. on D. possible C. for D. with Answer: Option A Answer: Option C

63 | P a g e

44. The parliament invested the new organisation ...... judicial authority. A. by B. with C. from D. through Answer: Option B 45. Owing to the power cut in the area, factories are being forced to ...... men A. throw away B. send off C. put off D. lay off Answer: Option D 46. Man must ...... to stop pollution. A. act B. perform C. operate D. behave Answer: Option A

47. He was sent to the prison for his ...... A. sin B. vice C. crime D. guilt Answer: Option C

52. What is the ...... for an air letter? A. fare B. value C. postage D. stamp Answer: Option C

57. Find the ...... that accompany these cartoons A. topics B. titles C. captions D. headings Answer: Option C

48. The answer was written ...... blue ink. A. with B. by C. in D. on Answer: Option C

53. He became enamored ...... her grace when he first saw her dance. A. with B. of C. by D. in Answer: Option B

58. I don't really know how to ...... the problem. A. tackle B. cope C. draw D. erase Answer: Option A

49. Many ...... decisions were taken at the meeting. A. hectic B. historic C. historical D. histrionic Answer: Option B 50. Those who persist in the endeavor at long last triumph ...... the odds of life. A. over B. on C. upon D. about Answer: Option A 51. In the modern materialistic society, the only aim of people appears to be ...... money by fair means or foul. A. print B. produce C. acquire D. extort Answer: Option C

64 | P a g e

54. Rama was so badly injured that he needed ...... care in the hospital. A. extensive B. little C. deep D. intensive Answer: Option D 55. You cannot devise a method which ...... all possibility of errors. A. excludes B. includes C. avoids D. ignores Answer: Option A 56. Many areas of the city were ...... into darkness for several hours. A. spread B. plunged C. merged D. deep Answer: Option B

59. The English schemed to continue their rule in India by playing off one community ...... the other. A. before B. upon C. against D. with Answer: Option C 60. The passengers were afraid, but the captain ...... them that there was no danger. A. promised B. assured C. advised D. counselled Answer: Option B 61. The family gave father a gold watch on the ...... of his fiftieth birthday. A. time B. event C. occasion D. celebration Answer: Option C

62. The park ...... as far as the river. A. extends B. advances C. enlarges D. emerges Answer: Option A 63. Many of the advances of civilisation have been conceived by young people just on the ...... of adulthood A. boundary B. threshold C. peak D. horizon Answer: Option B

66. I was astounded at his ...... lack of knowledge about the Continent of Africa. A. abundant B. colossal C. huge D. great Answer: Option B 67. Ayesha always ...... the permission of her father before going for movies. A. seeking B. seeks C. sought D. seeker Answer: Option B

64. Arti pulled a long ...... when she was told that she could not go to Agra. A. mouth B. skirt C. face D. hand Answer: Option C

68. He felt no ...... as he plunged the knife into her back. A. qualms B. scruple C. conscience D. morals Answer: Option A

65. Brothers must live in harmony. They must never fall ...... A. off B. out C. apart D. away Answer: Option B

69. You haven't had your lunch yet, ...... you? A. are B. aren't C. have D. haven't Answer: Option C

65 | P a g e

70. The manner in which bombs exploded in five trains with in a short span of time suggests that it is a part of a ...... A. game B. conspiracy C. villainy D. sabotage Answer: Option B

Selecting Words: Section 2 1. Wheat ...... carbohydrates, vitamins, proteins, and dietary fibre in our daily diet. A. has B. gives C. yields D. provides Answer: Option D

11. The old Nature versus ...... debate regarding crime continues even today. A. Man B. Universe C. Culture D. Nurture 7. The river overflowed its Answer: Option A 2. The waiter hasn't ...... and flooded the area. brought the coffee ...... I've A. edges 12. The Sun ...... at six this been here an hour already. B. fronts morning. A. till B. up C. limits A. raised C. yet D. still D. banks B. rose Answer: Option C Answer: Option D C. arose D. aroused 3. In our zeal for progress 8. ...... The arrival of the Answer: Option B we should not ...... police, nobody went near executive with more the victim. 13. That rule is applicable power. A. Unless ...... every one. A. avoid B. give B. Although A. to B. for C. enhance D. arm C. Even C. about D. with Answer: Option D D. Till Answer: Option A Answer: Option D 4. You've only three 14. Her parents will never months to complete the 9. When I was a child, I give their ...... to so much course, Don't give ...... ......to school everyday an unsuitable match. now. instead going by cycle. A. acquiescence A. out B. away A. had walked B. consent C. up D. off B. walked C. agreement Answer: Option C C. have walked D. willingness D. have been walking Answer: Option B 5. The traveler slept under Answer: Option B the ...... shade of banyan 15. He tried to ...... himself tree. 10. Farida sings very well against a horde of ruffians. A. cold and ...... does salim. A. collect B. cool A. even B. too B. save C. cooling C. also D. so C. support D. dark Answer: Option D D. defend Answer: Option B Answer: Option D 66 | P a g e

6. She has an aversion ...... taking even onion and garlic. A. with B. at C. against D. to Answer: Option D

16. According to corporate circles data is pushing through the merger to create financially ...... company in the processed foods business, the group's thrust area for the 1990's A. acceptable B. powerful C. leading D. straight Answer: Option B 17. Indeed, all over the world, more and more people are ...... coffee. A. wanting B. drinking C. liking D. partaking Answer: Option B

20. People who ...... on horses usually lose in the end. A. gamble B. play C. risk D. place Answer: Option A 21. About twenty clerks were made ...... when the banks introduced computers. A. dispensable B. redundant C. expandable D. obsolete Answer: Option B

22. After the rain the weather ...... and the sun came out. A. cleared out B. cleared up 18. Like any other country C. cleared away India has its ...... share of D. cleared off superstitions. Answer: Option B A. proper B. abundant 23. The building comprises C. fair ...... sixty rooms. D. peculiar A. of Answer: Option C B. onto C. by 19. The new education D. no preposition needed policy provides a useful Answer: Option D ...... for the planners to remove illiteracy. 24. Namrata was found to A. breakup ...... the required B. breakthrough qualifications for the job. C. breakaway A. contain D. break-in B. disclose Answer: Option B C. posses D. acquire Answer: Option C 67 | P a g e

25. Government buildings are ...... on the Republic day. A. enlightened B. lightened C. illuminated D. glowed Answer: Option C 26. we had ...... wonderful time at the party last night. A. such a B. so C. such D. very Answer: Option A 27. An employment advertisement should ...... the number of vacancies. A. provide B. specify C. contain D. declare Answer: Option B 28. When varun left the cocktail party he was as ...... as a judge. A. sober B. drunk C. brave D. wise Answer: Option D 29. The criminal seems to have acted in ...... the three others. A. collusion B. coalition C. collision D. cohesion Answer: Option A

30. Soft minded individuals are ...... to embrace all kinds superstitions. A. disposed B. eager C. reluctant D. prone Answer: Option D 31. The stenographer is very efficient. He is ...... to his firm. A. a boon B. a credit C. a blessing D. an asset Answer: Option D 32. However, the group's long-term strategy is to ...... on core sector business connected with infrastructure and energy. A. breed B. develop C. concentrate D. depend Answer: Option C 33. The man came in a van to ...... the television set. A. mend B. reform C. correct D. alter Answer: Option A 34. Nobody can ...... me to do anything which I do not want to do. A. encourage B. request C. oppose D. compel Answer: Option D

68 | P a g e

35. A sanguine outlook is associated with the ...... A. rationalist B. socialist C. philanthropist D. optimist Answer: Option D 36. His conduct is bad, and his honesty is not ...... suspicion. A. above B. beyond C. under D. in Answer: Option A

41. Health is too important to be ...... A. discarded B. despised C. detested D. neglected Answer: Option D

42. In hot weather I like lying in the ...... of a tree. A. shade B. shelter C. protection D. shadow 37. It is 14 years since i ...... Answer: Option A him. 43. It was indeed A. have seen unreasonable ...... him to B. had seen leave this job and start C. saw D. see business. Answer: Option C A. in B. with 38. It being an ...... issue, it C. upon is not correct to introduce D. of questions of morality in to Answer: Option D the debate. A. moral B. immoral 44. One major ...... between C. amoral D. irrelevant the Election Commission Answer: Option D and the Union 39. Natwarlal ...... them all Government related to the powers of the former in for a ride by producing respect of the deployment false documents. of central police forces at A. took B. cheated places where are elections C. kept D. let is held. Answer: Option A A. irritant B. conflict 40. True brevity ...... in saying only what needs to C. pain be said. D. culprit A. consists B. depicts Answer: Option B C. portrays D. resides Answer: Option A

45. We shall not to be able to use your ability in court unless we can find someone to ...... to statements. A. corroborate B. avouch C. verify D. approve Answer: Option A

50. He knew everything better than anybody else, and it was an affront to his ...... vanity that you should disagree with him. A. overstrung B. overweening C. overwhelming D. overwrought Answer: Option A

55. Ambition is one of those ......which are never satisfied. A. ideas B. fancies C. passions D. feeds Answer: Option C

56. He is a person of sound character and ...... disposition. A. beneficent 46. He is too dull ...... this 51. It's very kind of you to B. morous problem. ...... to speak at the C. amiable A. solving meeting. D. amicable B. to solving A. accept B. agree Answer: Option C C. to solve C. comply D. concur D. solves Answer: Option B 57. If I take a state Answer: Option C roadways bus, I'll get late, 52. Pakistan lost a ...... ...... ? 47. He is the person who is wicket just when they A. isn't it B. won't I ...... to blame. seemed to be doing so C. will I D. is it A. mostly well, and that led to their Answer: Option B B. sure eventual defeat. C. most A. critical 58. In high school many of D. bound B. crucial us never realised the Answer: Option C C. sensitive importance that grammar D. providential would ...... in later life. 48. We had to pay more Answer: Option B A. figure B. portray taxi fare because the driver C. play D. exercise brought us by a ...... route. 53. Sita's heart ...... at the Answer: Option C A. circular sight of the beautiful B. circumscribed diamond necklace. 59. In a changing and ...... C. longest A. leapt B. stopped unstructured business D. circuitous C. slowed D. ran environment, creativity and Answer: Option D Answer: Option A innovation are being ...... demanded of executives. 49. He was accused ...... 54. He lives near a lovely A. highly, extremely stealing his aunt's necklace. ...... of countryside. B. progressively, A. for B. with A. length B. piece increasingly C. of D. on C. section D. stretch C. increasingly, moderately Answer: Option C Answer: Option D D. excessively, rapidly Answer: Option B 69 | P a g e

60. On account of the dearth of grass on the arid plains the cattle became ...... A. flippant B. jubilant C. agitated D. emaciated Answer: Option D

64. The American ...... presented his credentials to the President of India. A. adviser B. ambassador C. delegate D. representative Answer: Option B

69. this brand of TV is quite inferior ...... that one. A. than B. to C. with D. over Answer: Option B

70. It is not what you say 65. Once he has signed the that ...... but what you do 61. A woman came in with agreement, he won't be A. matches a baby who, she said, ...... a able to ...... B. implies safety pin. A. back up C. matters A. was just swallowing B. back in D. moves B. swallowed C. back at Answer: Option C C. had just swallowed D. back out D. just swallowed Answer: Option D Answer: Option C 66. It was difficult to 62. The Hubble Space remove my feet as it had Telescope will search for got stuck ...... in the mud. planets around the stars, a A. fairly B. greatly key to the ...... C. widely D. firmly extraterrestrial life, and Answer: Option D examine interstellar dust and gases out of which 67. He is a very careful stars are born. person, he never takes side A. perception but remains ...... B. discovery A. impartial C. enquiry B. unbiased D. quest C. neutral Answer: Option D D. prejudiced Answer: Option A 63. She ...... Rs 80 out of the bank every Friday. 68. West Bengal ...... A. obtains plentiful rainfall and is B. draws consequently a very green C. pulls part of the country. D. extracts A. misses B. receives Answer: Option B C. expects D. regrets Answer: Option B 70 | P a g e

Selecting Words: Section 3 1. 'Please' and 'Thank you' are the little courtesies by which we keep the ...... of life oiled and running smoothly. A. path B. machine C. garden D. river Answer: Option B 2. I put ...... the light and slept. A. up B. down C. in D. out Answer: Option D 3. ...... all intents and purposes, the manager is the master of the firm. A. in B. upon C. with D. to Answer: Option D 4. The pilot had been warned about the storm, before he ...... A. took away B. took up C. took over D. took off Answer: Option D 5. Hardly had he arrived ...... it started raining. A. before B. when C. than D. after Answer: Option B 6. I ...... a car to be absolutely necessary these days. A. consider B. regard C. think D. agree Answer: Option A 71 | P a g e

7. Besides other provisions, that shopkeeper deals ...... cosmetics. A. with B. in C. at D. for Answer: Option B

12. I shall call ...... you tomorrow. A. at B. on C. with D. by Answer: Option C

8. Leave a two inch ...... on each page for the teacher's remarks. A. border B. margin C. blank D. gap Answer: Option B

13. She ...... from the crowd because of her height and flaming red hair. A. stood out B. stood off C. stood up D. stood by Answer: Option A

9. The team was well trained and strong, but somehow their ...... was low. A. feeling B. moral C. consciousness D. morale Answer: Option D 10. The lovers were meeting each other secretly, but their ...... affair was soon known to everyone. A. clandestine B. covert C. unknown D. candid Answer: Option A

14. In a large cities people are cut ...... from nature. A. away B. off C. out D. down Answer: Option A 15. The more your action and thought are allied and ...... the happier you grow. A. diverget B. unravelled C. integrated D. invincible Answer: Option C

16. We had a ...... of warm 11. The king ...... the rebel. weather in February. A. time A. excused B. spell B. forgave C. pardoned C. length D. none of these D. phase Answer: Option C Answer: Option B

17. Physically we are now all neighbors, but psychologically, we are ...... to each other. A. primitives B. complimentary C. strangers D. cowards Answer: Option C

22. Walking at 3'o clock, I heard the ...... of thunder. A. crackle B. rumble C. ripple D. clank Answer: Option B

23. A stone that goes on rolling ...... no moss. A. collects B. gets 18. When their C. gathers examinations are over, the D. accumulates children gleefully ...... the Answer: Option C books they had been reading. 24. The earth is at present A. shelve in great danger of B. sidetrack becoming uninhabitable C. overthrew because of ...... D. abandon environmental pollution Answer: Option D which is going on at an incredible rapid pace. 19. Chintu is ...... small to A. gigantic start playing cricket now. B. inhuman A. very C. stupendous B. much D. colossal C. too Answer: Option D D. more Answer: Option C 25. Moreover, a fact finding mission ...... by 20. The thief ...... all the BSN to India in January money. this year strongly A. made up recommended that the B. made off with French group should go it C. mode do with alone, and not hand over D. made good ...... to an Indian Partner. Answer: Option B A. organised, papers B. constituted, authority 21. The luggage was ...... C. sponsored, power heavy for him to lift D. dispatched, control A. much B. as Answer: Option D C. so D. too Answer: Option D 72 | P a g e

26. There is a keen ...... in each trade. A. contest B. comparison C. competition D. cooperation Answer: Option C 27. Now, the management graduate can expect to have a prosperous life on a ...... income without having to depend on finding a place in family busiess having to tend the paternal estates. A. professional B. regular C. meaningful D. dependable Answer: Option B 28. The boy fell ...... bicycle. A. of B. off C. from D. under Answer: Option B 29. The French ...... reputed to have a very good sense of humour. A. is B. was C. are D. will be Answer: Option C 30. Johny, where are you? ...... up this tree. A. There I am B. There am I C. Here am I D. Here I am Answer: Option D

31. That the poor in our country, are happy is A. a dream B. a vision C. an ideal D. an illusion Answer: Option D 32. The robbers were arrested and ...... prison yesterday. A. brought into B. brought to C. taken into D. taken to Answer: Option D 33. The cinema ...... a welcome escape from cramped and dull city life and the Indians are avid movie-goers. A. depicts B. highlights C. follows D. offers Answer: Option D

37. The communalist represents the ...... of everything noble that we have inherited from our culture and history. A. antithesis B. antidote C. immorality D. antagonism Answer: Option A

41. Colgate has also set an ambitious aim of ......on 8% value shared of tooth paste market by then end of first years. A. cornering B. soliciting C. disturbing D. keeping Answer: Option A

38. He is so ...... to light that he never leaves the house without sunglasses. A. insensitive B. sensitive C. afraid D. immune Answer: Option B

42. ...... a failure of some traffic lights, traffic is moving very slowly. A. Owing B. Due to C. Because D. Since Answer: Option B

39. The ...... of the Minister's statement cannot 34. I shall take revenge ...... be verified by people who you. have no access to official A. from B. with records. C. on D. at A. veracity Answer: Option C B. verbosity C. ambiguity 35. This book is a useful D. validity ...... to our library. Answer: Option A A. discovery

43. The battalion operating from the mountain was able to ...... three enemy divisons. A. tie up B. tie down C. tie on D. tie with Answer: Option B

44. I purposely ...... meet you during my last visit to Kashmir. B. asset 40. But the introductory A. didn't B. won't C. addition fee does not stop after the C. hadn't D. wouldn't D. arrival initial handshake, brokers Answer: Option A Answer: Option C have to ...... it up after each 36. The boy was cured ...... transaction. 45. I have read one novel typhoid. A. bring by Premchand. I want to A. from B. of B. cough read ...... novel by him. C. for D. through C. boost A. other B. another Answer: Option B D. give C. all D. few Answer: Option A Answer: Option B 73 | P a g e

46. A man remains narrow minded, self compliance and ignorant unless he visits other people and ...... from them. A. earns B. borrows C. learns D. hears Answer: Option C

50. Affix a revenue stamp and put your signature ...... it. A. on B. upo C. above D. over Answer: Option A

54. God is ...... A. graceful B. gracious C. grateful D. greatful Answer: Option B

55. He is so ...... that he immediately believe my 51. My first lesson ...... story of ghosts. forgiveness came from my A. innocent 47. My father ...... down for mother. B. credulous a nap. A. upon C. vociferous A. lays B. about D. credible B. laid C. in Answer: Option B C. lain D. on D. lie Answer: Option D Answer: Option A 52. At one point, it looked 48. I think they allow their as if an area of agreement children too much ...... would ...... specially over A. liberality the issue of productivity B. latitude linked wages. C. lassitude A. develop D. levity B. come out Answer: Option B C. emerge D. grow 49. Once you suspect a Answer: Option C person of double dealing, you ought to keep him at 53. Do you know ......? arm's ...... A. where she comes from A. distance B. where does she come B. length from C. aim C. where from she comes D. width D. from where does she Answer: Option B come Answer: Option A

74 | P a g e

Sentence Completion 5. Although he never learn to read, his exceptional memory and enquiring mind eventually made him a very _______ man. A. dedicated B. erudite C. pragmatic D. benevolent Answer: Option B

9. Jill was ________ by her employees because she often _______ them for not working hard enough. A. deified, goaded B. loathed, berated C. disregarded, eulogized D. cherished, decided Answer: Option B

6. More insurers are 2. There are _______ limiting the sale of views on the issue of giving property insurance in coastal areas and other bonus to the employees. regions _______ natural A. independent disasters. B. divergent A. safe from C. modest B. according to D. adverse C. despite Answer: Option B D. prone to Answer: Option D 3. Man who has committed such an _______ crime 7. Roman Regions must get the most severe _______the punishment. Mountain_______of A. injurious Masada for three years before they were able to B. unchritable seize it. C. unworhty A. dissembled, bastion D. abominable B. assailed, symbol Answer: Option D C. besieged, citadel 4. He has _______ people D. honed, stronghold Answer: Option C visiting him at his house because he fears it will 8. Unlike his calmer, more cause discomfort to easygoing colleagues ,the neighbours senator was _________ A. curtailed ,ready to quarrel at the B. requested slightest provocation. A. whimsical C. stopped B. irascible D. warned C. gregarious Answer: Option C D. ineffectual Answer: Option B

10. Reconstructing the skeletons of extinct species like dinosaurs is ________ process that requires much patience and effort by paleontologists. A. a nascent B. an aberrant C. a worthless D. an exacting Answer: Option D

1. Rajeev failed in the examination because none of his answers were _______ to the questions asked. A. allusive B. revealing C. pertinent D. referential Answer: Option C

75 | P a g e

11. This is a ...... on his character. A. blot B. blur C. slur D. spot E. mark Answer: Option C 12. This is a good ...... for a picnic. A. plot B. spot C. scene D. landscape Answer: Option B

13. The ....... of private limited companies is in the hands of its directors. A. managers B. adminstrators C. management D. deparment E. society Answer: Option C 14. Ram the prince of Ayodhya ....... his siblings. A. adorned B. adored C. vitiated D. endangered E. abhored Answer: Option B

18. A wise person is one who learns from his .......... A. errors B. mistakes C. falsities D. lies E. bad manners Answer: Option B 19. Many companies ask their employees to sign a ......... of secrecy. A. agreement B. bond C. bondage D. espionage E. treaty Answer: Option B

23. I wonder whether I ........ ever see him again. A. shall B. will C. should D. would Answer: Option D 24. No sooner ......... than the police handcuffed him. A. he came B. did he came C. did he come D. had he come Answer: Option C

25. ''What exactly does he want ?" "He would like to see you ...... a good job on the project." 15. The Christmas tree 20. Slaves were freed from A. got B. get was.......... with stars and .......... only after they died. C. to get D. will get other decorative items. A. ablution Answer: Option A A. adorned B. pilferage B. endowed C. agreement 26. "She was sorry she C. encased D. bondage didn't attend her friend's D. enticed E. abolition wedding." E. encompassed Answer: Option D "Yes, she ........." Answer: Option A A. regretted she can't have 21. Before the ........ of the gone 16. The ......... of the state is Europeans in India, India B. was regretting she does efficient. was a free country. not go A. administration A. entry C. will regret she didn't go B. democracy B. amalgamation D. was regretting that she C. policy C. emigration she couldn't go D. autocracy D. advent Answer: Option D E. plutscracy E. immigration Answer: Option A Answer: Option D 27. "Sujata is late in the office." 17. There is no place for 22. Some of the luggage "She rarely comes in time .......... in an accountant's ......... not yet arrived. ..... ?" job. A. has A. don't she A. assets B. have B. does she B. asserts C. is C. doesn't she C. errors D. are D. won't she D. duffers Answer: Option A Answer: Option B Answer: Option C 76 | P a g e

28. He behaves as if he ....... a king. A. was B. were C. has D. had Answer: Option B 29. The dog has been ....... by the car. A. run out B. run over C. run into D. run for Answer: Option B 30. ..... you apologize I shall punish you. A. until B. unless C. till D. None of these Answer: Option B

34. I suffer from no ....... about my capabilities. A. illusion B. doubts C. hallucinations D. imaginations Answer: Option A 35. The government, in a bid to make Bihar, a preferred investment destination, has _________ major schemes to create infrastructure at different stages. A. launched B. opposed C. opened D. pushed Answer: Option A

36. The student's aren't 31. Lord Buddha ......... his prepared _______ the kingship and become a examination hermit. A. to give A. Abolished B. to listen B. Abated C. to work C. Abdicated D. to take D. Abandoned Answer: Option D Answer: Option C 37. We should _______ 32. This is a ............... on opportunities as they arise. his character. A. seize B. crease A. Blot B. Blur C. size D. sneeze C. Slur D. Spot Answer: Option A E. Mark Answer: Option C 38. There is no culture in the world ........... absolutely 33. We didn't ....... the pure. programme to be such a A. That which is huge success. B. That which A. Accept B. Except C. That is C. Expect D. Access D. That what is Answer: Option C Answer: Option C 77 | P a g e

39. He took up the letter from the table and .......... it to his friend. A. was reading B. has been reading C. has read D. read Answer: Option D 40. He led me ......... the green laws to the palatial building. A. upon B. across C. along D. about Answer: Option C 41. The ship with all its crew ...... lost after the terrible storm. A. were B. has C. have D. was Answer: Option D 42. We ........ start now to work for our examination. A. had better B. better C. has better D. are better Answer: Option B 43. The lease of our premises has ......... and we have to vacate it. A. run out B. run off C. run over D. run down Answer: Option A 44. His boss .......... an explanation of his conduct with his colleagues. A. called up B. called upon C. called for D. called off Answer: Option C

45. The artist had to ______ a lot before _______ recognized for his talent. A. strugle, being B. toil, he C. practise, performing D. effort, he was E. strive , the Answer: Option A

48. They ______ a sound infrastructure in order to _________ with the new challenges. A. want, handle B. got, mets C. have , give D. need, cope E. had, reach Answer: Option D

51. The pen ______ mightier than the sword. A. shall be B. has C. will be D. is Answer: Option D

52. Rama took his _______ for the wrong done to him. 46. Most of the committee 49. As soon as she ______ A. avenge members _____ in favour the branch office, she was B. vengeance of the ______ of a new asked to _____ to the C. revenge dam in that village. head-office immediately. D. ravage A. was, building A. came, go Answer: Option C B. are, proposing B. went, attend C. were, construction C. reached, report 53. On his sudden demise, D. not, assembly D. found, look my emotions were so, E. still, breaking E. saw, remind complicated that it was Answer: Option C Answer: Option C ______ how I felt. A. unreasonable 47. She _______ to learn 50. Please don't _____ B. impossible basic German as she will when I am talking. C. intolerable be ______ for Germany A. disrupt D. unimaginable next month. B. interface Answer: Option C A. likes, reaching C. interrupt B. wants, leaving D. disturb C. intends, visiting Answer: Option C D. tries, touring E. knows, going Answer: Option B

78 | P a g e

Completing Statements 1. Despite his best efforts to conceal his anger ...... A. we could detect that he was very happy B. he failed to give us an impression of his agony C. he succeeded in camouflaging his emotions D. he could succeed in doing it easily E. people came to know that he was annoyed

4. She never visits any zoo because she is strong opponent of the idea of ...... A. setting the animals free into forest B. feeding the animals while others are watching C. watching the animals in their natural abode D. going out of the house on a holiday E. holding the animals in captivity for our joy Answer: Option E

Answer: Option E 5. I felt somewhat more relaxed ...... 2. Even if it rains I shall come means ...... A. but tense as compared to earlier A. if I come it will not rain B. and tense as compared to earlier B. if it rains I shall not come C. as there was already no tension at all C. I will certainly come whether it rains D. and tension-free as compared to or not earlier D. whenever there is rain I shall come E. because the worry had already E. I am less likely to come if it rains captured by mind Answer: Option C 3. His appearance is unsmiling but ...... A. his heart is full of compassion for others B. he looks very serious on most occasions C. people are afraid of him D. he is uncompromising on matters of task performance E. he is full of jealousy towards his colleagues Answer: Option A 79 | P a g e

Answer: Option D 6. It is not easy to remain tranquil when those around you ...... A. behave in a socially acceptable manner B. exhibit pleasant mannerism C. are losing their heads D. agree to whatever you say E. exhibit generous and magnanimous gestures Answer: Option C

7. "The food in this hotel is no match to what were forced at late hours in Hotel Kohinoor " means ...... A. The food in this hotel is quite good compared to what we ate at kohinoor B. Hotel kohinoor served us good quality food than what we get here C. Both hotels have maintained good quality of food D. Both hotels serve poor quality of food E. it is better to eat food than remain hungry Answer: Option B

10. "It is an uphill task but you will have to do it" means ...... A. The work is above the hill and you will have to do it B. It is a very easy task but you must do it C. It is very difficult task but you have to do it D. This work is not reserved for you but you will have to do it E. It is almost impossible for others but you can do it Answer: Option C

8. Although initial investigations pointed towards him ...... A. the preceding events corroborated his involvement in the crime B. the additional information confirmed his guilt C. the subsequent events established that he was guilt D. the subsequent events proved that he was innocent E. he gave an open confession of his crime

11. " You are thinking very highly about Ravi but he is not so" means ...... A. Ravi is as good as you think about him B. You have a good opinion about Ravi but he is not as good as you think C. Your view about Ravi is philosophical, keep it up D. Ravi is much better, than what you think of him E. You have a good opinion about Ravi but he does not have a good opinion about you

Answer: Option D

Answer: Option B

9. The weather outside was extremely pleasant and hence we decided to ...... A. utilise our time in watching the television B. refrain from going out for a morning walk C. enjoy a morning ride in the open D. employ this rare opportunity for writing letters E. remain seated in our rooms in the bungalow

12. "Anand stuck up a friendship with Mahesh in just 2 days" means ...... A. Anand friendship with Mahesh came to an end recently B. Anand found out the other friends of Mahesh C. Anand fixed a deal with Mahesh in 2 days D. Anand E. Anand became a friend of Mahesh in less than 2 days

Answer: Option C

Answer: Option E

80 | P a g e

13. Although, he is reputed for making very candid statements ...... A. his today speech was not fairly audible B. his promises had always been realistic C. his speech was very interesting D. people follow whatever he instructs to them E. his today Answer: Option E

16. He has no money now ...... A. although he was very poor once B. as hehas given up all his wealth C. because he was very rich once D. because he has received huge donation E. because he was very greedy about wealth Answer: Option B

17. He is so lazy that he ...... 14. The manager would like you to help A. cannot depend on others for getting his work done Dhiraj, means ...... A. the manager would like you if you help B. cannot delay the schedule of completing the work Dhiraj B. the manager desires you to help Dhiraj C. can seldom complete his work on time D. dislike to postpone the work that he C. the manager likes you because you undertakes to do help Dhiraj E. always help others to complete their D. Dhiraj expects the manager to tell you work to help him E. it will be a help to the manager if you Answer: Option C like Dhiraj 18. Dinesh is as stupid as he is lazy means ...... A. Dinesh is stupid because he is lazy 15. Owing to the acute power shortage, B. Dinesh is lazy because he is stupid the people of our locality have decided to C. Dinesh is either stupid or lazy D. Dinesh is hardly stupid but he is lazy ...... A. dispense with other non-conventional E. Dinesh is equally stupid and lazy energy sources Answer: Option E B. resort to abundant use of electricity for illumination 19. Practically, very little work could be C. off-switch the electrical appliance completed in the last week as it was ...... while not in use D. explore other avenues for utilising the A. full of working days B. a very hectic week excess power E. resort to use of electricity only when it C. full of holidays D. a very busy week is inevitable E. loaded with work Answer: Option B

Answer: Option E 81 | P a g e

Answer: Option C

20. Because he believes in democratic principles, he always ...... A. decides all the matters himself B. listen to others views and enforces his own C. shown respect to others opinions if they match his own D. reconciles with the majority views and gives us his own E. imposes his own views on others

23. He always stammers in public meetings, but his today's speech ...... A. was fairly audible to everyone present in the hall B. was not received satisfactorily C. could not be understood properly D. was not liked by the audience E. was free from that defect Answer: Option E

Answer: Option D

24. Even though it is very large house, ...... 21. With great efforts his son succeeded A. there is a lot of space available in it for in convincing him not to donate his children entire wealth to an orphanage ...... B. there is hardly any space available for A. and lead the life of a wealthy merchant children B. but to a home for the forsaken C. there is no dearth of space for children children D. the servants take a long time to clean C. and make an orphan of himself it D. as the orphanage needed a lot of E. the municipal taxes on it are very donations happy E. as the orphanage had been set up by him Answer: Option B Answer: Option C

25. It was an extremely pleasant surprise for the hutment-dweller when the 22. The employer appeared to be in such Government officials told him that ...... an affable mood that Rohit ...... A. he had to vacate hutment which he A. decided to ask for a raise in his salary had been unauthorisedly occupying B. was scared to talk to him about his B. he had been gifted with a furnished leave apartment in a multi-storeyed building C. felt very guilty for his inadvertent slip C. he would be arrested for wrongfully D. promised him that he would not encroaching on the pavement outside his commit mistake again dwelling E. was pained to press his demand for a D. they would not accede to his request new flat. E. they had received the orders from the court to take possession of all his Answer: Option A belongings Answer: Option B

82 | P a g e

26. In order to help the company attain its goal of enhancing profit, all the employees ...... A. urged the management to grant paid leave B. appealed the management to implement new welfare schemes C. voluntarily offered to work overtime with lucrative compensation D. voluntarily offered to render additional services in lieu of nothing E. decided to enhance production at the cost of quality of the product Answer: Option D

29. Mahesh need not have purchased the bag, means ...... A. it was not necessary for Mahesh to purchase the bag but he has purchased it B. it was necessary for Mahesh to purchase the bag and he has not purchased it C. it was not necessary for Mahesh to purchase the bag and he has not purchased it D. it was necessary for Mahesh to purchase the bag but he has not purchased it E. Mahesh already has a bag but still he purchased another one Answer: Option A

27. "Whatever Dev uttered was without rhyme or reason" means ...... A. Dev could not recite any poem or speech B. Dev said something which has no meaning, it was totally baseless C. Dev was talking something which was beyond our experience D. Dev spoke flatly without any emotion or reason E. Dev did not refer to any poem to support his statements

30. In order to raise company's profit, the employees ...... A. demanded two additional increments B. decided to go on paid holidays C. requested the management to implement new welfare schemes D. offered to work overtime without any compensation E. decided to raise the cost of raw material

Answer: Option B

Answer: Option D

28. He tames animals because he ...... A. is fond of them B. hates them C. is afraid of them D. want to set them free E. seldom loves them

31. The officer who had neglected to files his income tax returns had to ...... A. return the files B. pay a fine C. be rewarded D. play mischief E. give warning

Answer: Option A

Answer: Option B 83 | P a g e

32. Unless you work harder you will fail, means ...... A. if you fail you will work harder B. you must at least plan well than you will not fail C. hardly you will fail if you do not desire so D. if you do not put more efforts, then you will fail E. if you only work and work alone, you will fail Answer: Option D 33. His behaviour is so unpredictable that he ...... A. never depends upon others for getting his work done B. is seldom trusted by others C. always finds it difficult to keep his word D. always insists on getting the work completed on time E. seldom trusts others as far as the work schedule is concerned Answer: Option B

84 | P a g e

34. The officer who had neglected to files his income tax returns had to ...... A. return the files B. pay a fine C. be rewarded D. play mischief E. give warning Answer: Option B

Common Error Detection

Rules of Subject Verb Agreement Subject -Verb Agreement Subject - Verb Agreement relates to number agreement (singular or plural) between the subject and the verb that follows it. E.g. The girl plays all day. (Singular subject) The girls play all day. (Plural subject) •There are two important exceptions to subject-verb agreement. Firstly, agreement only applies when the verb is in the present tense. In the past tense, there is no overt agreement between the subject and the verb. E.g. The girl played all day. The girls played all day. •And secondly, agreement applies only to third person subjects. There is no distinction between a first person singular subject and a first person plural subject. E.g. I play all day. We play all day. The Rules Singular subjects need singular verbs and Plural subjects need ; plural subjects need plural verbs. 1. When subject of a sentence is composed of two or more nouns or pronouns connected by and, we use a plural verb. He and his friends are at the fair. This rule is not follow when two nouns refer to the same object. The wicket keeper and Captain was given the man of the match award. 2. When two or more singular nouns or pronouns are connected by or or nor, we use a singular verb. E.g. The paper or the pencil is in the desk. 3. When a compound subject contains both a singular and a plural noun or pronoun joined by or or nor, the verb should agree with the part of the subject that is nearer to the verb. E.g. Shivin or his friends run every day. His friends or Shivin runs every day. 4. Doesn’t is contraction of does not and should be used only with a singular subject. Don’t is a contraction of do not and should be used only with a plural subject. The exception to this rule appears in the case of the first person and second person pronouns I and you. With these pronouns, the contraction Don’t should be used. E.g. She doesn’t like it. They don’t like it.

85 | P a g e

5. A phrase between the subject and the verb should not be misleading, the verb agrees with the subject, not with a noun or pronoun in the phrase. E.g. The captain, as well as his players, is anxious. The book, in parts, is boring. The building, with all its contents, is insured. 6. Each, each one, either, neither, everyone, everybody, anybody, nobody, somebody, some one and no one are singular and require a singular verb. E.g. Each of these books is good. Everybody knows him. Either is correct. 7. Nouns such as civics, mathematics, rupees and news require singular verbs. E.g. The news is on at eight. • When talking about an amount of money, it requires a singular verb, but when referring to the rupees themselves, a plural verb is required. E.g. Five hundred rupees is a lot of money. Rupees are often used instead or Rupiahs in Nepal. 8. Nouns such as scissors, trousers, shears and shorts require plural verbs. E.g. These shears are dull. These shorts are made of cotton. 9. In sentences beginning with there is, there are, the subject follows the verb. Since there is not the subject, the verb agrees with what follows. E.g. There are many questions. There is a question. 10. Collective nouns are words that imply more than one person but are considered singular and take a singular verb, such as : group, team, committee, class and family. E.g. The family has a long history of writers. The committee has prepared the agenda. 11. With, together with, including, accompanied by, in addition to, or as well do not change the number of the subject. If the subject is singular, the verb is too. The Chairman, including his team, is travelling to China. The boys, together with their friends, are going for a picnic. 12. The relative pronouns (who, whom, which, and that) are either singular or plural, depending on the words they refer to. The sales manager is good researcher who spends a great amount of time surfing the Web for information. Subject : The sales manager Verbs: is, spends 86 | P a g e

13. A few nouns can be either plural or singular, depending on whether they mean a group or Separate individuals. These words are rarely used as plurals in modern writing. E.g. 1. The jury has decided unanimously. Subject: jury Verb: has decided 2. The jury are having an argument. Subject: jury Verb: are having 14. Do not be misled by a phrase that comes between the subject and the verb. The agrees with the subject, not with a noun or pronoun in the phrase. E.g. One of the boxes is open. The people who listen to music are few. 15. Two singular subjects connected by either/or or neither/nor require a singular verb as in Rule 1. E.g. Neither John nor Abraham is available. Either Bipasha or Neelam is helping today with stage decoration. 16. When I is one of the two subjects connected by either/or or neither/nor, put it second and follow it with the singular verb am. E.g. Neither she nor I am going to the festival. 17. With words that indicate portions-percent, fraction, part, majority, some, all, none, remainder, and so forth &ndash look at the noun in your ‘of phrase (object of the preposition) to determine whether to use a singular of plural verb. If the object of the preposition is singular, use a singular verb. If the object of the preposition is plural, use a plural verb. E.g. 1. Fifty percent of the pie has disappeared. (Pie is the object of the preposition of.) 2. Fifty percent of the pies have disappeared. 3. One-third of the city is unemployed. 4. One-third of the people are unemployed. NOTE: Hyphenate all spelled-out fractions. 5. All of the pie is gone. 6. All of the pies are gone. 7. Some of the pie is missing. 8. Some of the pies are missing. 18. Use a singular verb with sums of money or periods of time. E.g. Ten dollars is a high price to pay. Five years is the maximum sentence for that offense.

87 | P a g e

Identifying the subject Word groups often come between the subject and the verb. These word groups may contain a noun that at first appears to be the subject. Identify the word group between the subject and the verb in order to isolate the noun. E.g. The girl plays all day. (Singular subject) The slaughter of pandas for their pelts have caused panda population to decline drastically. (Incorrect) The slaughter of pandas for their pelts has caused the panda population to decline drastically. (Correct) Thus, the subject is slaughter and not pandas or pelts. • Phrases beginning with the prepositions as well as, in addition to, accompanied by, and along with, do not make a singular subject plural. E.g. The Chief Minister, as well as his principal secretary, was shot. If a customer buys a burger, he or she has to pay service tax. (correct) If a customer buys a burger, they have to buy a parking ticket. (Incorrect) Questions: Directions: Find out the error, if any in questions given in each practice set. If there is no error, the answer is (e), i.e No error (ignore the errors of punctuation, if any.) 1. A few kilometers beyond (a) /that villa was seen two aliens (b )/along with (c)/ a few Americans and Russian scientists (d)/ No error (e). 2. It is well known fact that (a)/the upper middle class have no soft (b)/ corner for the poor and the oppressed who are the (c)/real victims of this social system (d)/ No error (e). 3. Sheetal as well as (a)/ some of her friends (b)/have fallen in love with a hunky fellow, who (c)/ is the eldest son of an I.A.S. officer (d)/ No error (e). 4. More than one successful candidate of this institute (a)/have taken the interview for one (b)/of the most popular dailies (c)/being published from Los Angels (d)/ No error (e). 5. More than one book on life style (a)/ and motivation by this writer (b)/have been published so far and (c)/all are selling like a hot cake (d)/ No error (e). 6. Many a man along with (a)/a few friends of mine are going (b)/to DevGhar to offer flowers and (c)/milk to Lord Shiva (d)/ No error (e). 7. Many an employee of this company (a)/ are supposed to be transferred (b)/ to some other company (c)/ of the same category because of recession (d)/ No error (e). 8. The political leader as well as (a)/ his retinues are going on a hunger (b)/ Strike to protest this (c)/ unnecessary price hike (d)/ No error (e). 9. Although this book as well as those (a)/ books of yours are similar (b)/ in many ways, mine has (c)/ an edge over many a book of yours (d)/ No error (e). 10. The prime minister along with his (a)/ attendants are coming tomorrow (b)/ to address the sensational (c)/ and controversial issues of terrorism (d)/ No error (e). Answers 1. b 2. b 88 | P a g e

3. c

4. b

5. c

6. b

7. a

8. b

9. b

10. b

Understanding of Conjunction – Tips and Tricks Conjunction A conjunction is a word which merely joins together sentences or clauses, and sometimes words. 1. That is a beautiful place and I have been there. 2. That is the beautiful place where I have been to. Classes of Conjunctions Conjunctions are divided into two classes: Co-ordinate Conjunction : It joins together clauses of equal ranks. For eg. He is slow, but he is sure. Sub-ordinate Conjunction : It joins together clauses in which one is dependent on the other. E.g. Since you wish to excel in life, you must learn English. Some Important Rules Rule: Scarcely should be followed by when, not by than. Incorrect: Scarcely had he arrived than he had to leave again. Correct: Scarcely had he arrived when he had to leave again. Rule: No sooner is followed by than, not by but. Incorrect: No sooner had we reached the station but the train left. Correct: No sooner had we reached the station than the train left. Note : "No sooner A ... than B" is very similar to "Scarcely A... when B" and has almost the same meaning. Rule : Do not use seldom or ever in place of seldom or never. Incorrect: The national network seldom or ever telecasts good programmes. Correct: The national network seldom or never telecasts good programmes. Rule : Correlative conjunctions should be followed by the same part of speech. Incorrect: He neither agreed to my proposal nor to his. Correct: He agreed neither to my proposal nor to his. Rule : Neither is followed by nor, not by or. Incorrect: The phone neither went dead or worked properly. Correct: The phone neither went dead nor worked properly. Rule : The conjunction that is not used before i) A sentence in direct narration. ii) An interrogative adverb or interrogative pronoun in the indirect narration. Incorrect: He said that, "it is none of my business". Correct: He said, "It is none of my business." Incorrect: He asked me that which was the way to the station. 89 | P a g e

Correct: He asked me which was the way to the station. Rule : Although goes with yet or a comma (,) in the other clause. Incorrect: Although Manohar is hardworking, but he does not get a job. Correct: Although Manohar is hardworking, yet he does not get a job. Note : Though/Although is used with contradictory sentences (sentences expressing opposite views). Incorrect: Since she was sick, she attended the party. Correct: Though she was sick, she attended the party. Rule : Nothing else should be followed by but, not by than. Incorrect: Mr Bureaucrat! This is nothing else than red-tapism. Correct: Mr Bureaucrat! This is nothing else but red-tapism. Rule : The correlative conjunctions indeed... but are used to emphasis the contrast between the first and the second parts of the statement. Incorrect: I am indeed happy with my school but it produces famous men. Correct: I am indeed happy with my school that it produces famous men. Rule : In a "not only ... but also ... "sentence, the verb should agree with the noun or pronoun mentioned second, that is, the one after but also because this is the part being emphasised. Incorrect: Not only the students but also the teacher were responsible for what happened in the class. Correct: Not only the students but also the teacher was responsible for what happened in the class. Rule : Not only ... but also always comes before the same part of speech. Incorrect: They not only gave me standing ovation but also a cash prize of `1000. Correct: They gave me not only standing ovation but also a cash prize of `1000. Rule : Such...as is used to denote a category whereas such...that emphasises the degree of something by mentioning its consequence. Incorrect: Each member of the alliance agrees to take such action as it deems necessary. Correct: Each member of the alliance agrees to take such action that it deems necessary. Rule : The conjunction “lest” is followed by “should”. Incorrect: Work hard lest you would fail. Correct: Work hard lest you should fail. Note: “Lest” is a negative word. There should be no use of “not”, “nothing”, “never”, “no” with lest”. Incorrect: Word hard lest you should not fail. Correct: Word hard lest you should fail. Rule : The conjunction so-as is used for comparison in a negative sentence whereasas is used in a positive or affirmative sentence. Incorrect: He is not as tall as his brother. 90 | P a g e

Correct: He is not so tall as his brother. Rule : The conjunction, “both” is followed by “and” and not by “as well as” or “along with”. Incorrect: Both the boys as well as the girls were absent. Correct: Both the boys and the girls were absent. Rule : The conjunction “unless” denotes condition whereas “until” denotes time. Incorrect: Until you work hard, you will not succeed. Correct: Unless you work hard, you will not succeed. Note : “Unless” and “Until” are negative words. There should be no use of “not” “nothing”, “never” with unless and until. Questions based on above concepts. Directions : In each of the following questions, find out which part of the sentence has an error. The error may be grammatical or idiomatic. If there is no mistake, the answer is ‘No error’. 1. The managing director well as (a)/ the Board members was in (b)/ favour of taking strict action (c)/ against the striking workers. (d)/ No error (e) 2. Unless he does not return (a)/ all the library books (b)/ he will not be relieved (c)/ from the service.(d)/ No error (e) 3. You must either tell me (a)/ the whole story or (b)/, at least, the (c)/ first half of it. (d)/ No error (e) 4. The old woman has had the (a)/ best medical facilities available (b)/ but she will not be cured unless she does not have (c)/ a strong desire to live.(d)/ No error (e) 5. Hardly had I reached (a)/ the airport where (b)/ I learnt about the powerful (c)/ bomb explosion.(d)/ No error (e) 6. The manager of the bank (a)/ was busy; so he asked them (b)/ to come and see him between two to (c)/ three in the afternoon.(d)/ No error (e) 7. Because he is physically (a)/ strong, therefore he was (b)/ selected for the school (c)/ boxing team.(d)/ No error (e) 8. Being a strict (a)/ vegetarian he depended (b)/ on milk and fruit and (c)/ had to cook for himself.(d)/ No error (e) 9. Even though the shirt (a)/ is rather expensive but (b)/ I wish to purchase (c)/ it with my own money.(d)/ No error (e) 10. Unless Ashish does not (a)/ return all the library books, (b)/ he will not be awarded (c)/ the pass certificate. (d)/ No error (e) Answers 1. a; Add ‘as’ before ‘well’ 2. a; Replace ‘does not return’ with ‘returns’ 3. e; No error 4. c; Replace ‘does not have’ with ‘has’ 5. b; Replace ‘where’ with ‘when’ 6. c; Replace ‘to’ with ‘and’ 7. b; Remove ‘therefore’ 8. c; Replace ‘and’ with ‘or’ 9. b; Remove ‘but’ 91 | P a g e

10. a; Replace ‘unless’ with ‘if’

Understanding of Adjective – Tips and Tricks

What is Adjective? An adjective is a word used with a noun ( or a pronoun) to add something for its meaning. The adjective may be used Attributively: when it is used along with the noun as an epithet or attribute; for example, There were dark clouds in the sky. Predicatively: when it forms part of the predicate; for example. The clouds in the sky were dark. Rule: The adjective is correctly used with a verb when some quality of the subject rather than of the action of the verb, is to be expressed. Incorrect: Flowers were plucked freshly. Correct: Flowers were plucked fresh. Rule: Due to and prior to should be used as predicative adjectives. When prepositional functions are to be performed, use because of in place of due to, and before in place of prior to. Incorrect: Wickets fell due to superb bowling. Correct: Wickets fell because of superb bowling. Kinds of Adjective Adjective of Quality: Also known as Descriptive Adjective, it shows the kind of quality of a person or a thing. For example, He loves her soft skin. Adjective of Quantity: It is an adjective which shows how much of a thing is meant. For example, There is little sense in it. Adjective of Number: Also known as Numeral Adjective, it shows how many persons or things are meant, or in what order a person or a thing stands. For example, The face of a clock has sixty divisions. Numeral Adjectives are of three kinds: Definite Numeral Adjective: It denotes an exact number. It may be a cardinal denoting how many; as, one, two, three, etc. It may be an ordinal denoting the order, as first, second, third etc. Rule: When a number together with a unit of measurement is to be used as an adjective, it is a compound word and the unit of measurement is taken in the singular. Incorrect : It is a twenty-kilometres walk. Correct : It is a twenty-kilometre walk. Rule: Definite Numeral Adjective always takes plural countable noun. Incorrect: When I was in Punjab, I had twenty money. 92 | P a g e

Correct: When I was in Mumbai, I had twenty rupees. Rule: If in a sentence one noun is used for all ordinals and the first ordinal has 'the' while, other not, the noun will be in the plural. Incorrect: I have read the literature of the seventeenth and eighteenth century. Correct: I have read the literature of the seventeenth and eighteenth centuries. Indefinite Numeral Adjective : It does not denote an exact number. For example. There are many petitions lying on the tables. Rule: Many takes plural noun while "Many a/an singular noun. Incorrect: He has observed many an uncommon phenomena. Correct: He has observed many an uncommon phenomenon. Rule: The few is used before plural countable noun and the little before uncountable noun. Both follow a clause. Incorrect: I gave the beggar a little coins I had. Correct: I gave the beggar the few coins I had. Distributive Numeral Adjective: It refers to each one of a number. Distributive numeral adjectives are the same as distributive pronouns (see Pronoun). They only differ in function. For example, Each time he missed the bus. Demonstrative Adjective: It points out which person or thing is meant. For example, This table is made of wood. Rule: The plural forms these and those are often wrongly used with the singular nouns kind and sort. Incorrect: These kind of clothes are not acceptable. Correct: This kind of clothes are not acceptable. Interrogative Adjective: When an interrogative pronoun is used with a noun to ask a question, it is called an interrogative adjective. For example, What options are available after graduation? Exclamatory adjective: It is used with a noun in-exclamatory sentence. For example, What a show! Confusing Words Later, latter, latest, last: Later and latest refer to time. Latter and last refer to position. Incorrect: The later part of the novel is not neatly written. Correct: The latter part of the novel is not neatly written. Incorrect: What is the last news? Correct: What is the latest news? Elder, older, eldest, oldest: Elder and eldest are used of members of the same family. Besides, both of them are used only attributively (that is, before the noun). Older and Oldest are used in the general sense. Incorrect: I have an older brother. Correct : I have an elder brother.. (Since brother is a member of the same family) Incorrect : Mohan is the eldest boy in the town. 93 | P a g e

In a town there may be so many families. We are talking of boys in general sense. So eldest should be replaced by oldest. Correct : Mohan is the oldest boy in the town. Degree of Comparison Degrees are of three kinds. 1. Positive Degree 2. Comparative Degree 3. Superlative Degree Rule: When a selection is made out of the two, and two is given in the sentence, the is used before the adjective denoting comparison. Incorrect: Shikha is better of the two sisters. Correct: Shikha is the better of the two sisters. Rule: Preferable takes to and is not preceded by more. Incorrect: Coffee is more preferable than tea in Brazil. Correct: Coffee is preferable to tea in Brazil. Rule: When two adjectives in different degrees of comparison are used in the same sentence, each should be complete in itself. Incorrect: My father is as rich if not richer than your mother. Correct: My father is as rich as if not richer than your mother. Rule: If two adjectives are connected by and, they should be in the same degree throughout. Incorrect: Raj is more intelligent and most active. Correct: Raj is more intelligent and more active. Rule: The comparative in -er is not used when we compare two qualities of the same person or thing. Incorrect: Mohan is wiser than strong. Correct: Mohan is more wise than strong. Rule: When two objects are compared with each other, the latter term of comparison must exclude the former. Incorrect: Satish is cleverer than any student in his class. Correct : Satish is cleverer than any other student in his class. Rule: The comparative degree is generally followed by than but the following comparative adjectives are followed by the preposition to : inferior, superior, prior, anterior, posterior, senior, junior. Incorrect: India's fielding is inferior than South Africa's. Correct: India's fielding is inferior to South Africa's. Rule: Adjectives expressing qualities that do not admit of different degrees cannot be compared. Incorrect: Our field is more square. Correct: Our field is square. 94 | P a g e

Rule: Double comparatives and superlatives are to be avoided. Incorrect: Zarda mangoes are more sweeter than Maldah ones. Correct: Zarda mangoes are sweeter than Maldah ones. Test Yourself Directions: In the following questions, some of the sentences have errors and some have none. Find out which part of the sentence has an error. The letter corresponding to that part is your answer. If there is no mistake, the answer is ‘No error’. 1. Firstly you should a)/ think over the meaning of the words b)/ and then use them. c)/ No error d) 2. The driver tried his best a)/ to avert the accident by b)/ bringing the car to a suddenly stop. c)/ No error d) 3. The Sunrise Hotel was a)/ fully equipped to offer b)/ leisure stay c)/ to its clients. d)/ No error e) 4. The technician reminded a)/ them to have a thoroughly cleaning of the b)/ machine after each use. c)/ No error d) 5. I am much glad a)/ that you have b)/ won the prize. c)/ No error d) 6. My observation is that a)/ between Vivek and Shashi, b)/ Vivek is the most intelligent. c)/ No error d) 7. It very often a)/ happens that b)/ a man who talks most does little. c)/ No error d) 8. From all accounts a)/ I learn that b)/ he is the best and honest member of the new Cabinet. c)/ No error d) 9. He is a)/ too intelligent b)/ to make a mistake. c)/ No error d) 10. The flood situation this year a)/ is worst than b)/ that prevailed in the last year, c)/ No error d) Answers 1.a; Replace ‘Firstly’ with ‘First’ 2. c; Replace ‘suddenly’ with ‘sudden’ 3. c; Replace ‘leisure’ with ‘leisurely’ 4. b; Replace ‘thoroughly’ with ‘thorough’ 5. a; Replace ‘much’ with ‘very’ 6. c; Replace ‘the most’ with ‘more’ 7. c; Replace ‘most’ by ‘much’ 8. c; The correct form is ‘he is the best and most honest member....’ 9. d; No error 10. b; Replace ‘worst’ with ‘worse’

95 | P a g e

Understanding of Verb – Tips and Tricks Verb A Verb is a word that tells or asserts something about a person or thing. It tells about the person or thing in the state of (a) doing, (b) being or (c) being acted up on. Thus a verb is a doing or being word. Kinds of verbs : A Transitive verb is a verb that denotes an action which passes over from the doer or agent to an object, Transitive means passing over. For example, India won the World Cup. An Intransitive verb is a verb that denotes an action which does not pass over to an object, or that expresses a state of an object, or that expresses a state of being. Intransitive means not passing over. For example, The sun shines brightly. Rule: When an intransitive verb is used in a causative sense, it becomes transitive. 1. He sat in a chair. 2. He sat me on chair. Rule: Verbs like, fall, lie, rise and sit are made transitive by a slight change in their spellings. The transitive is the causative of the corresponding intransitive verb. Incorrect: The poachers were caught yesterday when they fell trees illegally. Correct : The poachers were caught yesterday when they felled trees illegally. Verbs of Incomplete Predication The intransitive verb which requires something to make the sense complete is called a verb of incomplete predication. For example be, become, seem, grow, taste, appear. That which is required to make the sense (of these verbs) complete is known as the complement of the verb or the completion of the predicate. For example, The family appears. Rule: When the subjective complement is a noun (or a pronoun), it is in the same case as the subject, that is, in the nominative case. For example The lawyer called the witness a liar. Rule: When the objective complement is a noun it is in the objective case in agreement with the object. Subject-Verb Agreement Language has its base in logic and harmony. The subject and the verb are the essential parts of a sentence. It is imperative that the verb agree with the subject in number and person.

96 | P a g e

Rule: Two or more singular subjects connected by and usually take a verb in the plural. Incorrect: Computer and telecommunication has metamorphosed information technology. Correct: Computer and telecommunication have metamorphosed information technology. Rule: If two singular nouns refer to the same person or thing, the verb must be singular. Incorrect : The poet and critic have been honoured. Correct : The poet and critic has been honoured. Rule: If two subjects together express one idea, the verb is in the singular. Incorrect: Bread and butter are essential for one's life. Correct: Bread and butter is essential for one's life. Rule: If the singular subjects are preceded by each or every, the verb is usually singular. Incorrect: Every man and woman in a family are responsible for the upbringing of a child. Correct: Every man and woman in a family is responsible for the upbringing of a child. Rule : Two or more singular subjects connected by or, nor, either ... or, neither... nor take a verb in the singular. Incorrect: Either Manoj or Madhukar have the key to this problem. Correct: Either Manoj or Madhukar has the key to this problem. Rule: When the subjects, joined by or or nor are of different numbers, the verb must be plural, and the plural subject must be placed next to the verb. Incorrect : Neither Rekha nor her friends was present at the party. Correct : Neither Rekha nor her friends were present at the party. Rule: When the subjects, joined by or or nor are of different persons, the verb agrees in person with the one nearest to it. Incorrect : Either she or I pays the fees. Correct : Either she or I pay the fees. Error of Proximity The verb is made to agree in number with a noun near it instead of with its proper subject. This is wrong and should be avoided. Incorrect : The behaviour of the students were not proper. Correct: The behaviour of the students was not proper. Rule: Words joined to a singular subject by with, as well as, etc. are parenthetical. The verb should therefore be put in the singular. Incorrect: The guru as well as his disciples are committed to celibacy. Correct : The guru as well as his disciples is committed to celibacy. 97 | P a g e

Rule: Either, neither, each, everyone, many a must be followed by a singular verb. Incorrect: Many a man have resigned in crisis. Correct: Many a man has resigned in crisis. Rule: When a plural noun denotes some specific quantity or amount considered as a whole, the verb is generally singular. Incorrect: Five hours are too short a time to judge one's character. Correct: Five hours is too short a time to judge one's character. The Infinitive Usually a verb is limited by the number and person of its subject. For example, He plays cricket. They play cricket. A verb, which is not limited by person and number as a verb that has a subject, is called the verb infinite or the infinitive. Infinitives are of two kinds. 1. Simple Infinitive and 2. Gerundial or Qualifying Infinitive. Simple Infinitive - When the infinitive is used like a noun. (i) as the subject of a verb. To name is to give an identity. (ii) as the object of a verb. I have decided to leave the place. (iii) as the complement of a verb. Her hobby is to dance. (iv) as the object of a preposition. He is about to go. (v) as the complement of the object. She heard her weep. Gerundial or Qualifying Infinitive-when the infinitive is used for the following purposes: (i) to qualify a verb. He slept to refresh himself. (ii) to qualify an adjective. Tea is easy to prepare. (iii) to qualify a noun. He has a point to make. (iv) to qualify a sentence. To be very frank, I hate your smile. Rule: The infinitive is used without to after verbs like help, watch, bid, dare, hear, let, make, need and see. Incorrect : I bid him to quit the organisation. Correct: I bid him quit the organisation. Rule: If, however, the structure is do + dare or do + need, the infinitive is used with to. Incorrect: You do not need worry about us. Correct: You do not need to worry about us. Rule: The following verbs are followed by the infinitive. Agree, arrange, attempt, care, cease, consent, decide, determine, endeavour, fall, forget, hesitate, hope,learn, manage, neglect, prepare, promise, propose, refuse, regret, remember, seem, swear, undertake. Incorrect: I agree for helping you out in trouble. Correct: I agree to help you out in trouble. 98 | P a g e

Rule: Prepositions but and except take the infinitive without to. Incorrect: There is no alternative but to help him out. Correct : There is no alternative but help him out. Rule: Expressions would rather, would sooner, rather than, sooner than, had better are followed by infinitive without to. Incorrect : I would rather to go for batting. Correct : I would rather go for batting. Participle 1. Present Participle 2. Past Participle 3. Perfect Participle Rule: Present participle is used when all the parts of a sentence have the same subject; Incorrect: Opened the drawer, he took out his gun. Correct: Opening the drawer, he took out his gun. Rule: When a sentence has two different subjects, avoid using present participle. Instead a clause should be used in such instances. Incorrect: While going to school, a dog jumped at me. Correct: While I was going to school, a dog jumped at me. Rule: Some past participles are used only as adjectives and qualify a noun. They can be used as past participles with changed spellings. For example, shrunken, drunken, sunken, graven, molten. Incorrect: He and his brother have drunken wine last night. Correct: He and his brother have drunk wine last night. Rule: Perfect participle is used when one activity is completely over. Having worked hard, he felt tired. Auxiliary Verb Auxiliary literally means giving help, So an auxiliary verb is one that helps the main verb form in tenses, moods, voices etc. The following verbs are auxiliaries: is, are, am, was, were, be, can, could, dare, do, does, did, have, has, had, may, might, must, need, ought, shall, should, will, would, used to. 1. May implies permission, doubt or possibility. May I come in? (permission) It may rain tonight. (possibility) 2. Might is the past form of “may” and it implies more doubt than “may”. If the clouds are salted, the rains might come. 3. Can is used to express ability. Ex. He can do the work. 4. Could is the past form of can but it does not necessarily represent past time. Often it implies a more uncertain condition. He could refuse, but she never does. 99 | P a g e

5. Should expresses the idea that something must be done or is important. Should and ought to have similar meanings, but ought is followed by “to” “Ought to” has a more objective force and is used when we are talking about laws, duties and regulations. We ought to see her tomorrow. 6. Must is used to give strong advice or orders. Ex. He really must stop drinking. 7. Do is used to make question and negative forms of ordinary verbs. Do you know him? I don’t like swimming. “Do” is always used in question-tags. You know painting, do you? Test Your Understanding In Verb Directions: In the following questions, some of the sentences have errors and some have none. Find out which part of the sentence has an error. If there is no mistake, the answer is ‘No error’. 1. Many a student (a)/ are frustrated (b)/ because of unemployment. (c)/ No error (d) 2. It is true (a)/ that the poor is unable (b)/ to get nourishing food even today. (c)/ No error (d) 3. People have a right to criticise (a)/ but at the same time each of them (b)/ have to remember his duty also. (c)/ No error (d) 4. The child (a)/ picked up a burned paper (b)/ from the street. (c)/ No error (d) 5. Never did this exhibition of contempt and anger ceased (a)/ so long as British rule remained (b)/ a live thing in India. (c)/ No error (d) 6. The teacher was angry (a)/ when he found that (b)/ you are not there. (c)/ No error (d) 7. I would like you to (a)/ complete this assignment (b) before you will leave for Mumbai. (c)/ No error (d) 8. Ordinarily, when in difficulty (a)/ Ruchi prefers keeping her counsel (b)/ than running about taking advice. (c)/ No error (d) 9. One of the drawbacks (a)/ of modern education are (b)/ that it does not encourage original thinking. (c)/ No error (d) 10. I enquired of him (a)/ why he is so negligent (b)/ in his studies. (c)/ No error (d) Answers 1. b; Replace ‘are’ with ‘is’ ‘Many a’ takes singular subject and singular verb. 2. b; Replace ‘is’ with ‘are’ Here, ‘poor’ stands for ‘poor people’. 3. c; Replace ‘have’ with ‘has’ 4. b; Replace ‘burned’ with ‘burnt’ 5. a; Replace ‘ceased’ with ‘cease’ 6. c; Replace ‘are’ with ‘were’ 7. c; Remove ‘will’ 8. c; Replace “than” with “to” 9. b; Replace ‘are’ with ‘is’ 10. b; Replace ‘is’ with ‘was’ 100 | P a g e

Pronouns – Tips and Tricks Pronoun A pronoun is used in place of a noun or nouns. Personal Pronoun I, we, you, he, she, it, they are called personal pronouns because they stand for the three persons. i.The person speaking (I, We) ii.The person spoken to, and (You) iii. The person spoken of. (He, She, it, They)

Pronouns are used so that our language is not cumbersome with the same nouns being repeated over and over in a paragraph. Subject Pronoun: (Subjective case) (I, we, you, he, she, it, they) Example: She is at work. ’She’ is main subject of the sentence, hence in the sentence, ‘She’ is the subjective personal pronoun. Objective pronoun (objective case) Example: He will meet us later. ‘Us’ is the objective personal pronoun, as it is the object of the verb ‘meet.’ Possessive pronoun (possessive case) Example: That is our clubhouse. ‘Our’ shows the possession of the object ‘clubhouse’. Gender Example: He went to the market. He is used for male gender. Other examples - (his, him, he etc.) Example: She is doing the laundry. ‘She’ is used for female gender. (Her, hers, etc.) Example: It is important to them. ‘It’ is gender neutral as it shows an object, ‘Them’ is also gender neutral as ‘Them’ can consists of both genders. Others gender neutral pronouns are - (Their, they, its.) 101 | P a g e

Number Singular Pronoun - where the pronoun is only referring to one specific Noun. Example: That book belongs to me. Plural Pronoun - where the pronoun is used to refer to a number of nouns. Example: That is Their book, not yours. Reflexive Pronoun “They are object pronouns that we use when the subject and the object are the same Noun.” Example: I told myself not to bet all my money on one horse. Example: The robber hurt himself chasing me through the alley. “Reflexive pronouns are those which are used to indicate a noun which has been used in an earlier part of the same sentence. (myself, themselves, yourself, ourselves, herself, himself, itself.) Example: She blamed herself for the accident. He injured himself today. Emphatic or Intensive Pronoun “These pronouns are used to emphasize a Noun or pronoun. (myself, himself, herself, themselves, itself, yourself, yourselves and ourselves.) Example: He himself is his worst critic. “These pronouns act as appositives of nouns or pronouns for the sake of emphasis,” Example: You yourself wrote those words. This request came from the employee themselves. They themselves know that the Prank was in bad taste. Avoid reporting things that you yourself haven’t witnessed. Demonstrative Pronoun “Demonstrative pronouns are used to show or identify one or a number of nouns that may be far or near in distance or time. They are only four in number (This, that, these, those) This, that are called Singular demonstrative pronoun These, those are called Plural demonstrative pronouns. Example: That is a beautiful house. They can also be used to show an unspecified quantity in a sentence. Example: These were made by me. (These is showing an unspecified quantity of something that was made by a person.) Example: Everyone remembers those days. (Those is showing a particular time or period of days in the past, it is being used in place of a noun that could be - school, summer, college etc.) Example: This is what he is charging. This is used as pronoun in place of a number. These pronouns point out someone or something. They are identical in form to demonstrative adjective/determiners. The difference is that • A demonstrative pronoun stands alone (because it is a substitute for a noun or noun phrase) • But a demonstrative adjective is accompanied by the noun it modifies. 102 | P a g e

Example: She gave me this gift. (This - demonstrative adjective) I like this. (This - demonstrative pronoun) (More example of demonstrative pronoun) These are my children. That is a good idea. The streets of Delhi are more crowded than those of Mumbai. Indefinite Pronoun “These pronouns do stand for some person or thing, but we don’t know for exactly whom.” When we say, “somebody stole my watch.” (We don’t know to whom the word ‘somebody’ refers to. The word ‘somebody’ is an indefinite pronoun. Example: One should speak the truth. Somebody immediately called the doctor. Anybody can solve this problem. Nobody was present. Many are called, but few are chosen. Do good to others. (Few, all, some, none, everything- indefinite pronouns) Distributive Pronoun “These Pronouns refer to individual elements in a group or a pair, one individual at a time.” Example: Each of the boys gets a prize. Either of these roads leads to the railway station. Either of you can go. Neither of the accusations is true. You may bring any of your friends. None of our students failed last year. Each, either, neither are called distributive pronouns because they refer to persons or things, one at a time. Eachis used to denote every one of a number of persons or things taken singly. Either means the one or the other of two. Neither means not the one nor the other of two. It is negative of either. Either and Neither should be used only in speaking of two persons or things. When more than two are spoken of (Any, No one, and none) should be used. Reciprocal Pronoun Each and one really belong to the subject, Other and another are objects, but Each other and one another have become compound pronouns, (and are called reciprocal pronouns) and are rarely separated even by a preposition) 103 | P a g e

Example: The brothers quarreled with each other. They all gave evidence against one another. Jamie and Jack always sit beside each other in break. They haven’t seen one another since last year. Relative Pronoun These pronouns are used to connect a clause or phrase to a noun or pronoun. These are: who, whom, which, whoever, whomever, whose, whichever and that. The driver who Ran the stop sign was careless. Which and that are generally used for objects. Who and whom are used for people and whose is used to show possession. Example: She will choose the color which looks good on everyone. She is complaining to whoever she comes across nowadays. There is a car in the parking lot that someone has painted a bright pink. Is there anyone here whose mobile phone has a signal? I met Hari who had just returned. I have found the pen which I lost. There is the book That you lent me. Interrogative Pronoun Who, whom, which and what are interrogative pronouns as they are used to ask questions about a person or object that we do not know about. Compounds of these words are made by attaching (-ever) to the words to strengthen the emphasis on the word. Example: Which one would you like? What is your Name? Who will be managing the bullet? Whom did you tell about this? Whoever could have done this? Whichever one will you choose? ‘Who’ is always the subject of the verb. ‘Whom’ is never the subject of the verb. It is object of the verb. It is used to show the person to or for whom the action is being done. whom were you meeting with?

104 | P a g e

How to make Verbs agree with Numbers Verb-Number Agreement The Problem Which is correct: "The cost of new books is dropping,” or “The cost of new books are dropping"? The Explanation To determine whether to use is or are in such a sentence, you need to understand that you are dealing with the Number of a verb. There are two numbers in grammar &ndash singular and plural. A singular verb is used with singular subject and a plural verb is used with plural subject.

The Subject A subject is one of the two parts into which every sentence may be divided. The other part is the predicate. The Subject of a sentence is what or whom we are talking about and the predicate of a sentence is what we are saying about the subject. Consider these examples: 1. The Goa summit was held in October. What are we talking about? The Goa summit. What we are talking about: (It) was held in October. Thus the sentence is divided in this way: "The Goa summit" [subject] "was held in October." [predicate]. 2. Truman was elected Vice-President of the United States in 1944. Whom we are talking about: Truman [subject]. What we are talking about him: (he) was elected Vice-President of the United States in 1944 [predicate]. The subject of a sentence may consist of single noun or pronoun, as in: • Rumi was operated for tumour. • Richa is beautiful. • Houses could once be built in eight weeks. • Flower blooms in spring. This single noun or pronoun is called the subject of the verb. Subject of the verb is the single word (noun or pronoun) which is directly, and exclusively, influences the number of the verb. The subject of the sentence (don’t confuse with the subject of the verb though they frequently identical) may contain not only the subject of the verb, but also one or more adjectives, conjunctions, adverbs, prepositional phrases, or other assorted parts of speech. 105 | P a g e

To spot the difference between subject of the verb and subject of the sentence, see the build up a sentence in below examples. Women are fascinating creatures. Subject of the verb and subject of the sentence: women. Let us add a prepositional phrase: Women of beauty are fascinating creatures. Subject of the sentence : Women of beauty. Subject of the verb : Women. Now, adding an adverb, a conjunction, and a second prepositional phrase: Healthy young women of great beauty and delightfully curvaceous lines are fascinating creatures. Subject of the Sentence: Healthy young women of great beauty and delightfully curvaceous lines. Subject of the verb : women. Key Point : Subject of the verb is single noun or pronoun; whereas, subject of the sentence may be single word, noun or pronoun as well as combination of adjectives, conjunctions, adverbs, prepositional phrases or other part of speech. Now, see these examples: 1. Are women fascinating creatures? Verb: Are Subject of the verb : women The subject may be split a verb phrase: 2. How do women improve their health? [verb phrase] Verb phrase: Do improve Subject of the verb : Women 3. There are many reasons for admiring the girl child. Verb: Are Subject of the verb : reasons 4. Beyond the Alps lies Italy. Verb: lies Subject of the verb: Italy. In above example, you can see the verb precedes its subject. * So, The subject of the verb need not precede the verb. To find subject of the sentence is a bit easy; however, finding the subject of the verb &ndash that is, the single noun or pronoun which drives the number of the verb- is a bit difficult. So, note these important points: * The subject of a sentence may, and often does, contain one or more prepositional phrases. * The single noun or pronoun subject of the verb is never found in within or as part of the prepositional phrase. Go through the each of the sentences given below: 1. In China’s Civil War, Nanking claimed further gains. The verb is claimed and its subject is Nanking. 106 | P a g e

2. Business and industrial leaders in Kolkata predicted a rush of applications for price adjustments. Here we have a rather long subject of the sentence (all the words up to the verb predicted) but usual one word subject of the verb is Leaders. 3. The cost of automobiles has just risen 27 per cent. The subject of the verb is Cost. Note cost is singular and that the verb has risen is also singular. 4. There go my brothers. Subject of the verb is Brothers. This is an instance in which the subject of the verb follows the verb (go) i.e. verb preceded the subject of the verb. There is an adverb. 5. One of my best friends was killed yesterday. One of my best friends is prepositional phrase. One is pronoun and so subject of the verb. Note the both subject and verb are singular. 6. The captain, with all his sisters, brothers, aunts, and cousins by dozens, is going abroad. The subject of the sentence is made up of all words up to is, the subject of the verb is single word captain. Since, captain is singular, is going is singular. 7. Ram, like his mother and father, has brown hair. Like is a preposition, and so nouns mother and father have no influence on the number of the verb. Ram is singular so the verb has is also singular. 8. Has either of your parents married since divorce? Of your parents is a prepositional phrase. Either in this sentence means either one ; therefore it is singular and takes singular verb has married. 9. Was each of the criminals tried separately? Like either, each means each one. It is a singular pronoun and takes singular verb was tried. 10. Divya’s beauty, as well as her versatility and charm, has made her a very popular girl in the college. As well as her versatility and charm is a prepositional phrase introduced by the tripleply preposition as well as. So, subject of the verb is Divya. Important Principles • The subject of the verb is usually a single-word noun or pronoun. • Such noun or pronoun is not found with a prepositional phrase. • Generally, the subject precedes the verb but in certain sentence patterns may follow the verb. • The number of the verb depends on the number of the single-word subject • Singular subjects take singular verbs and plural subjects take plural verb. Back to the starting Problem "The cost of new books is dropping," or "The cost of new books are dropping"? Subject of the sentence: The cost of new books • Subject of the verb : cost Cost is singular so it must take singular verb. • Hence correct sentence: The cost of new books is dropping. The Conclusion A verb must agree in number with single-word subject - noun or pronoun. 107 | P a g e

Understanding of Noun NOUN Kinds of Noun: 1. Proper Noun: a proper noun is the name of some particular person or place. Ex- Ram , Shyam, Delhi. 2. Common Noun: A Common noun is a name given in common to every person or thing of the same kind or class. Ex- boy, girl, teacher etc. 3. Collective noun: A collective noun is the name of a group of persons or things. Ex- army, committee, crowd etc. 4.Abstract Noun: a noun denoting an idea, quality, or state rather than a concrete object. Ex- strength, innocence, fear, judgment. Etc. 5. Material Noun:Material noun is the name given to the material, substance or things made up of The alloy. Ex- cotton, gold, silver etc. NOUN for Gender 1. Masculine Gender: A noun that denotes male animal is said to be of the Masculine Gender. Ex- Man, boy, Tiger, Sun etc. 2. Feminine Gender: A noun that denotes a female animal is said to be of the Feminine Gender. Ex- woman, girl, nature, lioness etc. 3. Common Gender: A noun that denotes either a male or a female is said to be of the common gender. Ex- Parent, child, student, cousin etc. 4.Neuter Gender: A noun that denotes a thing without life , neither male nor female , is said to be of the Neuter gender. Ex- Book, Pen, room etc. NOUN for NUMBER 1. Singular Noun: A noun that denotes one person or thing, is said to be in the Singular Number. Ex- pen, cow, boy etc. 2. Plural Noun: A noun that denotes more than one person or thing , is said to be in the plural Number. Ex- Pens, Boys, Cows etc NOUN: Countable / Uncountable Countable nouns are the names of objects, people etc that we can count. Ex- book, apple, doctor, horse etc. Uncountable nouns are the names of things which we can’t count. They mainly denote substance and abstract things. Ex- milk, oil, sugar, gold, honesty etc. 108 | P a g e

NOUN for CASES The case of a noun tells us about the position of that noun or pronoun in a sentence. In English, there are five cases. Nominative case: A noun is said to be in the nominative case if it is the subject of a verb. Ex- Ram is an intelligent boy. Objective case: Nouns or Pronouns are said to be in objective case if they are the direct object of verbs or the objects of the preposition. Dative case: A noun is said to be in Dative case if it is the indirect object of the verb. Rohan brought me a flower. (‘Me’ is in dative case) Possessive case: A noun is said to be in the possessive case if it denotes possession or ownership. Ex- This is your pencil. (‘your’ is in possessive case) Vocative case :A noun or pronoun is said to be in vocative case if it is used to call ( or to get attention of a person or persons) Ex- Mr. Mallya , people are waiting for you in the hall. (Mr. Mallya is in vocative case) NOUN in Apposition When one noun follows another to describe it, the noun which follows is said to be in apposition to the noun which comes before it. Ex- Ram , our captain, made fifty runs. Kabir , the great reformer, was a weaver. RULES AND EXAMPLES 1. Some nouns always take a singular verb. Scenery, advice, information, machinery, stationery, furniture, abuse, fuel, rice, gram, issue, bedding, repair, news, mischief, poetry, business, economics, physics, mathematics, classic, ethics, athletics, innings, gallows. (A) The scenery of Kashmir are enchanting. (Correct use- is) (A) He has given advices. (Correct use- advice) 2. Some nouns are singular in meaning, but they are used as plural nouns and always take a plural verb. Cattle, gentry, vermin, peasantry, artillery, people, clergy, company, police. (A) The cattle is grazing in the ground. (correct use- are) (B) Police has controlled the situation. ( correct use- have) 3. Some nouns are always used in a plural form and always take a plural verb. Trousers, scissors, spectacles, stockings, shorts, measles, goods, alms, premises, thanks, tidings, annals, chattels, etc. (A) Where is my trousers? (correct use- are) (B) Spectacles is a costly item. ( correct use- are) 109 | P a g e

4. There are some nouns that indicate length, measure, money, weight or number. When they are preceded by a numeral, they remain unchanged in form. Foot, meter, pair, score, dozen, head, year, hundred, thousand, million. (A) It is a five years degree course. (correct use- year) (A) I have seven dozens of shoes. (correct use- dozen) 5. Collective nouns such as jury, public, team, committee, government, audience, orchestra, company, etc. are used both as singular and Plural. It depends on the usage. (A) The jury was divided in their opinion. (correct use- were) (A) The team have not arrived yet. (correct use- has) 6. Some nouns have one meaning in the singular and another in the plural: a. Advice = counsel, advices = information b.Air = atmosphere, airs = proud c. Authority = command, authorities = persons in power d. Good = wise, goods = property e. Iron = metal, irons = fetters, chains f. Force = strength forces = army g. Content = satisfaction, contents = things contained h. Respect = regards, respects = compliments i. Work = job works = compositions, factories. 7. People are often confused or they commit mistakes in the use of certain nouns. (A) Lecturership is wrong: lectureship is correct. (B) Freeship is wrong; free studentship is correct. (C) Boarding is wrong; boarding house is correct. (D) Family members is wrong; members of the family is correct. (E) English teacher is wrong; the teacher of English is correct. (F) Cousin - brother or sister is wrong; only cousin is correct. (G) Wages means punishments when used in the singular. Ex- The wages of sin is death. (H) It also means charges for the labour when used in the plural sense. Ex- The wages of daily workers have been raised. 8. A pronoun must agree with its antecedent in person, number and gender. Ex- Every student must bring his luggage. All students must do their home work. Each of the boys must carry his own bag. 110 | P a g e

9. While using ‘everybody’ ‘everyone’, ‘anybody’, and ‘each’ the pronoun of the masculine or the feminine gender is used according to the context. I shall be happy to help each of the boys in this practice. But when the sex is not mentioned, we use the pronoun of the masculine gender. Anyone can qualify this exam if he tries. Each of the six boys in the class has finished their tasks. (Incorrect) Each of the six boys in the class has finished his task. (Correct) 10. The pronoun ‘one’ must be followed by ‘one’s’. One must finish his homework in time. (Incorrect) One must finish one’s homework in time. (Correct) 11. Enjoy, apply, resign, acquit, drive, exert, avail, pride, absent, etc., when used as transitive verbs, always take a reflexive pronoun after them. When ‘self’ is added to ‘my’, ‘your’, ‘him’, ‘her’, and ‘it’, and ‘selves’ to our and them - they are known as reflexive pronouns. He absented from the meeting. He absented himself from the meeting. 12. ‘Who’ denotes the subject and ‘whom’ is used for the object? Who : It’s the subject of a verb e.g., Who gave you that book? It’s a predicate nominative (a noun in the predicate that renames or refers to the sentence’s subject) e.g.,This is who I am. Whom is an objective pronoun, which is a pronoun that receives the action of a verb. It also has two main uses: It is the object of a verb e.g., Whom should I call? It is the object of a preposition e.g., From whom did you get this information? 13. When two or more singular nouns are connected by ‘either or’; ‘neither nor’, ; and ‘or’, the pronoun used is singular. Either Rohan or Sohan will give their bike. (Incorrect) Either Rohan or sohan will give his book. (Correct) 14. When a singular and a plural noun are joined by ‘or’, ‘nor’, the pronoun must be plural. Either the student or his teachers failed in his duty. (Incorrect) Either the student or his teachers failed in their duty. (Correct) 15. ‘Whose’ is used for living persons and ‘which’ for lifeless objects. Which novel did you select? Whose photograph is lying there? 16. ‘Each other’ is used when there are two subjects or objects and ‘one another’ when there are more than two. Ram and Sita loved each other. Those five students, who are sitting there, love one another. 17. When a pronoun stands for a collective noun, it must be in the singular number and in the neuter gender if the collective noun is viewed as a whole.but  if it gives an idea of different entities , plural pronoun is used. The jury gave ‘its’ verdict. Here the ‘jury’ gives the idea of one whole. If the collective noun conveys the idea of separate individuals comprising the whole, the pronoun standing for it must be plural. The jury were divided in their opinions. In above sentence , the ‘jury’ gives the idea of several individuals. 111 | P a g e

18. If pronouns of different persons are to be used together in a sentence, the serial order of persons should be as follows; Second person(2) + third (3)+ first person(1) in normal sentences. But when mistake or fault is expressed in the sentence, the order should be; first person(1) + second person(2) + third person(3). RULE-231 You, he and I have finished the work. (Normal idea) I, you and he are to blame. ( here Confession of guilt is expressed , it’s a negative idea, hence order is 123) 19. ‘Some’ is used in affirmative sentences to express quantity or degree. ‘Any’ is uses in negative or interrogative sentences. I shall buy some apples. I shall not buy any apples. Have you bought any apples? But ‘some’ may be correctly used in interrogative sentences which are, in fact, requests. Will you please give me some water? 20. The use of ‘few’, ‘a few’’ and ‘the few’ should be used with care. They denote ‘number’. ‘Few’ means ‘not many’, ‘almost nothing’. A ‘few’ is positive and means ‘some at least’. ‘The few’ means ‘whatever there is’. A few men are free from fault. (Incorrect) Few men are free from fault. (Correct) (Here the sense is negative and thus ‘a few’ is wrong.) Few teams will qualify for the world cup. (Incorrect) A few boys will pass in the examination. (Correct) Here the sense is positive and thus ‘few’ is incorrect. I have already read a few books that are in the library. (Incorrect) I have already read the few books that are in the library. (Correct) Here the sense is ‘whatever there is’. ’everything that is in the library’ 21. Use of ‘less’ and ‘fewer’ ‘Less’ denote quantity and ‘fewer’ denote number. No less than fifty students were selected. (Incorrect) No fewer than fifty students were selected. (Correct) There are no fewer than five liters of water in the bottle. (Incorrect) There are no less than five liters of water in the bottle. (Correct) 22. Use of little, a little, the little ‘Little’ means ‘hardly any’ There is little hope of his recovery. (Incorrect) There is a little hope of his recovery. (correct) ‘A little’ means ‘some’, though not much. Little knowledge is a dangerous thing. (Incorrect) A little knowledge is a dangerous thing. (Correct) ‘The little means ‘not much but all there is’. Little water that is in the bottle may be used for the child. (Incorrect) The little water that is in the bottle may be used for the patient. (Correct) 112 | P a g e

23. Use of elder, older. ‘Elder’ is used for persons of same family. ‘Older’ refers to persons as well as things and is followed by ‘than’. Rohan is elder than all other boys of this area. (Incorrect) Rohan is older than all other boys of this area. (Correct) Sabu is my older brother. (Incorrect) Sabu is my elder brother. (Correct) 24. Normally ‘than’ is used in the comparative degree, but with words like superior, inferior, senior, junior, prior, anterior, posterior and prefer ‘to’ is used. Sara is junior than Neeta.( Incorrect) Sara is junior to Neeta. (Correct) I prefer reading than walking. (Incorrect) I prefer reading to walking. (Correct) 25. when a comparison is made by using a comparative followed by ‘than’, the word ‘other’ must exclude the thing compared form the class of things with which it is compared. He is better than any man. (Incorrect) He is better than any other man. (Correct) ‘Any man’ includes the man himself and thus the sentences will be awkward. 26. In some cases, the comparison is subtle and must be given proper attention. Ex- The climate of Delhi is better than Mumbai. (Incorrect) Here the comparison should be between the climate of Delhi and the climate of Mumbai. The climate of Delhi is better than the climate of Mumbai. (Correct) Or The climate of Ranchi is better than that of Gaya. (Correct) (Here, ‘That of’ means ‘the climate of’) If the traits are in plural, it will be ‘those of'. The products of Reliance are better than those of Suzuki. The scenery of Kashmir is better than Shimla. (Incorrect) The scenery of Kashmir is better than that of shimla. (Correct) 27. ‘many a’ is always followed by the singular verb. Many a student were drowned in the river. (Incorrect) Many a student was drowned in the river. (Correct) 28. If the subject is ‘the number of’ the singular verb is used. And when the expression (‘a +number+of) is used , plural verb is used. The number of students are very small. (Incorrect) The number of students is very small. (Correct) A number of girls has passed in the examination. (Incorrect) A number of girls have passed in the examination.( correct) 29. When ‘as well as’, ‘along with’, together with’, ‘no less than’, ‘in addition to’ and ‘not’ and ‘with’ join two subjects, the verb will be used according to the first subject. Ram, as well as his five friends, are going.( Incorrect) Ram, as well as his five friends, is going. (Correct) The teacher, along with the students, were dancing.( Incorrect) The teacher, along with the students, was going. (Correct) 113 | P a g e

Common Error Detection Practice Set-1 Read these sentences carefully and try to understand the correct uses. Incorrect: My father gave me some advices. Correct: My father gave me some ADVICE. Advice is mainly used with the first meaning, and in this meaning it is a mass noun (that is, it has no plural). Incorrect: Mahi has grey hairs. Correct: Mahi has grey HAIR. The noun hair is a singular, plural, or collective noun. Incorrect: I asked the authority if they had any informations about the arrival of the flight. Correct: I asked the authority if they had any INFORMATION about the arrival of the flight. Information is a non-countable noun (you can't have 4 informations), so it is neither singular nor plural. The correct usage is "information" without the 's'. Incorrect: People should be true to their word. Correct: People should be true to their WORDS. Incorrect: Furnitures are now-a-days made of steel. Correct: FURNITURE IS now-a-days made of steel. Incorrect: I asked them to leave their luggages at the hotel room. Correct: I asked them to leave their LUGGAGE at the hotel room. Incorrect: The building is out of repairs. Correct: The building is out of REPAIR. Incorrect: The farmer took out the cattles for grazing. Correct: The farmer took out the CATTLE for grazing. Incorrect: I bought three dozens notebooks. Correct: I bought three DOZEN notebooks. Incorrect: Sachin has completed three-fourth of his homework till yet. Correct: Sachin has completed THREE-FOURTHS of his homework till yet.

114 | P a g e

Common Error Detection Practice Set-2 Here, we have brought to you another set of correct and incorrect uses of Grammar. Read it carefully and try to understand why one is incorrect. So, you don't make such silly mistakes during your exam. Incorrect: The price of the mobile phone is twenty thousands rupees. Correct: The price of the mobile phone is twenty THOUSAND rupees. Incorrect:Shruti cooked fishes for lunch. Correct: Shruti cooked FISH for lunch. Incorrect: He owns twelve sheeps and five deers. Correct:He owns twelve SHEEP and five DEER. Incorrect: My mother gave me a ten-rupees note for my pocket money. Correct: My mother gave me a TEN-RUPEE note for my pocket money. Incorrect: The table’s legs are broken. Correct: The LEGS OF THE TABLE are broken. Incorrect: I went to the party with my family members. Correct: I went to the party with THE MEMBERS OF MY FAMILY. Incorrect: He has six brother-in-laws. Correct: He has six BROTHERS-IN-LAW. Incorrect: This is Amol’s my friend’s book. Correct: This is AMOL my friemd’s book. Incorrect: I have many works to do today. Correct: I have MUCH WORK to do today. Incorrect: Politics are very popular these days. Correct: Politics IS very popular these days.

115 | P a g e

Common Error Detection Practice Set-3 Here, we have brought to you another set of correct and incorrect uses of Grammar. Read it carefully and try to understand why one is incorrect. So, you don't make such silly mistakes during your exam. Incorrect : All the money are kept in the locker. Correct : All the money IS kept in the locker. Incorrect :I kept my spectacle on the table. Correct I : kept my SPECTACLES on the table. Incorrect : The scissor should be kept out of reach of children. Correct : The SCISSORS should be kept out of reach of children. Incorrect : The Punjabis are brave peoples. Correct : The Punjabis are A BRAVE PEOPLE. Incorrect : The people of Europe have seen two World Wars. Correct : The peoples of Europe have seen two World Wars. Incorrect : Rita prefers wearing light-coloured cloth. Correct : Rita prefers wearing light-coloured CLOTHES. Incorrect : The Park Street is closed for repair. Correct : The Park Street is closed for REPAIRS. Incorrect : He does not like these kinds of movies. Correct : He does not like MOVIES OF THIS KIND. Incorrect : The Planning Commission are at work. Correct : The Planning Commission IS at work. Incorrect : Those news are good. Correct : THAT NEWS IS good.

116 | P a g e

Common Error Detection: Section 1 1. Ram was / senior to / Sam in college. A. Ram was B. senior to C. Sam in college D. No Error Answer: Option D 2. It was him / who came / running / into the classroom. A. It was him B. who came C. running D. into the classroom E. No Error Answer: Option A 3. The capital of Yemen / is situating / 2190 meters above / the sea level. A. The capital of Yemen B. is situating C. 2190 meters above D. the sea level E. No Error Answer: Option B 4. The captain along with his team / are practicing very hard / for the / forth coming matches. A. The captain along with his team B. are practicing very hard C. for the D. forth coming matches E. No Error Answer: Option B 117 | P a g e

5. Supposing if / it rains / what shall / we do? A. Supposing if B. it rains C. what shall D. we do? E. No Error Answer: Option A

9. I and my colleague / have carefully considered the important issues raised by / the report which you sent me and / we have decided to take the following action. A. I and my colleague B. have carefully considered the important issues rai 6. Either Nemmu or Mennu / are in the wrong; C. the report which you sent me and / both can / certainly never be. D. we have decided to take A. Either Nemmu or the following Mennu E. No Error B. are in the wrong; Answer: Option A C. both can D. certainly never be 10. Even if / the teacher has E. No Error completed the / course the Answer: Option B students cannot pass / unless 7. None of two girls / who they study themself. were present / appeared to A. Even if B. the teacher has completed be inclined / to listen to the sane advice. A. None of two girls C. course the students B. who were present cannot pass C. appeared to be inclined D. unless they study themself D. to listen to sane advice E. No Error E. No Error Answer: Option D Answer: Option A 8. Everyone knows / that the leopards is / faster / of all animals. A. Everyone knows B. that the leopards is C. faster D. of all animals E. No Error Answer: Option D

11. Ramu is the elected leader / and also a person / who we all / can confide in. A. Ramu is the elected leader B. and also a person C. who we all D. can confide in E. No Error Answer: Option E

12. The gambling stall / was a grand success / because almost everyone / tried their luck. A. The gambling stall B. was a grand success C. because almost everyone D. tired their luck E. No Error Answer: Option D

16. If you saw / the amount of Samosas / he consumed at breakfast this morning,/ you would understand why he is so over-weight. A. If you saw B. the amount of Samosas C. he consumed at breakfast this morning, D. you would understand why he is so over-weight 13. Little water / that was left E. No Error / in the bottle was not Answer: Option B enough / to quench the thirst of two persons. 17. As our room was / A. Little water upstairs, so we/ didn't hear B. that was left him. C. in the bottle was not A. As our room was enough B. upstairs, so we D. to quench the thirst of C. didn't hear him two persons D. No error E. No Error Answer: Option B Answer: Option E 18. There are / no 14. The father / as well as conducted tours / for tourist the sons were / mysteriously to visit all the / important missing / from the house. places. A. The father A. There are B. as well as the sons were B. no conducted tours C. mysteriously missing C. for tourist to visit all the D. from the house D. important places E. No Error E. No Error Answer: Option B Answer: Option C 15. Every man, woman and child / is now aware / of the terrible consequences of / habit of smoking. A. Every man, woman and child B. is now aware C. of the terrible consequences of D. habit of smoking E. No Error Answer: Option D 118 | P a g e

19. On Friday evening / Mamta and me / took a coach / for Patna and arrived at Amit's house. A. On Friday evening B. Mamta and me C. took a coach D. for Patna and arrived at Amit's house E. No Error Answer: Option B

20. This girl / wrote an essay / so well that / her teacher was exceedingly pleased with her. A. This girl B. wrote an essay C. so well that D. her teacher was exceedingly pleased with her E. No Error Answer: Option B 21. Everyone of the students / has bought / his / identity card. A. Everyone of the students B. has bought C. his D. identity card E. No Error Answer: Option A 22. The doctor said / that patient / should not have wasted / before coming for help. A. The doctor said B. that patient C. should not have wasted D. before coming for help E. No Error Answer: Option B 23. In my youth / I used to / visit / Sharma's and Chawla's company. A. In my youth B. I used to C. visit D. Sharma's and Chawla's company E. No Error Answer: Option D

24. As he stood / below the shower / the sharp jets of / ice cold water revived his spirit. A. As he stood B. below the shower C. the sharp jets of D. ice cold water revived his spirit E. No Error Answer: Option D 25. Many a person / have lost / their life / and all property in the riots. A. Many a person B. have lost C. their life D. and all property in the riots E. No Error Answer: Option B 26. I have / two sister-inlaws / whose husbands / are all well settled. A. I have B. two sister-in-laws C. whose husbands D. are all well settled E. No Error Answer: Option B

28. Cattles / were grazing / in the meadows / near our farm. A. Cattles B. were grazing C. in the meadows D. near our farm. E. No Error Answer: Option A 29. Many a man / have / been working / under me. A. Many a man B. have C. been working D. under me E. No Error Answer: Option B 30. The cricket / is a / game of change. A. The cricket B. is a C. game of change D. No Error Answer: Option C

31. Mr Alexgander / has purchased / new furnitures from Metro-Mall. A. Mr Alexgander B. has purchased C. new furnitures from 27. I told him / the story / Metro-Mall in details / to make him D. No Error understand it fully. Answer: Option C A. I told him B. the story 32. Last year / many people C. in details / died from cholera D. to make him epidemic. understand it fully. A. Last year E. No Error B. many people Answer: Option C C. died from cholera epidemic D. No Error Answer: Option D 119 | P a g e

33. Both the / girls helped / one another. A. Both the B. girls helped C. one another D. No Error Answer: Option C 34. Vijay is / very much / sorry about her misconduct. A. Vijay is B. very much C. sorry about her misconduct D. No Error Answer: Option B 35. She treated/ me as though / an old friend. A. She treated B. me as though C. an old friend D. No error Answer: Option B 36. I'll go with you / as long as/ I won't have to sing. A. I'll go with you B. as long as C. I won't have to sing D. No error Answer: Option C 37. She'll pay you / as soon as she / receive the parcel. A. She'll pay you B. as soon as she C. receive the parcel D. No error Answer: Option C

38. I don't have a car / and I don't want / one as well. A. I don't have a car B. and I don't want C. one as well D. No error Answer: Option C

42. Our history can be seen / as a long evolutionary dialectical development / in which there have been / a painstaking forgiving of rational and moral self. A. Our history can be seen B. as a long evolutionary 39. Indian scientists are no dialectical development way / inferior than / any C. in which there have other scientists in the been world. D. a painstaking forgiving A. Indian scientists are no of rational and moral self way E. No error B. inferior than Answer: Option C C. any other scientists in the world 43. Our knowledge of D. No error history does not come to / Answer: Option B Our help and some times we / even fail to remember 40. Belgium chocolate is / who invented America. considered / by many to A. Our knowledge of be finger / than any others history does not come to in the world. B. Our help and some A. Belgium chocolate is times we considered C. even fail to remember B. by many to be finger D. who invented America C. than any others in the E. No error world Answer: Option D D. No error Answer: Option C 44. The Committee has made / significant changes 41. My grandfather owns / in / the rules which / will fifty acre / of wet land. help to banks. A. My grandfather owns A. The Committee has B. fifty acre made C. of wet land B. significant change in D. No error C. the rules which Answer: Option B D. will help to banks E. No error Answer: Option D 120 | P a g e

45. This Project which is funded / by the united nations / has helped over four / lakh Indians overcome poverty. A. This Project which is funded B. by the united nations C. has helped over four D. lakh Indians overcome poverty E. No error Answer: Option E 46. The number of tourists / is expected to rise / from seven to ten percent / in the next ten years. A. The number of tourists B. is expected to rise C. from seven to ten percent D. in the next ten years E. No error Answer: Option C 47. I enquired / of him / whether he would / accompany me to Dharmsala. A. I enquired B. of him C. whether he would D. accompany me to Dharmsala E. No error Answer: Option E

48. Being that my car is getting / its annual check up. / I will not be / able to pick you up tomorrow morning. A. Being that my car is getting B. its annual check up. C. I will not be D. able to pick you up tomorrow morning E. No error Answer: Option A

51. The website, which does not / accept advertisement and is funded / entirely by donations, describes itself / as the fifth most popular website on the planet. A. The website, which does not B. accept advertisement and is funded C. entirely by donations, describe itself D. as the fifth most 49. We can't hardly believe popular website on the that the / situation is so planet serious as / to justify such E. No error precautions / as you have Answer: Option D taken. A. We can't hardly believe 52. As sharing crime that the statistics for / the year B. situation is so serious as 2011, the Commissioner C. to justify such admitted that / there had precautions been an undue delay in / D. as you have taken the setting up of an antiE. No error narcotics cell. Answer: Option A A. As sharing crime statistics for 50. The third season of / B. the year 2011, the the popular television Commissioner admitted show will ends / on a that grand note with / C. there had been an undue celebrities dancing and delay in having fun. D. the setting up of an A. The third season of anti-narcotics cell B. the popular television E. No error show will ends Answer: Option E C. on a grand note with D. celebrities dancing and having fun E. No error Answer: Option B 121 | P a g e

53. The Moon may be the best place / to look for aliens as their / footprints on their surface would / last far longer than radio signals. A. The Moon may be the best place B. to look for aliens as their C. footprints on their surface would D. last far longer than radio signals E. No error Answer: Option C 54. The judge advised the Government to / have metered auto-rickshaw across the state while / recounting his personal experience where an autorickshaw driver / made him to wait and also demanded Rs. 100. A. The judge advised the Government to B. have metered autorickshaw across the state while C. recounting his personal experience where an auto-r D. made him to wait and also demanded Rs. 100 E. No error Answer: Option D

55. The company aims / to nearly double / its revenues on the back / of a strongest product pipeline. A. The company aims B. to nearly double C. its revenues on the back D. of a strongest product pipeline E. No error Answer: Option D 56. The woman that had / kidnapped a child has now / been apprehended and is being / held in the city's jail. A. The woman that had B. kidnapped a child has now C. been apprehended and is being D. held in the city's jail E. No error Answer: Option D 57. Rose growers in / the city are waking up / to the benefits / of collective action. A. Rose growers in B. the city are waking up C. to the benefits D. of collective action E. No error Answer: Option C

122 | P a g e

58. The Minister will have / a tough task on his hand / where three different recommendations / for this year's rate reach his desk. A. The Minister will have B. a tough task on his hand C. where three different recommendations D. for this year's rate reach his desk E. No error Answer: Option B

61. If successful, the research could / pave the way towards / the prevention in untimely deaths / due to fatal illnesses. A. If successful, the research could B. pave the way towards C. the prevention in untimely deaths D. due to fatal illnesses E. No error Answer: Option D

59. The current economic scenario / could possibly undo / the growth that followed / the economic liberalization of 1991. A. The current economic scenario B. could possibly undo / the growth that followed C. the growth that followed D. the economic liberalization of 1991 E. No error Answer: Option E

62. The Ministry has directed banks / to do away with their / separate promotion policies, a move / strongly opposed by the officers' unions. A. The Ministry has directed banks B. to do away with their C. separate promotion policies, a move D. strongly opposed by the officers' unions E. No error Answer: Option D

60. In a first of its kind study, / a team of scientist have tried to / 'grow' new stem cells in / the ear that get damage with age. A. In a first of its kind study, B. a team of scientist have tried to C. 'grow' new stem cells in D. the ear that get damage with age E. No error Answer: Option B

63. After a complaint was filed, / police teams was given the photograph / of the accused from the CCTV footage / recorded at the hotel. A. After a complaint was filed, B. police teams was given the photograph C. of the accused from the CCTV footage D. recorded at the hotel E. No error Answer: Option B

64. Activists opposing the rail project said / that the eleven new flyovers to be built / would practically ring / the death knell for the city. A. Activists opposing the rail project said B. that the eleven new flyovers to be built C. would practically ring D. the death knell for the city E. No error Answer: Option E 65. It is common / knowledge that each / of us / loves our home. A. It is common B. knowledge that each C. of us D. loves our home E. No error Answer: Option D 66. The teacher repeatedly warned / the students not to make / noise but students / did not obey. A. The teacher repeatedly warned B. the students not to make C. noise but students D. did not obey. E. No error Answer: Option C

123 | P a g e

67. The education Council / is consisted of ten members / all of whom / are experienced men. A. The education Council B. is consisted of ten members C. all of whom D. are experienced men. E. No error Answer: Option B 68. The woman who / has been perpetually hesitating / which of the two books / she will read first will probably read none and will suffer. A. The woman who B. has been perpetually hesitating C. which of the two books D. she will read first will probably read none and wi E. No error Answer: Option D 69. Summons were / issued yesterday / by Sessions judge / against the suspected thief. A. Summons were B. issued yesterday C. by Sessions judge D. against the suspected thief. E. No error Answer: Option A

70. Teaching in this school / would certainly improve / if it had less boys / in all the high classes. A. Teaching in this school B. would certainly improve C. if it had less boys D. in all the high classes. E. No error Answer: Option C

Common Error Detection: Section 2 1. Because of the cricket tournament / organizational work hardly / I have had any / rest since one week. A. Because of the cricket tournament B. organizational work hardly C. have had any D. rest since one week. E. No error Answer: Option C 2. A property dealer was /shoot dead by four unidentified jacket clad men /while taking a morning walk /in a park. A. A property dealer was B. shoot dead by four unidentified jacket clad men C. while taking a morning walk D. in a park. Answer: Option B

124 | P a g e

3. The newer type of automatic machines/ wash/ the clothes faster. A. The newer type of automatic machines B. wash C. the clothes faster D. No error Answer: Option A

5. Each of the students in the computer class/ has to type/ their own research paper this semester. A. Each of the students in the computer class B. has to type C. their own research paper this semester D. No Error 4. By the time/ we got our Answer: Option D tickets and entered the cinema theater,/ the show 6. The fact of me/ being a was already begun. stranger/ does not excuse A. By the time his conduct. B. we got our tickets and A. The fact of me entered the cinema theater B. being a stranger C. the show was already C. does not excuse his begun conduct D. No Error D. No error Answer: Option C Answer: Option A

Spotting Errors: Section 1 1. Find out whether there is any grammatical error in below sentence. We discussed about the problem so thoroughly / on the eve of the examination / that I found it very easy to work it out. A. We discussed about the problem so thoroughly B. on the eve of the examination C. that I found it very easy to work it out. D. No error.

4. Find out whether there is any grammatical error in below sentence. The Indian radio / which was previously controlled by the British rulers / is free now from the narrow vested interests. A. The Indian radio B. which was previously controlled by the British rulers C. is free now from the narrow vested interests D. No error

Answer: Option A

Answer: Option C

2. Find out whether there is any grammatical error in below sentence. An Indian ship / laden with merchandise / got drowned in the Pacific Ocean. A. An Indian ship B. laden with merchandise C. got drowned in the Pacific Ocean D. No error

5. Find out whether there is any grammatical error in below sentence. If I had known / this yesterday / I will have helped him A. If I had known B. this yesterday C. I will have helped him D. No error

Answer: Option C

Answer: Option C

3. Find out whether there is any grammatical error in below sentence. I could not put up in a hotel / because the boarding and lodging charges / were exorbitant. A. I could not put up in a hotel B. because the boarding and lodging charges C. were exorbitant D. No error

6. Find out whether there is any grammatical error in below sentence. A lot of travel delay is caused / due to the inefficiency and lack of good management / on behalf of the railways. A. A lot of travel delay is caused B. due to the inefficiency and lack of good management C. on behalf of the railways D. No error

Answer: Option A

Answer: Option C

125 | P a g e

7. Find out whether there is any grammatical error in below sentence. One of the members / expressed doubt if / the Minister was an atheist. A. One of the members B. expressed doubt if C. the Minister was an atheist D. No error.

11. Find out whether there is any grammatical error in below sentence. According to the Bible / it is meek and humble / who shall inherit the earth. A. According to the Bible B. it is meek and humble C. who shall inherit the earth D. No error

Answer: Option B

Answer: Option B

8. Find out whether there is any grammatical error in below sentence. I have got / my M.Sc. degree / in 1988. A. I have got B. my M.Sc. degree C. in 1988 D. No error

12. Find out whether there is any grammatical error in below sentence. Do the roses in your garden smell / more sweetly / than those in ours? A. Do the roses in your garden smell B. more sweetly C. than those in ours D. No error

Answer: Option A 9. Find out whether there is any grammatical error in below sentence. Having received your letter / this morning, we are writing / to thank you for the same. A. Having received your letter B. this morning, we are writing C. to thank you for the same D. No error Answer: Option D 10. Find out whether there is any grammatical error in below sentence. If you lend him a book / he will lend it to some one else / and never you will get it back. A. If you lend him a book B. he will lend it to some one else C. and never you will get it back D. No error Answer: Option C

126 | P a g e

Answer: Option B 13. Find out whether there is any grammatical error in below sentence. Block of Residential flats / are coming up / near our house. A. Block of Residential flats B. are coming up C. near our house D. No error Answer: Option A 14. Find out whether there is any grammatical error in below sentence. You can get / all the information that you want / in this book. A. You can get B. all the information that you want C. in this book D. No error Answer: Option B

15. Find out whether there is any grammatical error in below sentence. The students were / awaiting for / the arrival of the chief guest. A. The students were B. awaiting for C. the arrival of the chief guest D. No error. Answer: Option B 16. Find out whether there is any grammatical error in below sentence. Sixty miles / are / a good distance. A. Sixty miles B. are C. a good distance D. No error. Answer: Option B 17. Find out whether there is any grammatical error in below sentence. They have been / very close friends / until they quarrelled. A. They have been B. very close friends C. until they quarrelled D. No error. Answer: Option A 18. Find out whether there is any grammatical error in below sentence. When the dentist came in / my tooth was stopped aching / out of fear that I might lose my tooth. A. When the dentist came in B. my tooth was stopped aching C. out of fear that I might lose my tooth D. No error. Answer: Option B 127 | P a g e

19. Find out whether there is any grammatical error in below sentence. It is the duty of every citizen to do his utmost / to defend the hardly-won / freedom of the country. A. It is the duty of every citizen to do his utmost B. to defend the hardly-won C. freedom of the country D. No error. Answer: Option B 20. Find out whether there is any grammatical error in below sentence. No sooner did I open the door / when the rain, heavy and stormy, rushed in / making us shiver from head to foot. A. No sooner did I open the door B. when the rain, heavy and stormy, rushed in C. making us shiver from head to foot D. No error. Answer: Option B 21. Find out whether there is any grammatical error in below sentence. If a man diligently seeks to come into the contact / with the best that has been thought and said in this world / he will become simple and unselfish. A. If a man diligently seeks to come into the contact B. with the best that has been thought and said in this world C. he will become simple and unselfish D. No error. Answer: Option A

22. Find out whether there is any grammatical error in below sentence. You must / remember me / to post this letter. A. You must B. remember me C. to post this letter D. No error. Answer: Option B

26. Find out whether there is any grammatical error in below sentence. He couldn't but help / shedding tears at the plight of the villagers / rendered homeless by a devastating cyclone. A. He couldn't but help B. shedding tears at the plight of the villagers C. rendered homeless by a devastating cyclone D. No error. Answer: Option A

23. Find out whether there is any grammatical error in below sentence. I shall certainly / write you / when I shall reach NewDelhi. A. I shall certainly B. write you C. when I shall reach NewDelhi D. No error. Answer: Option C 24. Find out whether there is any grammatical error in below sentence. On the busy Ring Road / we witnessed a collusion / between a truck and an auto. A. On the busy Ring Road B. we witnessed a collusion C. between a truck and an auto D. No error. Answer: Option B 25. Find out whether there is any grammatical error in below sentence. Mr.Praful Patel / is not attending his office / for the last one month. A. Mr.Praful Patel B. is not attending his office C. for the last one month D. No error. Answer: Option B 128 | P a g e

27. Find out whether there is any grammatical error in below sentence. He will certainly help you / if you will ask him / in a pleasant manner. A. He will certainly help you B. if you will ask him C. in a pleasant manner D. No error. Answer: Option B 28. Find out whether there is any grammatical error in below sentence. The brand propositon now therefore had to be that Keokarpin Antiseptic Cream is more effective / because it penetrates deepdown (beinglight and non-sticky) and works from within / (because of its ayurvedic ingredients) to keep skin blemish, free and helps cope with cuts nicks, burns and nappy rash. A. The brand propositon now therefore had to be that Keokarpin Antiseptic Cream is more effective B. because it penetrates deepdown (beinglight and non-sticky) and works from within C. (because of its ayurvedic ingredients) to keep skin blemish, free and helps cope with cuts nicks, burns and nappy rash D. No error. Answer: Option A

29. Find out whether there is any grammatical error in below sentence. Will you please buy / some jaggery for me / if you go to the market? A. Will you please buy B. some jaggery for me C. if you go to the market? D. No error.

32. Find out whether there is any grammatical error in below sentence. He persisted / to do it / in spite of my advice. A. He persisted B. to do it C. in spite of my advice D. No error.

Answer: Option D

Answer: Option B

30. Find out whether there is any grammatical error in below sentence. Most of the members at the meeting felt / that the group appointed for investigating the case / were not competent to do the job efficiently. A. Most of the members at the meeting felt B. that the group appointed for investigating the case C. were not competent to do the job efficiently D. No error.

33. Find out whether there is any grammatical error in below sentence. The long-awaited moment at last came, / and we set out for the station / as merry a band of children as I have ever seen before or since. A. The long-awaited moment at last came, B. and we set out for the station C. as merry a band of children as I have ever seen before or since D. No error. Answer: Option C

Answer: Option C 31. Find out whether there is any grammatical error in below sentence. In these days of inflation / a ten rupee's note will not buy you / even an ordinary meal. A. In these days of inflation B. a ten rupee's note will not buy you C. even an ordinary meal D. No error.

34. Find out whether there is any grammatical error in below sentence. Our conception of / what should a science of mental life be / has changed considerably since James' time. A. Our conception of B. what should a science of mental life be C. has changed considerably since James' time D. No error.

Answer: Option B

Answer: Option B

129 | P a g e

35. Find out whether there is any grammatical error in below sentence. He is not coming tomorrow / as he is having a pain in the chest / and has to see a doctor. A. He is not coming tomorrow B. as he is having a pain in the chest C. and has to see a doctor D. No error.

38. Find out whether there is any grammatical error in below sentence. Azharuddin is one of the finest batsmen / that India have produced / over the decades. A. Azharuddin is one of the finest batsmen B. that India have produced C. over the decades D. No error.

Answer: Option C

Answer: Option B

36. Find out whether there is any grammatical error in below sentence. Many times the news has been published / in the papers that the end of the world will be certain / if a nuclear war breaks out. A. Many times the news has been published B. in the papers that the end of the world will be certain C. if a nuclear war breaks out D. No error.

39. Find out whether there is any grammatical error in below sentence. The thief broke in the / house at the / dead of night A. The thief broke in the B. house at the C. dead of night D. No error.

Answer: Option D 37. Find out whether there is any grammatical error in below sentence. The reason Ram / is absent from his duty / is because he is unwell. A. The reason Ram B. is absent from his duty C. is because he is unwell D. No error. Answer: Option A

Answer: Option A 40. Find out whether there is any grammatical error in below sentence. May I / know who you want / to see please A. May I B. know who you want C. to see please D. No error. Answer: Option B 41. Find out whether there is any grammatical error in below sentence. He said that he / will mind if / I refused his offer. A. He said that he B. will mind if C. I refused his offer. D. No error. Answer: Option B

130 | P a g e

42. Find out whether there is any grammatical error in below sentence. Arun's parents died when he was young and / he looked after his aunt / who had no children. A. Arun's parents died when he was young and B. he looked after his aunt C. who had no children D. No error. Answer: Option B 43. Find out whether there is any grammatical error in below sentence. Though child marriage / has been banned / the custom still prevailed among some groups in India. A. Though child marriage B. has been banned C. the custom still prevailed among some groups in India D. No error. Answer: Option C

46. Find out whether there is any grammatical error in below sentence. In spite of several reminders, / he did not so far send / any reply to me, letters. A. In spite of several reminders, B. he did not so far send C. any reply to me, letters. D. No error. Answer: Option B 47. Find out whether there is any grammatical error in below sentence. As much as I admire him for his sterling qualities / I cannot excuse him for / being unfair to his friends. A. As much as I admire him for his sterling qualities B. I cannot excuse him for C. being unfair to his friends. D. No error. Answer: Option A

44. Find out whether there is any grammatical error in below sentence. My papa is / in bad mood / today / A. My papa is B. in bad mood C. today D. No error.

48. Find out whether there is any grammatical error in below sentence. Please try to understand / that the dispute on this issue is between my Answer: Option B brother and myself, / and concerns nobody else. A. Please try to understand 45. Find out whether there is any B. that the dispute on this issue is grammatical error in below sentence. The warden / forbade the student / from between my brother and myself, C. and concerns nobody else leaving the hostel. D. No error. A. The warden B. forbade the student C. from leaving the hostel Answer: Option B D. No error. Answer: Option D 131 | P a g e

49. Find out whether there is any grammatical error in below sentence. All the furnitures have been / sent to the new house / located in a village. A. All the furnitures have been B. sent to the new house C. located in a village D. No error. Answer: Option A 50. Find out whether there is any grammatical error in below sentence. It does not matter how you do it; / what I want is that / you should finish the work within a month. A. It does not matter how you do it; B. what I want is that C. you should finish the work within a month D. No error.

53. Find out whether there is any grammatical error in below sentence. Everyone visiting the house asked the young girl / how could she kill the wolf / single handed and without a weapon. A. Everyone visiting the house asked the young girl B. how could she kill the wolf C. single handed and without a weapon D. No error. Answer: Option B 54. Find out whether there is any grammatical error in below sentence. Many health-conscious people / prefer margarine / than butter. A. Many health-conscious people B. prefer margarine C. than butter D. No error.

Answer: Option B

Answer: Option C

51. Find out whether there is any grammatical error in below sentence. Though senior in age, / his father is junior than / my father in service. A. Though senior in age, B. his father is junior than C. my father in service D. No error.

55. Find out whether there is any grammatical error in below sentence. The retiring principal asked his old pupils / to take the interest in the school / after he has retired. A. The retiring principal asked his old pupils B. to take the interest in the school C. after he has retired D. No error.

Answer: Option B 52. Find out whether there is any grammatical error in below sentence. While walking slowly in the park / on a quiet summer afternoon / a mad dog suddenly attacked him from behind A. While walking slowly in the park B. on a quiet summer afternoon C. a mad dog suddenly attacked him from behind D. No error. Answer: Option A 132 | P a g e

Answer: Option B 56. Find out whether there is any grammatical error in below sentence. At present juncture / however, the supercomputer / would be a costly toy. A. At present juncture B. however, the supercomputer C. would be a costly toy D. No error. Answer: Option A

57. Find out whether there is any grammatical error in below sentence. The crews were on board, / and they soon busied themselves / in preparing to meet the storm. A. The crews were on board, B. and they soon busied themselves C. in preparing to meet the storm D. No error. Answer: Option C 58. Find out whether there is any grammatical error in below sentence. Troy was taken by Greeks / this formed the basis of a story / which has become famous. A. Troy was taken by Greeks B. this formed the basis of a story C. which has become famous D. No error. Answer: Option A 59. Find out whether there is any grammatical error in below sentence. I am much pleased / to know that / you have topped the list. A. I am much pleased B. to know that C. you have topped the list D. No error. Answer: Option A 60. Find out whether there is any grammatical error in below sentence. He has not been attending / English classes / since one month A. He has not been attending B. English classes C. since one month D. No error. Answer: Option C 133 | P a g e

61. Find out whether there is any grammatical error in below sentence. It is time / we should accept all our people as equals / and as partners in the task of building a strong and united nation. A. It is time B. we should accept all our people as equals C. and as partners in the task of building a strong and united nation D. No error. Answer: Option B 62. Find out whether there is any grammatical error in below sentence. Twice twelve / makes / twenty-four A. Twice twelve B. makes C. twenty-four D. No error. Answer: Option B 63. Find out whether there is any grammatical error in below sentence. Regretfully, profits earned by your company / fell by 20 per cent last year / despite higher sales. A. Regretfully, profits earned by your company B. fell by 20 per cent last year C. despite higher sales D. No error. Answer: Option A

64. Find out whether there is any grammatical error in below sentence. In a report issued by Indian Statistical Institute, / the Iron and Steel Industry is investing more than any other / Indian industry in fighting pollution. A. In a report issued by Indian Statistical Institute, B. the Iron and Steel Industry is investing more than any other C. Indian industry in fighting pollution D. No error. Answer: Option C 65. Find out whether there is any grammatical error in below sentence. He is going everyday / for a morning walk / with his friends and neighbours A. He is going everyday B. for a morning walk C. with his friends and neighbours D. No error. Answer: Option A 66. Find out whether there is any grammatical error in below sentence. My father goes / to the office / five day week. A. My father goes B. to the office C. five day week D. No error.

68. Find out whether there is any grammatical error in below sentence. If I were him, / I would have taught / those cheats a lesson. A. If I were him, B. I would have taught C. those cheats a lesson D. No error. Answer: Option A 69. Find out whether there is any grammatical error in below sentence. Looking back, I find that among the many impressions ofthe people of India, / absorbed while I lived among them, / are their reverence for great men and women. A. Looking back, I find that among the many impressions ofthe people of India, B. absorbed while I lived among them, C. are their reverence for great men and women D. No error. Answer: Option A

70. Find out whether there is any Answer: Option C grammatical error in below sentence. If you work hard, / you will get good grades / in examinations 67. Find out whether there is any A. If you work hard, grammatical error in below sentence. If she will be promoted / she will get / a B. you will get good grades C. in examinations. higher salary. D. No error A. If she will be promoted B. she will get C. A higher salary Answer: Option C D. No error. Answer: Option A 134 | P a g e

Spotting Errors: Section 2 1. Find out whether there is any grammatical error in below sentence. He managed to make sense of the book / even though it was the first time / he read anything on the subject. A. He managed to make sense of the book B. even though it was the first time C. he read anything on the subject D. No error.

4. Find out whether there is any grammatical error in below sentence. I have / an appointment / on the 9th September on five o'clock A. I have B. an appointment C. on the 9th September on five o'clock D. No error. Answer: Option C

Answer: Option C 2. Find out whether there is any grammatical error in below sentence. She reluctantly said that / if nobody else was doing it / she will do it. A. She reluctantly said that B. if nobody else was doing it C. she will do it D. No error. Answer: Option C 3. Find out whether there is any grammatical error in below sentence. I fail to understand / why he replied in negative / when the proposal was in his favour. A. I fail to understand B. why he replied in negative C. when the proposal was in his favour D. No error.

5. Find out whether there is any grammatical error in below sentence. They / enjoyed thoroughly / at the party. A. They B. enjoyed thoroughly C. at the party D. No error. Answer: Option B 6. Find out whether there is any grammatical error in below sentence. The method suggested in the lecture / enables a student to learn more quickly / and to have remembered for a longer period of time. A. The method suggested in the lecture B. enables a student to learn more quickly C. and to have remembered for a longer period of time D. No error. Answer: Option C

Answer: Option A

135 | P a g e

7. Find out whether there is any grammatical error in below sentence. My friend asked me / if I can lend him my Parker pen / for a few days. A. My friend asked me B. if I can lend him my Parker pen C. for a few days D. No error. Answer: Option B 8. Find out whether there is any grammatical error in below sentence. The test will not need / more than one and half hour / to finish. A. The test will not need B. more than one and half hour C. to finish D. No error. Answer: Option B 9. Find out whether there is any grammatical error in below sentence. The school is / within hundred yards / from the church. A. The school is B. within hundred yards C. from the church D. No error. Answer: Option B 10. Find out whether there is any grammatical error in below sentence. His father died of cholera / but his mother also, / though very weak, is out of danger. A. His father died of cholera B. but his mother also, C. though very weak, is out of danger D. No error. Answer: Option B

136 | P a g e

11. Find out whether there is any grammatical error in below sentence. The police broke upon the robbers / when they were in the lonely place / to divide their booty. A. The police broke upon the robbers B. when they were in the lonely place C. to divide their booty D. No error. Answer: Option A 12. Find out whether there is any grammatical error in below sentence. Since it was his first election campaign, the candidate was confused; / none could clearly understand / either the principles he stood for or the benefits he promised. A. Since it was his first election campaign, the candidate was confused; B. none could clearly understand C. either the principles he stood for or the benefits he promised D. No error. Answer: Option D 13. Find out whether there is any grammatical error in below sentence. Jayesh is getting / fatter because he / does not take exercise at all A. Jayesh is getting B. fatter because he C. does not take exercise at all D. No error. Answer: Option B

14. Find out whether there is any grammatical error in below sentence. He loved / none but / his neighbour's daughter. A. He loved B. none but C. his neighbour's daughter D. No error.

18. Find out whether there is any grammatical error in below sentence. Neeraj said / that he would rather fail than copying / in the examination A. Neeraj said B. that he would rather fail than copying C. in the examination D. No error.

Answer: Option D

Answer: Option B

15. Find out whether there is any grammatical error in below sentence. The criminal was / caught, convicted the hung / in a short period of time. A. The criminal was B. caught, convicted the hung C. in a short period of time D. No error. Answer: Option C

19. Find out whether there is any grammatical error in below sentence. I had hoped to have met him yesterday / to discuss the matter with him / but he was not in his house, and so I could not meet him. A. I had hoped to have met him yesterday B. to discuss the matter with him C. but he was not in his house, and so I could not meet him D. No error.

16. Find out whether there is any grammatical error in below sentence. I am thinking of / to go to Agra / for my cousin's marriage. Answer: Option A A. I am thinking of B. to go to Agra 20. Find out whether there is any C. for my cousin's marriage grammatical error in below sentence. D. No error. You will come / to my party tomorrow, / isn't it? Answer: Option B A. You will come B. to my party tomorrow, 17. Find out whether there is any C. isn't it? grammatical error in below sentence. In management, as you rise higher, / the D. No error. problems you face becomemore and Answer: Option C more unstructured and you can't just fall back on / the tools you had been 21. Find out whether there is any A. In management, as you rise higher, grammatical error in below sentence. B. the problems you face becomemore and more unstructured and you can't just I wonder / how am I / to do it A. I wonder B. how am I fall back on C. to do it D. No error. C. the tools you had been D. No error. Answer: Option B Answer: Option B 137 | P a g e

22. Find out whether there is any grammatical error in below sentence. He is not to blame / for what has happened / for he is in no way connected with it. A. He is not to blame B. for what has happened C. for he is in no way connected with it D. No error. Answer: Option A 23. Find out whether there is any grammatical error in below sentence. Had you not / reached in time / he would have lost allour belongings. A. Had you not B. reached in time C. he would have lost allour belongings D. No error. Answer: Option C 24. Find out whether there is any grammatical error in below sentence. The man told to her / that he had not brought his dog / out for a walk as he was afraid that it would rain. A. The man told to her B. that he had not brought his dog C. out for a walk as he was afraid that it would rain D. No error. Answer: Option A 25. Find out whether there is any grammatical error in below sentence. If I am you / I would have seen to it / that I won the prize. A. If I am you B. I would have seen to it C. that I won the prize D. No error. Answer: Option A 138 | P a g e

26. Find out whether there is any grammatical error in below sentence. It is unfortunate that / many youngsters get / addicted to gamble. A. It is unfortunate that B. many youngsters get C. addicted to gamble D. No error. Answer: Option C 27. Find out whether there is any grammatical error in below sentence. Kamala's fountain-pen / is as expensive / as Shyama. A. Kamala's fountain-pen B. is as expensive C. as Shyama D. No error. Answer: Option C 28. Find out whether there is any grammatical error in below sentence. When we consider all the factors, which are many, / the number of school dropouts / are quite disturbing. A. When we consider all the factors, which are many, B. the number of school dropouts C. are quite disturbing D. No error. Answer: Option C 29. Find out whether there is any grammatical error in below sentence. She has never / approve of him / working as a clerk. A. She has never B. approve of him C. working as a clerk D. No error. Answer: Option B

30. Find out whether there is any grammatical error in below sentence. At the station, / I'll hire a coolie / to carry my baggages A. At the station, B. I'll hire a coolie C. to carry my baggages D. No error. Answer: Option C 31. Find out whether there is any grammatical error in below sentence. The number of marks carried by each question / are indicated / at the end of the question A. The number of marks carried by each question B. are indicated C. at the end of the question D. No error. Answer: Option B 32. Find out whether there is any grammatical error in below sentence. There is no question / of my failing / in the examination. A. There is no question B. of my failing C. in the examination D. No error. Answer: Option B 33. Find out whether there is any grammatical error in below sentence. She is / no longer popular as she has / a friends A. She is B. no longer popular as she has C. a friends D. No error. Answer: Option C 139 | P a g e

34. Find out whether there is any grammatical error in below sentence. It is necessary / that everybody / must have a house. A. It is necessary B. that everybody C. must have a house D. No error. Answer: Option C 35. Find out whether there is any grammatical error in below sentence. Students should not take part / in party politics and political demonstrations / as they interfere in serious study A. Students should not take part B. in party politics and political demonstrations C. as they interfere in serious study D. No error. Answer: Option C 36. Find out whether there is any grammatical error in below sentence. To facilitate exports and improve sales in the domestic market / some of the improvised fabrics and garments fabricated out from them / are displayed in the main pavilion A. To facilitate exports and improve sales in the domestic market B. some of the improvised fabrics and garments fabricated out from them C. are displayed in the main pavilion. D. No error. Answer: Option B 37. Find out whether there is any grammatical error in below sentence. Both of you two / can come with me / to the play tonight A. Both of you two B. can come with me C. to the play tonight D. No error. Answer: Option A

38. Find out whether there is any grammatical error in below sentence. No sooner did the sun rise / when we took a hasty breakfast / and resumed the journey. A. No sooner did the sun rise B. when we took a hasty breakfast C. and resumed the journey D. No error. Answer: Option B 39. Find out whether there is any grammatical error in below sentence. The charges in this hospital / are less than / the hospital near my house. A. The charges in this hospital B. are less than C. the hospital near my house D. No error Answer: Option C 40. Find out whether there is any grammatical error in below sentence. The brakes and steering failed / and the bus ran down the hill / without anyone being able control it. A. The brakes and steering failed B. and the bus ran down the hill C. without anyone being able control it D. No error. Answer: Option C 41. Find out whether there is any grammatical error in below sentence. The tall three girls / had left / the day before. A. The tall three girls B. had left C. the day before. D. No error. Answer: Option A

140 | P a g e

42. Find out whether there is any grammatical error in below sentence. When he was asked what is wrong with him. / he said that he was not well. / and asked for leave of absence for one day. A. When he was asked what is wrong with him B. he said that he was not well C. and asked for leave of absence for one day D. No error. Answer: Option A 43. Find out whether there is any grammatical error in below sentence. Wherever they go / Indians easily adapt to / local circumstances. A. Wherever they go B. Indians easily adapt to C. local circumstances D. No error. Answer: Option B 44. Find out whether there is any grammatical error in below sentence. Remember that you are part of / the team and your success depends on the support / you are able to give and get from your other team members. A. Remember that you are part of B. the team and your success depends on the support C. you are able to give and get from your other team members D. No error. Answer: Option A

45. Find out whether there is any grammatical error in below sentence. It is an established fact that the transcendental American poets and philosophers. / who lived in the latter half of the nineteenth century. / were more influenced by Indian philosophy, in particular by Upanishadic Philosophy. A. It is an established fact that the transcendental American poets and philosophers B. who lived in the latter half of the nineteenth century C. were more influenced by Indian philosophy, in particular by Upanishadic Philosophy D. No error. Answer: Option C 46. Find out whether there is any grammatical error in below sentence. That house / is costing me / ten thousand rupees A. That house B. is costing me C. ten thousand rupees D. No error. Answer: Option B

48. Find out whether there is any grammatical error in below sentence. It is true / that God helps those / who helps themselves. A. It is true B. that God helps those C. who helps themselves D. No error. Answer: Option C 49. Find out whether there is any grammatical error in below sentence. Happily, zoos were / unwilling to cooperate / in a scheme that was potentially harmful to animal welfare A. Happily, zoos were B. unwilling to cooperate C. in a scheme that was potentially harmful to animal welfare D. No error. Answer: Option C 50. Find out whether there is any grammatical error in below sentence. Neither he / nor his father is interested / in joining the party. A. Neither he B. nor his father is interested C. in joining the party. D. No error.

47. Find out whether there is any grammatical error in below sentence. Firstly you should / think over the meaning of the words / and then use them. A. Firstly you should B. think over the meaning of the words C. and then use them D. No error.

Answer: Option D

Answer: Option A

Answer: Option A

141 | P a g e

51. Find out whether there is any grammatical error in below sentence. With little patience / you will be able to / cross this hurdle A. With little patience B. you will be able to C. cross this hurdle D. No error.

52. Find out whether there is any grammatical error in below sentence. She was told / to give the award to whosoever / she thought has done the most for the downtrodden. A. She was told B. to give the award to whosoever C. she thought has done the most for the downtrodden. D. No error.

55. Find out whether there is any grammatical error in below sentence. Since we are friends / there should be no secret / between you and I. A. Since we are friends B. there should be no secret C. between you and I. D. No error. Answer: Option C

Answer: Option C 53. Find out whether there is any grammatical error in below sentence. At the end of the year / every student who had done adequate work / was automatically promoted. A. At the end of the year B. every student who had done adequate work C. was automatically promoted. D. No error.

56. Find out whether there is any grammatical error in below sentence. Since the attachment of airconditioned sleeping cars to all important trains, / travelling became very pleasant, / especially during the summer season. A. Since the attachment of airconditioned sleeping cars to all important trains, B. travelling became very pleasant, C. especially during the summer season. D. No error. Answer: Option B

Answer: Option D 54. Find out whether there is any grammatical error in below sentence. The reason why / he was rejected / was because he was too young. A. The reason why B. he was rejected C. was because he was too young. D. No error. Answer: Option C

142 | P a g e

57. Find out whether there is any grammatical error in below sentence. If I will have the time / I shall try and make it / to the zoo this afternoon. A. If I will have the time B. I shall try and make it C. to the zoo this afternoon. D. No error. Answer: Option A

58. Find out whether there is any grammatical error in below sentence. We are four brothers and sisters living in this house / but neither of us is / satisfied with it. A. We are four brothers and sisters living in this house B. but neither of us is C. satisfied with it. D. No error. Answer: Option B 59. Find out whether there is any grammatical error in below sentence. A leading textile manufacturer, one of the fastest growing in the industry. / is looking for a marketing manager / to look up the marketing network of the company A. A leading textile manufacturer, one of the fastest growing in the industry. B. is looking for a marketing manager C. to look up the marketing network of the company D. No error.

61. Find out whether there is any grammatical error in below sentence. The single biggest gainer in this process / was ITC's Gold Flake Kings sales are estimated / to have moved up from 50 million to 200 million sticks per month during 1987 and last year. A. The single biggest gainer in this process B. was ITC's Gold Flake Kings sales are estimated C. to have moved up from 50 million to 200 million sticks per month during 1987 and last year. D. No error. Answer: Option B 62. Find out whether there is any grammatical error in below sentence. They left / their luggages / at the railway station. A. They left B. their luggages C. at the railway station. D. No error.

Answer: Option C

Answer: Option B

60. Find out whether there is any grammatical error in below sentence. Not one of the hundreds / of striking workers. / were allowed to go near the factory. A. Not one of the hundreds B. of striking workers. C. were allowed to go near the factory. D. No error.

63. Find out whether there is any grammatical error in below sentence. Salim and Antony are such good friends / that one won't go to the pictures. / without his coming too. A. Salim and Antony are such good friends B. that one won't go to the pictures. C. without his coming too. D. No error.

Answer: Option C

Answer: Option C

143 | P a g e

64. Find out whether there is any grammatical error in below sentence. She is / five years / senior than me. A. She is B. five years C. senior than me. D. No error. Answer: Option C 65. Find out whether there is any grammatical error in below sentence. The President had hardly spoken / a few words / when the microphone stopped functioning. A. The President had hardly spoken B. a few words C. when the microphone stopped functioning. D. No error. Answer: Option D 66. Find out whether there is any grammatical error in below sentence. Locke's treatises on government toleration and education / show a mind fully awake in / the possibilities of social reconstruction. A. Locke's treatises on government toleration and education B. show a mind fully awake in C. the possibilities of social reconstruction. D. No error. Answer: Option B

68. Find out whether there is any grammatical error in below sentence. None of the students attending your class / answered your questions / did they? A. None of the students attending your class B. answered your questions C. did they? D. No error. Answer: Option C 69. Find out whether there is any grammatical error in below sentence. An animal / can be just as unhappy in a vast area / or in a small one A. An animal B. can be just as unhappy in a vast area C. or in a small one D. No error. Answer: Option C 70. Find out whether there is any grammatical error in below sentence. He is working in / a bank in New Delhi / for the past several months. A. He is working in B. a bank in New Delhi C. for the past several months. D. No error.

Answer: Option A 67. Find out whether there is any grammatical error in below sentence. You will get / all the information / if you read this booklet carefully. A. You will get B. all the information C. if you read this booklet carefully. D. No error. Answer: Option A 144 | P a g e

Spotting Errors: Section 3 1. Find out whether there is any grammatical error in below sentence. The scientist must follow / his hunches and his data / wherever it may lead. A. The scientist must follow B. his hunches and his data C. wherever it may lead. D. No error.

4. Find out whether there is any grammatical error in below sentence. Had I / known it earlier / I would contact you. A. Had I B. known it earlier C. I would contact you. D. No error. Answer: Option C

Answer: Option C 2. Find out whether there is any grammatical error in below sentence. Each one of the boys / have paid / the tuition-fee. A. Each one of the boys B. have paid C. the tuition-fee. D. No error. Answer: Option B 3. Find out whether there is any grammatical error in below sentence. A large scale exchange of nuclear weapons / will produce unprecedented amounts of radiation / that can penetrate into the biological tissue. A. A large scale exchange of nuclear weapons B. will produce unprecedented amounts of radiation C. that can penetrate into the biological tissue. D. No error. Answer: Option C 145 | P a g e

5. Find out whether there is any grammatical error in below sentence. He asked me / why did I call / him a rogue. A. He asked me B. why did I call C. him a rogue. D. No error. Answer: Option B 6. Find out whether there is any grammatical error in below sentence. Were you / given a choice / or you had to do it? A. Were you B. given a choice C. or you had to do it? D. No error. Answer: Option A 7. Find out whether there is any grammatical error in below sentence. The person which was / recommended for the position / did not fulfil the prescribed qualifications. A. The person which was B. recommended for the position C. did not fulfil the prescribed qualifications. D. No error. Answer: Option A

8. Find out whether there is any grammatical error in below sentence. What does Professor Dhavan / spend so many hours / in the laboratory? A. What does Professor Dhavan B. spend so many hours C. in the laboratory? D. No error. Answer: Option A 9. Find out whether there is any grammatical error in below sentence. The presumption that the average investor does not understand / or take interest in the affairs of the company / is not correct. A. The presumption that the average investor does not understand B. or take interest in the affairs of the company C. is not correct. D. No error.

12. Find out whether there is any grammatical error in below sentence. At the moment the house / was burgled the family / attended a night party in the neighbourhood. A. At the moment the house B. was burgled the family C. attended a night party in the neighbourhood. D. No error. Answer: Option A 13. Find out whether there is any grammatical error in below sentence. Scarcely had / I arrived than / the train left. A. Scarcely had B. I arrived than C. the train left. D. No error. Answer: Option B

14. Find out whether there is any grammatical error in below sentence. Answer: Option D Though he stoutly persisted in denying his involvement in the case, / the facts made it very clear / that he had hand in 10. Find out whether there is any the cruel murder of his wife. grammatical error in below sentence. Shanghai is / bigger than any city / of the A. Though he stoutly persisted in denying his involvement in the case, world A. Shanghai is B. bigger than any city B. the facts made it very clear C. that he had hand in the cruel murder C. of the world D. No error. of his wife. D. No error. Answer: Option B 11. Find out whether there is any grammatical error in below sentence. Sheela has scored a first class / in her final exams, / isn't it? A. Sheela has scored a first class B. in her final exams, C. isn't it? D. No error. Answer: Option C

Answer: Option C 15. Find out whether there is any grammatical error in below sentence. One of my favourite actor / is acting / in this play also. A. One of my favourite actor B. is acting C. in this play also. D. No error. Answer: Option A

146 | P a g e

19. Find out whether there is any 6. Find out whether there is any grammatical error in below sentence. grammatical error in below sentence. Supposing if / there is no bus. / how will Emphasis on equality of life ensures / for you get there? the health and happiness / of every A. Supposing if B. there is no bus. individual. C. how will you get there? D. No error. A. Emphasis on equality of life ensures B. for the health and happiness Answer: Option A C. of every individual. D. No error. Answer: Option B

20. Find out whether there is any grammatical error in below sentence. Because of the emergency help / that the 17. Find out whether there is any patient received / he would have died grammatical error in below sentence. A. Because of the emergency help There was very heavy rain last night, / and the rivers have overflown their banks B. that the patient received / causing severe hardship to the people C. he would have died D. No error. living by them. A. There was very heavy rain last night, B. and the rivers have overflown their Answer: Option A banks C. causing severe hardship to the people living by them. 21. Find out whether there is any D. No error. grammatical error in below sentence. He was in such hurry / that he didn't / wait Answer: Option B for me A. He was in such hurry B. that he didn't 18. Find out whether there is any C. wait for me D. No error. grammatical error in below sentence. Every motorist knows / road signs--Answer: Option A whether symbols or colour codes--- have an immediacy / that neither the spoken nor the written word can match. 22. Find out whether there is any A. Every motorist knows grammatical error in below sentence. B. road signs---whether symbols or Will you be / at Board meeting / on next colour codes--- have an immediacy Wednesday? C. that neither the spoken nor the written A. Will you be word can match. B. at Board meeting D. No error. C. on next Wednesday? D. No error. Answer: Option D Answer: Option C 147 | P a g e

23. Find out whether there is any grammatical error in below sentence. Do you know / A. Do you know B. to play C. the guitar? D. No error. Answer: Option B 24. Find out whether there is any grammatical error in below sentence. Few scientists changed / people's ideas as much as / Darwin with his Theory of Evolution. A. Few scientists changed B. people's ideas as much as C. Darwin with his Theory of Evolution. D. No error. Answer: Option A

27. Find out whether there is any grammatical error in below sentence. The eminent speaker's speech / was broadcasted over / all the major radiostations. A. The eminent speaker's speech B. was broadcasted over C. all the major radio-stations. D. No error. Answer: Option B 28. Find out whether there is any grammatical error in below sentence. The meeting adjourned abruptly / by the Chairman after / about three hours of deliberation. A. The meeting adjourned abruptly B. by the Chairman after C. about three hours of deliberation. D. No error.

25. Find out whether there is any grammatical error in below sentence. The course provide / not only theoretical Answer: Option A inputs / but also practical training A. The course provide B. not only theoretical inputs 29. Find out whether there is any C. but also practical training grammatical error in below sentence. D. No error. The company has put up an advertisement / in newspapers / all over Answer: Option A the country. A. The company has put up an 26. Find out whether there is any advertisement grammatical error in below sentence. B. in newspapers After the humiliating exposure / he C. all over the country. hanged his head / in shame. D. No error. A. After the humiliating exposure B. he hanged his head Answer: Option A C. in shame. D. No error. Answer: Option B 148 | P a g e

30. Find out whether there is any grammatical error in below sentence. Will you lend me / little money / to tide over this crisis. A. Will you lend me B. little money C. to tide over this crisis. D. No error. Answer: Option B

34. Find out whether there is any grammatical error in below sentence. The ability to plan, / organise and coordinate work is all fundamental / to working within deadline. A. The ability to plan, B. organise and coordinate work is all fundamental C. to working within deadline. D. No error. Answer: Option C

31. Find out whether there is any grammatical error in below sentence. He gave them no money / nor did help them / in any way. A. He gave them no money B. nor did help them C. in any way. D. No error.

35. Find out whether there is any grammatical error in below sentence. A group of friends / want to visit / the new plant as early as possible. A. A group of friends B. want to visit C. the new plant as early as possible. D. No error.

Answer: Option B

Answer: Option B

32. Find out whether there is any grammatical error in below sentence. The Sharmas / are living in this colony / for the last eight years. A. The Sharmas B. are living in this colony C. for the last eight years. D. No error. Answer: Option B

36. Find out whether there is any grammatical error in below sentence. After leaving his office / he went directly / to a restaurant. A. After leaving his office B. he went directly C. to a restaurant. D. No error. Answer: Option D

33. Find out whether there is any grammatical error in below sentence. My wife has got / a new job / a month ago. A. My wife has got B. a new job C. a month ago. D. No error.

37. Find out whether there is any grammatical error in below sentence. He was in a hurry / because he had an appointment / with the company's director. A. He was in a hurry B. because he had an appointment C. with the company's director. D. No error.

Answer: Option A

Answer: Option D

149 | P a g e

38. Find out whether there is any grammatical error in below sentence. A body of volunteers / have been organised / to spread the message of the saint. A. A body of volunteers B. have been organised C. to spread the message of the saint. D. No error. Answer: Option B 39. Find out whether there is any grammatical error in below sentence. When I get a cold / it takes me weeks / to shake it off. A. When I get a cold B. it takes me weeks C. to shake it off. D. No error.

41. Find out whether there is any grammatical error in below sentence. There is still / little tea / left in the cup. A. There is still B. little tea C. left in the cup. D. No error. Answer: Option B 42. Find out whether there is any grammatical error in below sentence. He says that / his car does / eight kilometers in a litre A. He says that B. his car does C. eight kilometers in a litre D. No error. Answer: Option C

43. Find out whether there is any grammatical error in below sentence. Answer: Option A After opening the door / we entered into the room / next to the kitchen 40. Find out whether there is any A. After opening the door grammatical error in below sentence. B. we entered into the room Another reason for pharmaceutical C. next to the kitchen companies beefing up their / OTC (Over D. No error. the Country) divisions is that prescription drugs with proven safety records which Answer: Option B have been reached / the end of the their patent protection periodare 44. Find out whether there is any A. Another reason for pharmaceutical grammatical error in below sentence. companies beefing up their B. OTC (Over the Country) divisions is Can I lend / your pencil / for a minute, that prescription drugs with proven safety please? A. Can I lend records which have been reached C. the end of the their patent protection B. your pencil C. for a minute, please? periodare D. No error. D. allowed to be sold without a prescription. No error Answer: Option A Answer: Option B 150 | P a g e

45. Find out whether there is any grammatical error in below sentence. Last month we celebrated / the wedding of our sister for whom / we have been looking for a suitable alliance for three years. A. Last month we celebrated B. the wedding of our sister for whom C. we have been looking for a suitable alliance for three years. D. No error.

48. Find out whether there is any grammatical error in below sentence. Unless you stop to make noise at once / I will have no option but to / bring the matter to the attention of the police. A. Unless you stop to make noise at once B. I will have no option but to C. bring the matter to the attention of the police. D. No error.

Answer: Option C

49. Find out whether there is any grammatical error in below sentence. He is generally / more hungry / than she is A. He is generally B. more hungry C. than she is D. No error.

46. Find out whether there is any grammatical error in below sentence. In an English paper / examiners should give as much weightage to language / as they give to contents. A. In an English paper B. examiners should give as much weightage to language C. as they give to contents. D. No error Answer: Option D 47. Find out whether there is any grammatical error in below sentence. I am hearing / a lot about / the problem of AIDS these days. A. I am hearing B. a lot about C. the problem of AIDS these days. D. No error. Answer: Option A

Answer: Option A

Answer: Option D 50. Find out whether there is any grammatical error in below sentence. Since India has gained Independence / 49 years ago. / much progress has been made in almost every field. A. Since India has gained Independence B. 49 years ago. C. much progress has been made in almost every field. D. No error. Answer: Option A 51. Find out whether there is any grammatical error in below sentence. He ensured his bank manager / that he would soon / repay the loan. A. He ensured his bank manager B. that he would soon C. repay the loan. D. No error. Answer: Option A

151 | P a g e

52. Find out whether there is any grammatical error in below sentence. A free press is not a privilege / but the organic necessity / in a free society. A. A free press is not a privilege B. but the organic necessity C. in a free society. D. No error. Answer: Option B 53. Find out whether there is any grammatical error in below sentence. He explained the matter / at great length / but I was not the wiser A. He explained the matter B. at great length C. but I was not the wiser D. No error. Answer: Option B 54. Find out whether there is any grammatical error in below sentence. He will end up his work / in the city / by the end of the year. A. He will end up his work B. in the city C. by the end of the year. D. No error. Answer: Option A

56. Find out whether there is any grammatical error in below sentence. He enquired me / why I had not seen him the previous day / as I had promised to do. A. He enquired me B. why I had not seen him the previous day C. as I had promised to do. D. No error. Answer: Option B 57. Find out whether there is any grammatical error in below sentence. I was there / many a time / in the past. A. I was there B. many a time C. in the past. D. No error. Answer: Option A 58. Find out whether there is any grammatical error in below sentence. All the four sons / of the old man / quarrelled between themselves. A. All the four sons B. of the old man C. quarrelled between themselves. D. No error. Answer: Option C

55. Find out whether there is any grammatical error in below sentence. Even though the shirt is rather expensive / but I wish to / purchase it with my own money. A. Even though the shirt is rather expensive B. but I wish to C. purchase it with my own money. D. No error.

59. Find out whether there is any grammatical error in below sentence. He wanted to work all night / but we saw that he was completely worn out A. He wanted to work all night B. but we saw that he was completely worn out C. and so we persuaded him to stop. D. No error.

Answer: Option B

Answer: Option B

152 | P a g e

60. Find out whether there is any grammatical error in below sentence. Mr.Smith was accused for murder / but the court found him not guilty / and acquitted him. A. Mr.Smith was accused for murder B. but the court found him not guilty C. and acquitted him. D. No error. Answer: Option A 61. Find out whether there is any grammatical error in below sentence. She sang / very well / isn't it? A. She sang B. very well C. isn't it? D. No error. Answer: Option C 62. Find out whether there is any grammatical error in below sentence. I am sure that all my monthly expenses / would exceed the income / If I do not economic A. I am sure that all my monthly expenses B. would exceed the income C. If I do not economic D. No error.

64. Find out whether there is any grammatical error in below sentence. Having read a number of stories / about space travel / his dream now is about to visit the moon A. Having read a number of stories B. about space travel C. his dream now is about to visit the moon D. No error. Answer: Option C 65. Find out whether there is any grammatical error in below sentence. The party chief made it a point to state that / the Prime Minister and the Union Home Minister should also come. / and they see what his party men had seen. A. The party chief made it a point to state that B. the Prime Minister and the Union Home Minister should also come. C. and they see what his party men had seen. D. No error. Answer: Option C

Answer: Option C

66. Find out whether there is any grammatical error in below sentence. It is easy to see that / a lawyer's 63. Find out whether there is any demeanour in court / may be prejudicial grammatical error in below sentence. Whenever you go to a temple / you must against the interests of his client. put off / your shoes at the entrance. A. It is easy to see that A. Whenever you go to a temple B. a lawyer's demeanour in court B. you must put off C. may be prejudicial against the interests C. your shoes at the entrance. of his client. D. No error. D. No error. Answer: Option B 153 | P a g e

Answer: Option D

67. Find out whether there is any grammatical error in below sentence. He is wiring / for the / last four hours A. He is wiring B. for the C. last four hours D. No error. Answer: Option A

69. Find out whether there is any grammatical error in below sentence. Were he / to see you, / he would have been surprised. A. Were he B. to see you, C. he would have been surprised. D. No error. Answer: Option A

68. Find out whether there is any grammatical error in below sentence. She walked in / the room where the murder / had taken place. A. She walked in B. the room where the murder C. had taken place. D. No error. Answer: Option A

154 | P a g e

70. Find out whether there is any grammatical error in below sentence. I could not convince them / because they persisted to suggest / that I was lying. A. I could not convince them B. because they persisted to suggest C. that I was lying. D. No error. Answer: Option B

Spotting Errors: Section 4 1. Find out whether there is any grammatical error in below sentence. If you listen with / the question carefully / you will be able to answer them easily A. If you listen with B. the question carefully C. you will be able to answer them easily D. No error. Answer: Option A 2. Find out whether there is any grammatical error in below sentence. It is difficult / for anyone / to past time thus. A. It is difficult B. for anyone C. to past time thus. D. No error. Answer: Option C 3. Find out whether there is any grammatical error in below sentence. The customer handed over / a hundredrupees note / to the shopkeeper. A. The customer handed over B. a hundred-rupees note C. to the shopkeeper. D. No error. Answer: Option B 4. Find out whether there is any grammatical error in below sentence. Myself and Gopal / will take care of / the function on sunday. A. Myself and Gopal B. will take care of C. the function on sunday. D. No error. Answer: Option A 155 | P a g e

5. Find out whether there is any grammatical error in below sentence. It is the newspaper / that exposes us to the widest range / of human experiences and behaviour. A. It is the newspaper B. that exposes us to the widest range C. of human experiences and behaviour. D. No error. Answer: Option D 6. Find out whether there is any grammatical error in below sentence. A small baby breathes about / 45 times per minute while / a child of about six years breathes about 25 times per minute. A. A small baby breathes about B. 45 times per minute while C. a child of about six years breathes about 25 times per minute. D. No error. Answer: Option D 7. Find out whether there is any grammatical error in below sentence. I have read / too many books / by R . K . Narayan. A. I have read B. too many books C. by R . K . Narayan. D. No error. Answer: Option B 8. Find out whether there is any grammatical error in below sentence. He was sure / that he should / win the Prize. A. He was sure B. that he should C. win the Prize. D. No error. Answer: Option B

9. Find out whether there is any grammatical error in below sentence. He is / too intelligent / to make a mistake. A. He is B. too intelligent C. to make a mistake. D. No error. Answer: Option D

12. Find out whether there is any grammatical error in below sentence. He deserted the path of honour / in order to / satisfy his ambition / and then went down his doom very quickly. A. He deserted the path of honour B. in order to C. satisfy his ambition D. and then went down his doom very quickly. E. No error. Answer: Option D

10. Find out whether there is any grammatical error in below sentence. He fell from a running train / and would have died / if the villagers did not get him admitted in the nearby hospital immediately. A. He fell from a running train B. and would have died C. if the villagers did not get him admitted in the nearby hospital immediately. D. No error. Answer: Option C 11. Find out whether there is any grammatical error in below sentence. Several issues raising / in the meeting could / be amicably resolved / due to his tactful handling. A. Several issues raising B. in the meeting could C. be amicably resolved D. due to his tactful handling. E. No error. Answer: Option A

13. Find out whether there is any grammatical error in below sentence. Nitin's nephew, who lives / in Bangalore, is eager to travel to / Delhi to have / a glimpse of the capital. A. Nitin's nephew, who lives B. in Bangalore, is eager to travel to C. Delhi to have D. a glimpse of the capital. E. No error. Answer: Option E 14. Find out whether there is any grammatical error in below sentence. Bible / can be / regarded as / a great. A. Bible B. can be C. regarded as D. a great. E. No error. Answer: Option A 15. Find out whether there is any grammatical error in below sentence. The institute / imparts training / in / the French. A. The institute B. imparts training C. in D. the French. E. No error. Answer: Option D

156 | P a g e

16. Find out whether there is any grammatical error in below sentence. His father promised to / give him anything what he / wants if he / passes in the examination. A. His father promised to B. give him anything what he C. wants if he D. passes in the examination. E. No error.

19. Find out whether there is any grammatical error in below sentence. I shall be able / to complete the work in / stipulated time provide / ou do not disturb me. A. I shall be able B. to complete the work in C. stipulated time provide D. ou do not disturb me.

Answer: Option B

20. Find out whether there is any grammatical error in below sentence. Our neighbours had repeated / the same illogical sequence of activities / if we had not brought the / facts to their notice. A. Our neighbours had repeated B. the same illogical sequence of activities C. if we had not brought the D. facts to their notice. E. No error.

17. Find out whether there is any grammatical error in below sentence. Being a very fast worker, / he is always liking / by all his colleagues. / and superiors too. A. Being a very fast worker, B. he is always liking C. by all his colleagues. D. and superiors too. E. No error. Answer: Option B 18. Find out whether there is any grammatical error in below sentence. While luminaries of the dance world / have a dearth of opportunities to display their art / upcoming dancers suffer from / an unfortunate lack of exposure. A. While luminaries of the dance world B. have a dearth of opportunities to display their art C. upcoming dancers suffer from D. an unfortunate lack of exposure. E. No error. Answer: Option B 157 | P a g e

Answer: Option C

Answer: Option A 21. Find out whether there is any grammatical error in below sentence. We are happy / A. We are happy B. to know that C. the project completed D. strictly as per the schedule. E. No error. Answer: Option C 22. Find out whether there is any grammatical error in below sentence. He was too / irritated to / concentrate on his work / for a long time. A. He was too B. irritated to C. concentrate on his work D. for a long time. E. No error. Answer: Option E

23. Find out whether there is any grammatical error in below sentence. The environment in which / companies operating / today is / undergoing Frenetic changes. A. The environment in which B. companies operating C. today is D. undergoing Frenetic changes. E. No error.

27. Find out whether there is any grammatical error in below sentence. When I / last see him / he was / in Calcutta. A. When I B. last see him C. he was D. in Calcutta. E. No error.

Answer: Option B

28. Find out whether there is any grammatical error in below sentence. The interviewer asked the actress / how could she / manage to attain fame / in a short period. A. The interviewer asked the actress B. how could she C. manage to attain fame D. in a short period. E. No error.

24. Find out whether there is any grammatical error in below sentence. He has been undergoing / the special training course / which each of the employees / is required to. A. He has been undergoing B. the special training course C. which each of the employees D. is required to. E. No error.

Answer: Option B

Answer: Option B

Answer: Option D

29. Find out whether there is any grammatical error in below sentence. 25. Find out whether there is any I cannot / make from / what you are grammatical error in below sentence. saying / about him. A lot of money / is wasted in / the A. I cannot duplication of work / in any organisation. B. make from A. A lot of money C. what you are saying B. is wasted in D. about him. C. the duplication of work E. No error. D. in any organisation. Answer: Option B E. No error. Answer: Option B

30. Find out whether there is any grammatical error in below sentence. 26. Find out whether there is any He has a scheme / of his own which he grammatical error in below sentence. thinks / more preferable to / that of any We decided not tell to / the patient about other person. / the disease he was / suffering from. A. He has a scheme A. We decided not tell to B. of his own which he thinks B. the patient about C. more preferable to C. the disease he was D. that of any other person. D. suffering from. E. No error. E. No error. Answer: Option C Answer: Option A 158 | P a g e

31. Find out whether there is any grammatical error in below sentence. Looking forward / to seeing / you / soon. A. Looking forward B. to seeing C. you D. soon. E. No error. Answer: Option B 32. Find out whether there is any grammatical error in below sentence. The historian / has been working / on the project / from last 12 years. A. The historian B. has been working C. on the project D. from last 12 years. E. No error. Answer: Option D 33. Find out whether there is any grammatical error in below sentence. You should inform / your superiors about / all those events which are / directly related to your work. A. You should inform B. your superiors about C. all those events which are D. directly related to your work. E. No error. Answer: Option E 34. Find out whether there is any grammatical error in below sentence. We now look forward for / some great achievements / which to some extent / can restore the country's prestige once again. A. We now look forward for B. some great achievements C. which to some extent D. can restore the country's prestige once again. E. No error. Answer: Option A 159 | P a g e

35. Find out whether there is any grammatical error in below sentence. No sooner the / clock strike six than / all the employees / rushed out of office. A. No sooner the B. clock strike six than C. all the employees D. rushed out of office. E. No error. Answer: Option A 36. Find out whether there is any grammatical error in below sentence. We play / tennis together / every morning / since last June. A. We play B. tennis together C. every morning D. since last June. E. No error. Answer: Option A 37. Find out whether there is any grammatical error in below sentence. Don't go by his appearance; / he is capable at / doing any work / without any hesitation. A. Don't go by his appearance; B. he is capable at C. doing any work D. without any hesitation. E. No error. Answer: Option B 38. Find out whether there is any grammatical error in below sentence. Even at the planning stage / Rajesh was quite sure, / that the scheme was / binding to fail. A. Even at the planning stage B. Rajesh was quite sure, C. that the scheme was D. binding to fail. E. No error. Answer: Option D

39. Find out whether there is any grammatical error in below sentence. Our school is making / every possible effort / to provide best facilities / and personal attention for each child. A. Our school is making B. every possible effort C. to provide best facilities D. and personal attention for each child. E. No error. Answer: Option D

42. Find out whether there is any grammatical error in below sentence. In the last week / I tell him to come / in time but he still / comes late every day. A. In the last week B. I tell him to come C. in time but he still D. comes late every day. E. No error. Answer: Option B

43. Find out whether there is any 40. Find out whether there is any grammatical error in below sentence. grammatical error in below sentence. You may not always / get whatever you In my opinion / the balance sheet deserve / but that does not belittle / the exhibits / a true and fair / view of the importance for your work. state of affairs of the bank. A. You may not always A. In my opinion B. get whatever you deserve B. the balance sheet exhibits C. but that does not belittle C. a true and fair D. the importance for your work. D. view of the state of affairs of the bank. E. No error. E. No error. Answer: Option D Answer: Option E 41. Find out whether there is any grammatical error in below sentence. At last the rain ceased / and the sky was / cleared by clouds / and lightning. A. At last the rain ceased B. and the sky was C. cleared by clouds D. and lightning. E. No error.

44. Find out whether there is any grammatical error in below sentence. When I stood up spontaneously / and questioned the speaker; / someone commented that / it was a boldly step. A. When I stood up spontaneously B. and questioned the speaker; C. someone commented that D. it was a boldly step. E. No error.

Answer: Option C

Answer: Option D

160 | P a g e

45. Find out whether there is any grammatical error in below sentence. The Head of the Department along with his colleagues / are coming to attend / the conference which is / the conference which is A. The Head of the Department along with his colleagues B. are coming to attend C. the conference which is D. the conference which is E. No error.

48. Find out whether there is any grammatical error in below sentence. The patient recover / so fast that / the expert doctors / also were surprised. A. The patient recover B. so fast that C. the expert doctors D. also were surprised. E. No error. Answer: Option A

Answer: Option B 49. Find out whether there is any grammatical error in below sentence. 46. Find out whether there is any Naren could not / decide as to which / grammatical error in below sentence. The Head of the Department along with course he should do / after obtaining his Degree. his colleagues / are coming to attend / the conference which is / the conference A. Naren could not which is B. decide as to which A. The Head of the Department along C. course he should do with his colleagues D. after obtaining his Degree. B. are coming to attend E. No error. C. the conference which is D. the conference which is Answer: Option E E. No error. Answer: Option B 47. Find out whether there is any grammatical error in below sentence. From over last five years / I have been trying / my best to convince him / but without success. A. From over last five years B. I have been trying C. my best to convince him D. but without success. E. No error. Answer: Option A

50. Find out whether there is any grammatical error in below sentence. Due to certain inevitable circumstances / the scheduled programme had to be / postponed indefinite / but the members could not be informed. A. Due to certain inevitable circumstances B. the scheduled programme had to be C. postponed indefinite D. but the members could not be informed. E. No error. Answer: Option C

161 | P a g e

51. Find out whether there is any grammatical error in below sentence. The mission provides / able service to all / the needy people in this area / during last few years. A. The mission provides B. able service to all C. the needy people in this area D. during last few years. E. No error.

54. Find out whether there is any grammatical error in below sentence. By arresting the local criminals / and encouraging good people / we can end / hostilities of that area. A. By arresting the local criminals B. and encouraging good people C. we can end D. hostilities of that area. E. No error.

Answer: Option A

Answer: Option C

52. Find out whether there is any grammatical error in below sentence. The success of our efforts / depend upon the / number of people who / actually use the new methods. A. The success of our efforts B. depend upon the C. number of people who D. actually use the new methods. E. No error.

55. Find out whether there is any grammatical error in below sentence. Schools in that area which / had English as a / medium of instruction / as the majority there spoke English. A. Schools in that area which B. had English as a C. medium of instruction D. as the majority there spoke English. E. No error.

Answer: Option B

Answer: Option A

53. Find out whether there is any grammatical error in below sentence. I will put on / a note in this regard / for your consideration / and necessary decision. A. I will put on B. a note in this regard C. for your consideration D. and necessary decision. E. No error.

56. Find out whether there is any grammatical error in below sentence. I have had to work / at the fountain for almost / ten hours before it could / start functioning well. A. I have had to work B. at the fountain for almost C. ten hours before it could D. start functioning well. E. No error.

Answer: Option A

Answer: Option A

162 | P a g e

57. Find out whether there is any grammatical error in below sentence. We admired the way / he had completed all his work / and appreciating the method / adopted by him. A. We admired the way B. he had completed all his work C. and appreciating the method D. adopted by him. E. No error. Answer: Option C

61. Find out whether there is any grammatical error in below sentence. The Trust has succeeded / admirably in raising / money for / its future programmes. A. The Trust has succeeded B. admirably in raising C. money for D. its future programmes. E. No error. Answer: Option E

58. Find out whether there is any grammatical error in below sentence. Had I come / to know about / his difficulties / would have certainly helped. A. Had I come B. to know about C. his difficulties D. would have certainly helped. E. No error. Answer: Option D 59. Find out whether there is any grammatical error in below sentence. Whatever he was / today is only because / of his mother who / was a renowned scientist. A. Whatever he was B. today is only because C. of his mother who D. was a renowned scientist. E. No error. Answer: Option A 60. Find out whether there is any grammatical error in below sentence. No girl / in her troupe is / so sprightly as / your daughter. A. No girl B. in her troupe is C. so sprightly as D. your daughter. E. No error. Answer: Option C 163 | P a g e

62. Find out whether there is any grammatical error in below sentence. The committee is thankful to Shri Desai / for preparing not only the main report / but also for preparing / the agenda notes and minutes. A. The committee is thankful to Shri Desai B. for preparing not only the main report C. but also for preparing D. the agenda notes and minutes. E. No error. Answer: Option C 63. Find out whether there is any grammatical error in below sentence. Motivating employees with / traditional authority and financial / incentives have become / increasingly difficult. A. Motivating employees with B. traditional authority and financial C. incentives have become D. increasingly difficult. E. No error. Answer: Option C

64. Find out whether there is any grammatical error in below sentence. We have observed that / many good programmes / suffer of shortage / of funds and other resources. A. We have observed that B. many good programmes C. suffer of shortage D. of funds and other resources. E. Error.

68. Find out whether there is any grammatical error in below sentence. Do not trouble / yourself about writing to me / unless you are quite / in the humour for it. A. Do not trouble B. yourself about writing to me C. unless you are quite D. in the humour for it. E. No error.

Answer: Option C

Answer: Option D

65. Find out whether there is any grammatical error in below sentence. If the teacher / is good the students / will respond / positively to them. A. If the teacher B. is good the students C. will respond D. positively to them. E. No error. Answer: Option D 66. Find out whether there is any grammatical error in below sentence. The apparently obvious solutions / to most of his problems / were overlook by / many of his friends. A. The apparently obvious solutions B. to most of his problems C. were overlook by D. many of his friends. E. No error. Answer: Option C 67. Find out whether there is any grammatical error in below sentence. Such / rules do not / apply to / you and I A. Such B. rules do not C. apply to D. you and I E. No error. Answer: Option D 164 | P a g e

69. Find out whether there is any grammatical error in below sentence. Scarcely had I / finished washing the car A. Scarcely had I B. finished washing the car C. than the master came D. and asked me to clean the floor of the house. E. No error. Answer: Option C 70. Find out whether there is any grammatical error in below sentence. The Head of the Department / advised all the staff / to not to / A. The Head of the Department B. advised all the staff C. to not to D. indulge in gossip. E. No error. Answer: Option C

Spotting Errors: Section 5 1. Find out whether there is any grammatical error in below sentence. As it was Ramesh's / first interview he dressed him / in his most / formal suit. A. As it was Ramesh's B. first interview he dressed him C. in his most D. formal suit. E. No error. Answer: Option B 2. Find out whether there is any grammatical error in below sentence. Developmental activities of / the government come / to a standstill due / to paucity of funds. A. Developmental activities of B. the government come C. to a standstill due D. to paucity of funds. E. No error. Answer: Option B

4. Find out whether there is any grammatical error in below sentence. I was to about / go out of my house / when it suddenly / started raining. A. I was to about B. go out of my house C. when it suddenly D. started raining. E. No error. Answer: Option A 5. Find out whether there is any grammatical error in below sentence. I am pleased to sanction / one special increment / to all the employees / with this month. A. I am pleased to sanction B. one special increment C. to all the employees D. with this month. E. No error. Answer: Option D

3. Find out whether there is any grammatical error in below sentence. Of the two / sisters / Meena is / the elder. A. Of the two B. sisters C. Meena is D. the elder. E. No error.

6. Find out whether there is any grammatical error in below sentence. While Mahendra was away / on a long official tour / his office receive an important letter / which was marked 'Urgent'. A. While Mahendra was away B. on a long official tour C. his office receive an important letter D. which was marked 'Urgent'. E. No error.

Answer: Option D

Answer: Option C

165 | P a g e

7. Find out whether there is any grammatical error in below sentence. No country can long endure / if its foundations / were not laid deep / in the material prosperity. A. No country can long endure B. if its foundations C. were not laid deep D. in the material prosperity. E. No error. Answer: Option C 8. Find out whether there is any grammatical error in below sentence. Rajesh was expecting / a telegram from his uncle / which would inform / him whether he went or not. A. Rajesh was expecting B. a telegram from his uncle C. which would inform D. him whether he went or not. E. No error. Answer: Option C 9. Find out whether there is any grammatical error in below sentence. I see you / in Kanpur / during my next visit / in the month of May. A. I see you B. in Kanpur C. during my next visit D. in the month of May. E. No error Answer: Option A 10. Find out whether there is any grammatical error in below sentence. He ought / not have / done such a / filthy thing. A. He ought B. not have C. done such a D. filthy thing. E. No error. Answer: Option B 166 | P a g e

11. Find out whether there is any grammatical error in below sentence. We have keep / our promise and / you can expect / a lot from us in future. A. We have keep B. our promise and C. you can expect D. a lot from us in future. E. No error. Answer: Option A 12. Find out whether there is any grammatical error in below sentence. I would have lost / my luggage and other belongings / if I would have left the compartment / A. I would have lost B. my luggage and other belongings C. if I would have left the compartment D. and gone out to fetch drinking water. E. No error. Answer: Option C 13. Find out whether there is any grammatical error in below sentence. The last year proved / quite bad / as major industries / witness lot of problems. A. The last year proved B. quite bad C. as major industries D. witness lot of problems. E. No error. Answer: Option D

14. Find out whether there is any grammatical error in below sentence. In spite of toiling / very hardly he / realised that he had / not earned anything substantial. A. In spite of toiling B. very hardly he C. realised that he had D. not earned anything substantial. E. No error.

17. Find out whether there is any grammatical error in below sentence. We have to take / him to the hospital yesterday / because he was / suffering from fever. A. We have to take B. him to the hospital yesterday C. because he was D. suffering from fever. E. No error. Answer: Option A

Answer: Option B 15. Find out whether there is any grammatical error in below sentence. Of all the men / known to me none is / less inclined than he is / to think ill to others. A. Of all the men B. known to me none is C. less inclined than he is D. to think ill to others. E. No error. Answer: Option D 16. Find out whether there is any grammatical error in below sentence. I believe that / next time when you / will be coming to Mumbai / A. I believe that B. next time when you C. will be coming to Mumbai D. you will bring my book. E. No error. Answer: Option C

18. Find out whether there is any grammatical error in below sentence. He / has seen / the picture / yesterday. A. He B. has seen C. the picture D. yesterday. E. No error. Answer: Option B 19. Find out whether there is any grammatical error in below sentence. But for your / kind help, this / task could not / have been completed. A. But for your B. kind help, this C. task could not D. have been completed. E. No error. Answer: Option E 20. Find out whether there is any grammatical error in below sentence. Not one of the children / has ever sang / on any occasion / in public before. A. Not one of the children B. has ever sang C. on any occasion D. in public before. E. No error. Answer: Option B

167 | P a g e

21. Find out whether there is any grammatical error in below sentence. Somewhere along the / line I lost / track of what Ashish / says about heart ailments A. Somewhere along the B. line I lost C. track of what Ashish D. says about heart ailments E. No error.

24. Find out whether there is any grammatical error in below sentence. Everyone is / impress by / his zeal / and enthusiasm. A. Everyone is B. impress by C. his zeal D. and enthusiasm. E. No error.

Answer: Option D

25. Find out whether there is any grammatical error in below sentence. His father told me / that though his son had / worked very hard, / but he had failed to make any mark in the examination. A. His father told me B. that though his son had C. worked very hard, D. but he had failed to make any mark in the examination. E. No error.

22. Find out whether there is any grammatical error in below sentence. An anti-extortion cell is opened / by the district police headquarter / six months ago / as a precautionary measure. A. An anti-extortion cell is opened B. by the district police headquarter C. six months ago D. as a precautionary measure. E. No error.

Answer: Option B

Answer: Option D

Answer: Option A 26. Find out whether there is any 23. Find out whether there is any grammatical error in below sentence. grammatical error in below sentence. The teacher promised / that he will Hindi and Marathi are / different forms explain it / if they come / before school of the Sanskrit / which were once spoken the following day. / in almost every part of India. A. The teacher promised A. Hindi and Marathi are B. that he will explain it B. different forms of the Sanskrit C. if they come C. which were once spoken D. before school the following day. D. in almost every part of India. E. No error. E. No error. Answer: Option B Answer: Option C

168 | P a g e

27. Find out whether there is any grammatical error in below sentence. After toiling very hard / over a long period / he found / he had hardly made no profit at all. A. After toiling very hard B. over a long period C. he found D. he had hardly made no profit at all. E. No error.

30. Find out whether there is any grammatical error in below sentence. One of the objective / of the meeting which / of the meeting which / elect new office-bearers. A. One of the objective B. of the meeting which C. of the meeting which D. elect new office-bearers. E. No error.

Answer: Option D

Answer: Option A

28. Find out whether there is any grammatical error in below sentence. Scarcely had / he gone a few steps / that he was told / that his mother was no more. A. Scarcely had B. he gone a few steps C. that he was told D. that his mother was no more. E. No error.

31. Find out whether there is any grammatical error in below sentence. Why did people get / somuch annoyed / even with little provocation / is a matter of investigation. A. Why did people get B. somuch annoyed C. even with little provocation D. is a matter of investigation. E. No error.

Answer: Option C

Answer: Option A

29. Find out whether there is any grammatical error in below sentence. The observers felt that / the stronger team had to face / defeat because the players / didn't play whole hearted. A. The observers felt that B. the stronger team had to face C. defeat because the players D. didn't play whole hearted. E. No error.

32. Find out whether there is any grammatical error in below sentence. When the national / anthem was being / sung, everyone were / standing in silence. A. When the national B. anthem was being C. sung, everyone were D. standing in silence. E. No error. Answer: Option C

Answer: Option D

169 | P a g e

33. Find out whether there is any grammatical error in below sentence. Having learn my lessons / I was very careful / in dealing with him / in front of his room-mate. A. Having learn my lessons B. I was very careful C. in dealing with him D. in front of his room-mate. E. No error. Answer: Option A

36. Find out whether there is any grammatical error in below sentence. Mr.Raman said that / he had a difference / with / the chairman at his statement. A. Mr.Raman said that B. he had a difference C. with D. the chairman at his statement. E. No error. Answer: Option D

37. Find out whether there is any 34. Find out whether there is any grammatical error in below sentence. grammatical error in below sentence. During last days / I was continuously He neglects / attending lectures / trying / to contact you / but you were regularly / though college was only a few not available. yards away from his house. A. During last days A. He neglects B. I was continuously trying B. attending lectures C. to contact you C. regularly D. but you were not available. D. though college was only a few yards E. No error away from his house. E. No error. Answer: Option A Answer: Option D 35. Find out whether there is any grammatical error in below sentence. He tried as he could / Naveen did not / succeed in getting / his car to start up. A. He tried as he could B. Naveen did not C. succeed in getting D. his car to start up. Answer: Option D

170 | P a g e

38. Find out whether there is any grammatical error in below sentence. We were shocking / to hear the news / of the untimely death / of the leader. A. We were shocking B. to hear the news C. of the untimely death D. of the leader. E. No error. Answer: Option A

39. Find out whether there is any grammatical error in below sentence. I do not recall / exactly what he said to me / but when I was quit / he said something to me. A. I do not recall B. exactly what he said to me C. but when I was quit D. he said something to me. E. No error. Answer: Option B

42. Find out whether there is any grammatical error in below sentence. After listening to / his advice, I / decided to not to / go abroad for studies. A. After listening to B. his advice, I C. decided to not to D. go abroad for studies. E. No error. Answer: Option

43. Find out whether there is any 40. Find out whether there is any grammatical error in below sentence. grammatical error in below sentence. As the ticket was / firm affixed / on the The health workers are / being tried their envelope, he / could not remove it. best / to popularise / preventive A. As the ticket was measures. B. firm affixed A. The health workers are C. on the envelope, he B. being tried their best D. could not remove it. C. to popularise E. No error. D. preventive measures. E. No error. Answer: Option B Answer: Option B 44. Find out whether there is any grammatical error in below sentence. 41. Find out whether there is any He had to / seek legal help in / order for grammatical error in below sentence. settling / the dispute. Kamlesh asked the dealer / what was the A. He had to price / of that bicycle and whether / it is B. seek legal help in really made in Germany? C. order for settling A. Kamlesh asked the dealer D. the dispute. B. what was the price E. No error. C. of that bicycle and whether D. it is really made in Germany? Answer: Option C E. No error. Answer: Option B

171 | P a g e

45. Find out whether there is any grammatical error in below sentence. I asked him / whom he thought / would be able to / get the first prize. A. I asked him B. whom he thought C. would be able to D. get the first prize. E. No error. Answer: Option B 46. Find out whether there is any grammatical error in below sentence. Mahatma Gandhi did not solve / all the problems of the future / but he did solve / problems of his own age. A. Mahatma Gandhi did not solve B. all the problems of the future C. but he did solve D. problems of his own age. E. No error. Answer: Option B

48. Find out whether there is any grammatical error in below sentence. One of the party / members were dismissed / for speaking against / the leader. A. One of the party B. members were dismissed C. for speaking against D. the leader. E. No error. Answer: Option B 49. Find out whether there is any grammatical error in below sentence. No sooner the plane landed / at the airport than / a group of armed / commandos surrounded it. A. No sooner the plane landed B. at the airport than C. a group of armed D. commandos surrounded it. E. No error. Answer: Option A

47. Find out whether there is any grammatical error in below sentence. We never thought / that Mahesh is / oldest than the other / players in the team. A. We never thought B. that Mahesh is C. oldest than the other D. players in the team. E. No error.

50. Find out whether there is any grammatical error in below sentence. No sooner the plane landed / at the airport than / a group of armed / commandos surrounded it. A. No sooner the plane landed B. at the airport than C. a group of armed D. commandos surrounded it. E. No error.

Answer: Option C Answer: Option A

172 | P a g e

51. Find out whether there is any grammatical error in below sentence. The job is much worse than I expected / If I would have realised / how awful it was going to be / I would not have accepted it. A. The job is much worse than I expected B. If I would have realised C. how awful it was going to be D. I would not have accepted it. E. No error.

54. Find out whether there is any grammatical error in below sentence. Well, I spend six or seven years / after high school / trying to find a job for me / but could not succeed in it. A. Well, I spend six or seven years B. after high school C. trying to find a job for me D. but could not succeed in it. E. No error. Answer: Option A

Answer: Option B

55. Find out whether there is any grammatical error in below sentence. All renew licences / may be collected from / 52. Find out whether there is any the cashier's counter after / paying the fees. grammatical error in below sentence. A. All renew licences The job is much worse than I expected / B. may be collected from If I would have realised / how awful it C. the cashier's counter after was going to be / I would not have D. paying the fees. accepted it. E. No error. A. The job is much worse than I Answer: Option A expected B. If I would have realised C. how awful it was going to be 56. Find out whether there is any D. I would not have accepted it. grammatical error in below sentence. E. No error. All his relatives / except his daughter / have gone on / a month's vacation tour. Answer: Option B A. All his relatives B. except his daughter C. have gone on 53. Find out whether there is any D. a month's vacation tour. grammatical error in below sentence. We have done everything / that could be E. No error. done / to avert the storm / which is now Answer: Option C coming on. A. We have done everything 57. Find out whether there is any B. that could be done grammatical error in below sentence. C. to avert the storm No sooner did he / got up from bed / than D. which is now coming on. he was sent / to the dairy. E. No error. A. No sooner did he Answer: Option A B. got up from bed C. than he was sent D. to the dairy. E. No error. Answer: Option B 173 | P a g e

58. Find out whether there is any grammatical error in below sentence. In spite of the difficulties / on the way / they enjoyed their / trip to Gangothri. A. In spite of the difficulties B. on the way C. they enjoyed their D. trip to Gangothri. E. No error. Answer: Option E 59. Find out whether there is any grammatical error in below sentence. There is just not enough / timing in my job to sit around / talking about how we feel / about each other. A. There is just not enough B. timing in my job to sit around C. talking about how we feel D. about each other. E. No error. Answer: Option B

61. Find out whether there is any grammatical error in below sentence. We can not handle / this complicated case today / unless full details are not given / to us by now. A. We can not handle B. this complicated case today C. unless full details are not given D. to us by now. E. No error. Answer: Option C 62. Find out whether there is any grammatical error in below sentence. The customer scarcely had / enough money to pay / to the cashier / at the cash counter. A. The customer scarcely had B. enough money to pay C. to the cashier D. at the cash counter. E. No error. Answer: Option A

60. Find out whether there is any grammatical error in below sentence. Hardly had the / sad news reached her / ears when she / broke into tears. A. Hardly had the B. sad news reached her C. ears when she D. broke into tears. E. No errors. Answer: Option E

63. Find out whether there is any grammatical error in below sentence. We don't deny / your right to know / whatever happened while / you were not in the office. A. We don't deny B. your right to know C. whatever happened while D. you were not in the office. E. No error. Answer: Option E

174 | P a g e

64. Find out whether there is any grammatical error in below sentence. Neither of the plans / suits him and therefore / he decided not to / go out yesterday. A. Neither of the plans B. suits him and therefore C. he decided not to D. go out yesterday. E. No error Answer: Option B 65. Find out whether there is any grammatical error in below sentence. I was being astonished / when I heard that / he had left the country / without informing anyone of us. A. I was being astonished B. when I heard that C. he had left the country D. without informing anyone of us. E. No error. Answer: Option A

67. Find out whether there is any grammatical error in below sentence. He is smarter / enough to get / selected for this / prestigious post. A. He is smarter B. enough to get C. selected for this D. prestigious post. E. No error. Answer: Option A 68. Find out whether there is any grammatical error in below sentence. What / you will think / if school boys / make fun of you? A. What B. you will think C. if school boys D. make fun of you? E. No error. Answer: Option B 69. Find out whether there is any grammatical error in below sentence. I have / the firm / belief in the basic goodness of / all men. A. I have B. the firm C. belief in the basic goodness of D. all men. E. No error. Answer: Option B

66. Find out whether there is any grammatical error in below sentence. Since I had been gone / through the book / long back, I could / not remember the contents. A. Since I had been gone B. through the book C. long back, I could D. not remember the contents. E. No error. Answer: Option A

175 | P a g e

70. Find out whether there is any grammatical error in below sentence. We / have been living / in this house / since two years. A. We B. have been living C. in this house D. since two years. E. No error. Answer: Option D

Spotting Errors: Section 6 1. Find out whether there is any grammatical error in below sentence. He is / a mason / who / built my house. A. He is B. a mason C. who D. built my house. E. No error. Answer: Option B 2. Find out whether there is any grammatical error in below sentence. He served / the country with / heart and soul / but got nothing in return. A. He served B. the country with C. heart and soul D. but got nothing in return. E. No error. Answer: Option B 3. Find out whether there is any grammatical error in below sentence. He served / the country with / heart and soul / but got nothing in return. A. He served B. the country with C. heart and soul D. but got nothing in return. E. No error. Answer: Option B

4. Find out whether there is any grammatical error in below sentence. I am contacting you / sometime in next week / to explain to you / my problem in detail. A. I am contacting you B. sometime in next week C. to explain to you D. my problem in detail. E. No error. Answer: Option B 5. Find out whether there is any grammatical error in below sentence. Manohar has not only opened / a restaurant, but also / a grocery shop in the / village where we live A. Manohar has not only opened B. a restaurant, but also C. a grocery shop in the D. village where we live E. No error. Answer: Option A 6. Find out whether there is any grammatical error in below sentence. The daily wages that / the workers of this / factory receive range / between twenty to thirty rupees. A. The daily wages that B. the workers of this C. factory receive range D. between twenty to thirty rupees. E. No error. Answer: Option D

176 | P a g e

7. Find out whether there is any grammatical error in below sentence. His speech was / judged by many / as one of the most important speech / given in the function. A. His speech was B. judged by many C. as one of the most important speech D. given in the function. E. No error. Answer: Option C 8. Find out whether there is any grammatical error in below sentence. You must had / a kind and gentle heart / if you want / to be a successful doctor. A. You must had B. a kind and gentle heart C. if you want D. to be a successful doctor. E. No error. Answer: Option A 9. Find out whether there is any grammatical error in below sentence. They would not / have able to plan / the details of the job, / if you had not cooperated. A. They would not B. have able to plan C. the details of the job, D. if you had not cooperated. E. No error.

11. Find out whether there is any grammatical error in below sentence. He has in / his possession a / price collection of very old coins. / and some ancient paintings. A. He has in B. his possession a C. price collection of very old coins. D. and some ancient paintings. E. No error. Answer: Option C 12. Find out whether there is any grammatical error in below sentence. They wanted money / to purchase certain things / for themeselves and / for donated to their colleagues. A. They wanted money B. to purchase certain things C. for themeselves and D. for donated to their colleagues. E. No error. Answer: Option D

13. Find out whether there is any grammatical error in below sentence. According to one survey / only those forests which were / not under village Answer: Option B management / succumbed from fires recently. A. According to one survey 10. Find out whether there is any B. only those forests which were grammatical error in below sentence. Looking forward / to / meet you / here. C. not under village management A. Looking forward B. to D. succumbed from fires recently. C. meet you D. here. E. No error. E. No error Answer: Option C 177 | P a g e

Answer: Option D

14. Find out whether there is any grammatical error in below sentence. According to one survey / only those forests which were / not under village management / succumbed from fires recently. A. According to one survey B. only those forests which were C. not under village management D. succumbed from fires recently. E. No error.

17. Find out whether there is any grammatical error in below sentence. Not only the judges acquited / him of all the charges / levelled against him, but / also commended all his actions. A. Not only the judges acquited B. him of all the charges C. levelled against him, but D. also commended all his actions. E. No error. Answer: Option A

Answer: Option D 18. Find out whether there is any 15. Find out whether there is any grammatical error in below sentence. grammatical error in below sentence. Not only the judges acquited / him of all We must go / and congratulate him for / the charges / levelled against him, but / his brilliant / performance. also commended all his actions. A. We must go A. Not only the judges acquited B. and congratulate him for B. him of all the charges C. his brilliant C. levelled against him, but D. performance. D. also commended all his actions. E. No error. E. No error. Answer: Option B

Answer: Option A

16. Find out whether there is any grammatical error in below sentence. Are you awarefor along time that / by making me waiting / for a long time / my schedule is getting upset? A. Are you awarefor along time that B. by making me waiting C. for a long time D. my schedule is getting upset? E. No error.

19. Find out whether there is any grammatical error in below sentence. This book is / designed to increase / a working knowledge of / spoken and written English. A. This book is B. designed to increase C. a working knowledge of D. spoken and written English. E. No error.

Answer: Option B

Answer: Option C

178 | P a g e

20. Find out whether there is any grammatical error in below sentence. This book is / designed to increase / a working knowledge of / spoken and written English. A. This book is B. designed to increase C. a working knowledge of D. spoken and written English. E. No error.

23. Find out whether there is any grammatical error in below sentence. According to me / the Indians in general is / not a vigilant / and security conscious people. A. According to me B. the Indians in general is C. not a vigilant D. and security conscious people. E. No error.

Answer: Option C

Answer: Option B

21. Find out whether there is any grammatical error in below sentence. Even if the doctor / put in his best efforts, he / could not succeed in / saving the patient. A. Even if the doctor B. put in his best efforts, he C. could not succeed in D. saving the patient. E. No error.

24. Find out whether there is any grammatical error in below sentence. Honesty and integrity are / the qualities which cannot be / done away with / and hence assume a lot of importance. A. Honesty and integrity are B. the qualities which cannot be C. done away with D. and hence assume a lot of importance. E. No error.

Answer: Option A

Answer: Option E

22. Find out whether there is any grammatical error in below sentence. The soap brought in the / market recently is not / very fragrant and / is too expensive. A. The soap brought in the B. market recently is not C. very fragrant and D. is too expensive. E. No error.

25. Find out whether there is any grammatical error in below sentence. Mangesh cannot be able / to finish this work / unless he takes the / help of his colleagues. A. Mangesh cannot be able B. to finish this work C. unless he takes the D. help of his colleagues. E. No error.

Answer: Option A

Answer: Option A

179 | P a g e

26. Find out whether there is any grammatical error in below sentence. Since I meet / Dinesh last Saturday / he has been contacting me / everyday over phone. A. Since I meet B. Dinesh last Saturday C. he has been contacting me D. everyday over phone. E. No error. Answer: Option A 27. Find out whether there is any grammatical error in below sentence. Even after worked in the office / for as many as fifteen years, / he still does not understand / the basic objectives of the work. A. Even after worked in the office B. for as many as fifteen years, C. he still does not understand D. the basic objectives of the work. E. No error. Answer: Option A 28. Find out whether there is any grammatical error in below sentence. The principals of equal justice / for all is one of / the cornerstones of our / democratic way of life. A. The principals of equal justice B. for all is one of C. the cornerstones of our D. democratic way of life. E. No error. Answer: Option A 180 | P a g e

29. Find out whether there is any grammatical error in below sentence. We are trying / to locate the / historical city for / the past two years. A. We are trying B. to locate the C. historical city for D. the past two years. E. No error. Answer: Option A 30. Find out whether there is any grammatical error in below sentence. Hardly had I / entered the compartment / than I met / my friend Vishwas. A. Hardly had I B. entered the compartment C. than I met D. my friend Vishwas. E. No error. Answer: Option C 31. Find out whether there is any grammatical error in below sentence. It seems evidents / to me that / the visits they made to the island / were not very frequent. A. It seems evidents B. to me that C. the visits they made to the island D. were not very frequent. E. No error. Answer: Option A

32. Find out whether there is any grammatical error in below sentence. Unintentionally I let / out the secret / by talking about / it loudly in the library. A. Unintentionally I let B. out the secret C. by talking about D. it loudly in the library. E. No error. Answer: Option E 33. Find out whether there is any grammatical error in below sentence. You deserve to be / rewarded by your successful / completion of the entire work / in such a short time. A. You deserve to be B. rewarded by your successful C. completion of the entire work D. in such a short time. E. No error. Answer: Option B 34. Find out whether there is any grammatical error in below sentence. Ever since the government / announced its new policy / the private institutions had / run into heavy weather. A. Ever since the government B. announced its new policy C. the private institutions had D. run into heavy weather. E. No error. Answer: Option C

35. Find out whether there is any grammatical error in below sentence. How you eat / is as important / as what / you eat. A. How you eat B. is as important C. as what D. you eat. E. No error. Answer: Option E 36. Find out whether there is any grammatical error in below sentence. Ramesh did not like / leaving his old parents alone in the house / but he had no alternative / as he has to go out to work. A. Ramesh did not like B. leaving his old parents alone in the house C. but he had no alternative D. as he has to go out to work. E. No error. Answer: Option D 37. Find out whether there is any grammatical error in below sentence. We wanted to purchase / something but all the three stores / in that area / were closed on that day. A. We wanted to purchase B. something but all the three stores C. in that area D. were closed on that day. E. No error. Answer: Option C 38. Find out whether there is any grammatical error in below sentence. Both the brothers are / so good-natured that / they look at their / old and aged parents very well. A. Both the brothers are B. so good-natured that C. they look at their D. old and aged parents very well. E. No error. Answer: Option C

181 | P a g e

39. Find out whether there is any grammatical error in below sentence. Ketan had a lot / of work to complete yesterday / and wishes / that he had my help A. Ketan had a lot B. of work to complete yesterday C. and wishes D. that he had my help E. No error.

42. Find out whether there is any grammatical error in below sentence. No sooner did / the chairman begin speaking / some participants started / shouting slogans. A. No sooner did B. the chairman begin speaking C. some participants started D. shouting slogans. E. No error.

Answer: Option C

Answer: Option C

40. Find out whether there is any grammatical error in below sentence. Foolishly Rajani opened / the cooker when / it was full steam / and burnt her hands. A. Foolishly Rajani opened B. the cooker when C. it was full steam D. and burnt her hands. E. No error.

43. Find out whether there is any grammatical error in below sentence. Scarcely had / he gone / when a policeman / knocked at the door. A. Scarcely had B. he gone C. when a policeman D. knocked at the door. E. No error. Answer: Option E

Answer: Option C 44. Find out whether there is any 41. Find out whether there is any grammatical error in below sentence. grammatical error in below sentence. He fixed a metal ladder / for the wall The book is making / waves and the sale below his window / so as to be able to / / is quite brisk in / all major cities. escape if there was a fire. A. The book is making A. He fixed a metal ladder B. waves and the sale B. for the wall below his window C. is quite brisk in C. so as to be able to D. all major cities. D. escape if there was a fire. E. No error. E. No error. Answer: Option C

182 | P a g e

Answer: Option B

45. Find out whether there is any grammatical error in below sentence. The recent study has / indicated that there is / perceptible change in / the attitudes of the people. A. The recent study has B. indicated that there is C. perceptible change in D. the attitudes of the people. E. No error. Answer: Option C

48. Find out whether there is any grammatical error in below sentence. In spite of the rumours / of an impending takeover / by the government / Ramlal bought more shares of that company. A. In spite of the rumours B. of an impending takeover C. by the government D. Ramlal bought more shares of that company. E. No error. Answer: Option B

46. Find out whether there is any grammatical error in below sentence. Jayant told me / that Mihir expect / every friend to meet him / once in a week. A. Jayant told me B. that Mihir expect C. every friend to meet him D. once in a week. E. No error. Answer: Option B

49. Find out whether there is any grammatical error in below sentence. Ketaki would have / surely got the job / if she would have / attended the interview. A. Ketaki would have B. surely got the job C. if she would have D. attended the interview. E. No error. Answer: Option C

47. Find out whether there is any grammatical error in below sentence. No sooner had the jeep / arrives the station / than a young police officer / jumped out of it. A. No sooner had the jeep B. arrives the station C. than a young police officer D. jumped out of it. E. No error. Answer: Option B

183 | P a g e

50. Find out whether there is any grammatical error in below sentence. No sooner / we reached there / than it started raining / No mistake. A. No sooner B. we reached there C. than it started raining D. No mistake. E. No error. Answer: Option B

51. Find out whether there is any grammatical error in below sentence. The student / answered to / the question / asked by the Inspector. A. The student B. answered to C. the question D. asked by the Inspector. E. No error. Answer: Option B 52. Find out whether there is any grammatical error in below sentence. From a tiny roon in / the slums, they have / managed to move to a good / house in a better locality. A. From a tiny roon in B. the slums, they have C. managed to move to a good D. house in a better locality. E. No error. Answer: Option C 53. Find out whether there is any grammatical error in below sentence. As always have been said, / parents should not / impose their desires / on their wards. A. As always have been said, B. parents should not C. impose their desires D. on their wards. E. No error. Answer: Option A

54. Find out whether there is any grammatical error in below sentence. I offered him part-time work / but he turned it over / saying that he would / rather wait for a full-time job. A. I offered him part-time work B. but he turned it over C. saying that he would D. rather wait for a full-time job . E. No error. Answer: Option B 55. Find out whether there is any grammatical error in below sentence. Sarang is the only / person who can / complete this work / within stipulated deadline. A. Sarang is the only B. person who can C. complete this work D. within stipulated deadline. E. No error. Answer: Option D 56. Find out whether there is any grammatical error in below sentence. Why some people don't get / what they deserve / and why others get what they don't deserve / is a matter decided by luck. A. Why some people don't get B. what they deserve C. and why others get what they don't deserve D. is a matter decided by luck. E. No error. Answer: Option A 57. Find out whether there is any grammatical error in below sentence. Pass on / the salt / please / do you? A. Pass on B. the salt C. please D. do you? E. No error. Answer: Option D

184 | P a g e

58. Find out whether there is any grammatical error in below sentence. He being the eldest son, / his father expects him / to take care of several things / A. He being the eldest son, B. his father expects him C. to take care of several things D. besides his regular studies. E. No error. Answer: Option E 59. Find out whether there is any grammatical error in below sentence. The great actor was / angry with the treatment / he had received / without any hesitation. A. The great actor was B. angry with the treatment C. he had received D. without any hesitation. E. No error. Answer: Option B 60. Find out whether there is any grammatical error in below sentence. The results of the recognition / of this fact are seen / in the gradual improvement / of the diet of the poor. A. The results of the recognition B. of this fact are seen C. in the gradual improvement D. of the diet of the poor. E. No error. Answer: Option D 61. Find out whether there is any grammatical error in below sentence. We will pack not only / the material properly / but will also deliver it / to your valued customers. A. We will pack not only B. the material properly C. but will also deliver it D. to your valued customers. E. No error. Answer: Option A 185 | P a g e

62. Find out whether there is any grammatical error in below sentence. He could succeed / in catching the ball / before it reaches / the boundary line. A. He could succeed B. in catching the ball C. before it reaches D. the boundary line. E. No error. Answer: Option C 63. Find out whether there is any grammatical error in below sentence. Mohan has collected / all the necessary documents / and have written a good paper / for this conference. A. Mohan has collected B. all the necessary documents C. and have written a good paper D. for this conference. E. No error. Answer: Option C 64. Find out whether there is any grammatical error in below sentence. I am trying to finish / this letter for the last one hour / I wish you would / go away or stop disturbing me. A. I am trying to finish B. this letter for the last one hour C. I wish you would D. go away or stop disturbing me. E. No error. Answer: Option C

65. Find out whether there is any grammatical error in below sentence. The foremost criterion of selection we adopted / were the number of years of training / a dancer had received / under a particular guru. A. The foremost criterion of selection we adopted B. were the number of years of training C. a dancer had received D. under a particular guru. E. No error. Answer: Option B

68. Find out whether there is any grammatical error in below sentence. the five-member committee were / of the view that the present service conditions / of the employees of this company / are quite good. A. the five-member committee were B. of the view that the present service conditions C. of the employees of this company D. are quite good. E. No error. Answer: Option A

66. Find out whether there is any grammatical error in below sentence. If you would have / gone to his house / before 10 a.m., you would have / got his autograph. A. If you would have B. gone to his house C. before 10 a.m., you would have D. got his autograph. E. No error.

69. Find out whether there is any grammatical error in below sentence. The man / is / a / social animal. A. The man B. is C. a D. social animal. E. No error Answer: Option A

Answer: Option A 70. Find out whether there is any 67. Find out whether there is any grammatical error in below sentence. grammatical error in below sentence. Jayesh loved his Guru immensely / and If the by-stander had not been / familiar gave him fullest loyalty, / yet he had his with first-aid techniques / the driver own / independent way of thinking. which had met / with the accident would A. Jayesh loved his Guru immensely have died. B. and gave him fullest loyalty, A. If the by-stander had not been C. yet he had his own B. familiar with first-aid techniques D. independent way of thinking. C. the driver which had met E. No error. D. with the accident would have died. E. No error. Answer: Option D Answer: Option C

186 | P a g e

Spotting Errors: Section 7 1. Find out whether there is any grammatical error in below sentence. We fail to understand / why do most educated people / lose their temper even / without any apparent reason. A. We fail to understand B. why do most educated people C. lose their temper even D. without any apparent reason. E. No error.

4. Find out whether there is any grammatical error in below sentence. The chairman welcomed / all the guests and / gave an outline of the / activities performing by the Company. A. The chairman welcomed B. all the guests and C. gave an outline of the D. activities performing by the Company. E. No error.

Answer: Option B

Answer: Option D

2. Find out whether there is any grammatical error in below sentence. Day in and day out / he keep telling / his friends that / he wants to go abroad. A. Day in and day out B. he keep telling C. his friends that D. he wants to go abroad. E. No error.

5. Find out whether there is any grammatical error in below sentence. Reasonable ambition, if supported / at persistent efforts, / is likely to yield / the desired results. A. Reasonable ambition, if supported B. at persistent efforts, C. is likely to yield D. the desired results. E. No error.

Answer: Option B Answer: Option B 3. Find out whether there is any grammatical error in below sentence. I was standing / at the bus stop / waiting for him / since eight o'clock. A. I was standing B. at the bus stop C. waiting for him D. since eight o'clock. E. No error. Answer: Option A

6. Find out whether there is any grammatical error in below sentence. Their only demand / for additional wages were / considered sympathetically / by the progressive management. A. Their only demand B. for additional wages were C. considered sympathetically D. by the progressive management. E. No error. Answer: Option B

187 | P a g e

7. Find out whether there is any grammatical error in below sentence. They have been / struggling with the management / from the past five years / but their demands are not considered. A. They have been B. struggling with the management C. from the past five years D. but their demands are not considered. E. No error. Answer: Option C 8. Find out whether there is any grammatical error in below sentence. Your machine would not have / given you so much trouble / if you had / maintained it proper. A. Your machine would not have B. given you so much trouble C. if you had D. maintained it proper. E. No error.

10. Find out whether there is any grammatical error in below sentence. Even though it was / raining bad I / went out to / get some medicines. A. Even though it was B. raining bad I C. went out to D. get some medicines. E. No error. Answer: Option B 11. Find out whether there is any grammatical error in below sentence. The new facts he / has discovered and the new / arguments he has advanced / has not changed my opinion. A. The new facts he B. has discovered and the new C. arguments he has advanced D. has not changed my opinion. E. No error. Answer: Option D

Answer: Option D 12. Find out whether there is any 9. Find out whether there is any grammatical error in below sentence. grammatical error in below sentence. Immediately after boarding the bus, / Our system of assigning / different jobs Mahesh asked the conductor. / that if he to different people / should be based on knew / where the museum was, / their strengths and weaknesses. A. Immediately after boarding the bus, A. Our system of assigning B. Mahesh asked the conductor. B. different jobs to different people C. that if he knew C. should be based on D. where the museum was, D. their strengths and weaknesses. E. No error. E. No error. Answer: Option C Answer: Option D

188 | P a g e

13. Find out whether there is any grammatical error in below sentence. Twenty-five kilometres / from Bhubaneswar to Cuttack / are / a long distance. A. Twenty-five kilometres B. from Bhubaneswar to Cuttack C. are D. a long distance. E. No error.

16. Find out whether there is any grammatical error in below sentence. In a very harsh tone, / he shouted at his servants / and told them that / he does not need their services. A. In a very harsh tone, B. he shouted at his servants C. and told them that D. he does not need their services. E. No error.

Answer: Option C

Answer: Option D

14. Find out whether there is any grammatical error in below sentence. He loosened his temper / whenever he knows / things do not take place / as per his planning. A. He loosened his temper B. whenever he knows C. things do not take place D. as per his planning. E. No error.

17. Find out whether there is any grammatical error in below sentence. Honesty, integrity and being intelligent / are the qualities which / we look for when / we interview applicants. A. Honesty, integrity and being intelligent B. are the qualities which C. we look for when D. we interview applicants. E. No error.

Answer: Option A

Answer: Option A

15. Find out whether there is any grammatical error in below sentence. He was very disappointed / when he found / that someone else / had secured higher marks. A. He was very disappointed B. when he found C. that someone else D. had secured higher marks. E. No error.

18. Find out whether there is any grammatical error in below sentence. They could have / helped him / had they approached by him / for help well in advance. A. They could have B. helped him C. had they approached by him D. for help well in advance. E. No error.

Answer: Option E

Answer: Option C

189 | P a g e

19. Find out whether there is any grammatical error in below sentence. So longer as / you are honest / and forthright I will / support you in this task. A. So longer as B. you are honest C. and forthright I will D. support you in this task. E. No error.

22. Find out whether there is any grammatical error in below sentence. Though he suffered of fever, / he attended office / and completed all the pending / work by sitting late. A. Though he suffered of fever, B. he attended office C. and completed all the pending D. work by sitting late. E. No error.

Answer: Option A

Answer: Option A

20. Find out whether there is any grammatical error in below sentence. On his attitude / it seems that what he wants / is that the decision-making power / should rest with him. A. On his attitude B. it seems that what he wants C. is that the decision-making power D. should rest with him. E. No error.

23. Find out whether there is any grammatical error in below sentence. All individuals are / different so their / tastes vary from / one another. A. All individuals are B. different so their C. tastes vary from D. one another. E. No error. Answer: Option D

Answer: Option A 24. Find out whether there is any 21. Find out whether there is any grammatical error in below sentence. grammatical error in below sentence. The future is / yet to come / but you Foolishly Madhu threw / some water on have a / a lot from us in future. the electric heater / when it catches fire / A. The future is and she got a shock. B. yet to come A. Foolishly Madhu threw C. but you have a B. some water on the electric heater D. a lot from us in future. C. when it catches fire E. No error. D. and she got a shock. E. No error. Answer: Option A Answer: Option C

190 | P a g e

25. Find out whether there is any grammatical error in below sentence. Ten kilometres / are / a / long distance. A. Ten kilometres B. are C. a D. long distance. E. No error. Answer: Option B 26. Find out whether there is any grammatical error in below sentence. Rajdeep always introduces / himself by his / first name and never mentions / his family name. A. Rajdeep always introduces B. himself by his C. first name and never mentions D. his family name. E. No error. Answer: Option D

29. Find out whether there is any grammatical error in below sentence. Rosy herself wash / all the clothes and / never gives them / to the laundry. A. Rosy herself wash B. all the clothes and C. never gives them D. to the laundry. E. No error. Answer: Option A 30. Find out whether there is any grammatical error in below sentence. Generally, people have not receptive / and fail to notice the / irrelevant answers they / get for straight questions. A. Generally, people have not receptive B. and fail to notice the C. irrelevant answers they D. get for straight questions. E. No error. Answer: Option A

27. Find out whether there is any grammatical error in below sentence. Rajdeep always introduces / himself by his / first name and never mentions / his family name. A. Rajdeep always introduces B. himself by his C. first name and never mentions D. his family name. E. No error.

31. Find out whether there is any grammatical error in below sentence. Pramod said that / he prefers a white shirt / to coloured one / A. Pramod said that B. he prefers a white shirt C. to coloured one D. on any festival occasion. E. No error.

Answer: Option D

Answer: Option B

28. Find out whether there is any grammatical error in below sentence. We have helped them not only / with money but also / with new machinery / and raw material. A. We have helped them not only B. with money but also C. with new machinery D. and raw material. E. No error. Answer: Option E 191 | P a g e

32. Find out whether there is any grammatical error in below sentence. A high level meeting / of officials is reporting / to have discussed / the issue in great detail. A. A high level meeting B. of officials is reporting C. to have discussed D. the issue in great detail. E. No error. Answer: Option B

33. Find out whether there is any grammatical error in below sentence. Whatever work / that which you undertake / put your best / efforts in it. A. Whatever work B. that which you undertake C. put your best D. efforts in it. E. No error. Answer: Option B 34. Find out whether there is any grammatical error in below sentence. I personally feel that / cleanliness in the city / is one proof of the / efficiently civic administration. A. I personally feel that B. cleanliness in the city C. is one proof of the D. efficiently civic administration. E. No error. Answer: Option D 35. Find out whether there is any grammatical error in below sentence. If you cannot / sympathy with the poor, / how will you be / able todo social work? A. If you cannot B. sympathy with the poor, C. how will you be D. able todo social work? E. No error.

37. Find out whether there is any grammatical error in below sentence. The majority of the / computer professionals recommends / that effective measures / should be taken against software piracy. A. The majority of the B. computer professionals recommends C. that effective measures D. should be taken against software piracy. E. No error. Answer: Option A 38. Find out whether there is any grammatical error in below sentence. The photograph will give / the reader a far better / notion of the structure / than any verbal description. A. The photograph will give B. the reader a far better C. notion of the structure D. than any verbal description. E. No error. Answer: Option C

39. Find out whether there is any grammatical error in below sentence. One of the most effective / solutions is that / she should work on Sunday / and Answer: Option B complete the assignment. A. One of the most effective B. solutions is that 36. Find out whether there is any C. she should work on Sunday grammatical error in below sentence. D. and complete the assignment. While going / through the report / yesterday I find / several factual mistakes. E. No error. A. While going B. through the report Answer: Option E C. yesterday I find D. several factual mistakes. E. No error. Answer: Option C 192 | P a g e

40. Find out whether there is any grammatical error in below sentence. On resuming his duty, / he asked his Superiors / that whether he would be / permitted to leave early. A. On resuming his duty, B. he asked his Superiors C. that whether he would be D. permitted to leave early. E. No error.

44. Find out whether there is any grammatical error in below sentence. In the absence of / clear instuctions / one cannot be expected / to be functioned effectively. A. In the absence of B. clear instuctions C. one cannot be expected D. to be functioned effectively. E. No error.

Answer: Option C

Answer: Option D

41. Find out whether there is any grammatical error in below sentence. Very few employees / in our company are / so dedicated as / Mahesh will. A. Very few employees B. in our company are C. so dedicated as D. Mahesh will. E. No error.

45. Find out whether there is any grammatical error in below sentence. What is needed today is / a new breed of managers / with a new set of concepts / and a flexible way about thinking. A. What is needed today is B. a new breed of managers C. with a new set of concepts D. and a flexible way about thinking. E. No error.

Answer: Option D 42. Find out whether there is any grammatical error in below sentence. Each cigarette / a person smoke / does some harm and eventually / it may cause a serious disease. A. Each cigarette B. a person smoke C. does some harm and eventually D. it may cause a serious disease. E. No error.

Answer: Option D

Answer: Option B

46. Find out whether there is any grammatical error in below sentence. Rajesh won the case as / he argued very forcefully and / in such the intelligent way / that the judge changed his opinion. A. Rajesh won the case as B. he argued very forcefully and C. in such the intelligent way D. that the judge changed his opinion. E. No error. Answer: Option C

43. Find out whether there is any grammatical error in below sentence. A detailed inquiry / in the incident / has been initiated / by the Central Government. A. A detailed inquiry B. in the incident C. has been initiated D. by the Central Government. E. No error. Answer: Option B

47. Find out whether there is any grammatical error in below sentence. The minister's speech / has been reported / to the newspaper / No mistake A. The minister's speech B. has been reported C. to the newspaper D. No mistake E. No error Answer: Option C

193 | P a g e

48. Find out whether there is any grammatical error in below sentence. Yesterday, a visitor to / the park was attacked / by a tiger and / had to hospitalise. A. Yesterday, a visitor to B. the park was attacked C. by a tiger and D. had to hospitalise. E. No error. Answer: Option D

51. Find out whether there is any grammatical error in below sentence. In order to save petrol, / motorists must have to / be very cautious / while driving along the highways. A. In order to save petrol, B. motorists must have to C. be very cautious D. while driving along the highways. E. No error. Answer: Option B

49. Find out whether there is any grammatical error in below sentence. All of you will agree with me / that no problem faced by our society / is as grave and intractable / as this problem is A. All of you will agree with me B. that no problem faced by our society C. is as grave and intractable D. as this problem is E. No error.

52. Find out whether there is any grammatical error in below sentence. Even after requesting / him, he did not / tell us that how / he wolved the problem A. Even after requesting B. him, he did not C. tell us that how D. he wolved the problem E. No error.

Answer: Option B

Answer: Option C

50. Find out whether there is any grammatical error in below sentence. The basket of apples / sent by the gardener / contained a number of / green mangoes also. A. The basket of apples B. sent by the gardener C. contained a number of D. green mangoes also. E. No error.

53. Find out whether there is any grammatical error in below sentence. Every animal in the zoo / is fed regularly / and attended to / very promptly. A. Every animal in the zoo B. is fed regularly C. and attended to D. very promptly. E. No error.

Answer: Option E

194 | P a g e

Answer: Option C

54. Find out whether there is any grammatical error in below sentence. Since it was a memory test / the students were instructed / to learn the / passage with heart. A. Since it was a memory test B. the students were instructed C. to learn the D. passage with heart. E. No error. Answer: Option D 55. Find out whether there is any grammatical error in below sentence. This is one of / the most interesting book / I have / ever read. A. This is one of B. the most interesting book C. I have D. ever read. E. No error Answer: Option B

195 | P a g e

56. Find out whether there is any grammatical error in below sentence. Neither the earthquake / nor the subsequent fire / was able to dampen / the spirit of the residents. A. Neither the earthquake B. nor the subsequent fire C. was able to dampen D. the spirit of the residents. E. No error. Answer: Option C

Sentence Correction: Section 1 1. The small child does whatever his father was done. A. has done B. did C. does D. had done E. No correction required Answer: Option C

5. They were all shocked at his failure in the competition. A. were shocked at all B. had all shocked at C. had all shocked by D. had been all shocked on E. No correction required Answer: Option E

6. I need not offer any 2. You need not come unless explanation regarding this you want to. incident - my behaviour is A. You don't need to come speaking itself. unless you want to A. will speak to itself B. You come only when you B. speaks for itself want to C. has been speaking C. You come unless you D. speaks about itself don't want to E. No correction required D. You needn't come until Answer: Option B you don't want to E. No correction required 7. He is too important for Answer: Option A tolerating any delay. A. to tolerate 3. There are not many men B. to tolerating who are so famous that they C. at tolerating are frequently referred to by D. with tolerating their short names only E. No correction required A. initials Answer: Option A B. signatures C. pictures 8. The population of Tokyo D. middle names is greater than that of any E. No correction required other town in the world. Answer: Option A A. greatest among any other B. greater than all other 4. The man to who I sold my C. greater than those of any house was a cheat. other A. to whom I sell D. greater than any other B. to who I sell E. No correction required C. who was sold to Answer: Option E D. to whom I sold E. No correction required Answer: Option D 196 | P a g e

9. The performance of our players was rather worst than I had expected. A. bad as I had expected B. worse than I had expected C. worse than expectation D. worst than was expected E. No correction required Answer: Option B 10. Why did you not threw the bag away? A. did you not throw B. had you not threw C. did you not thrown D. you did not thrown E. No correction required Answer: Option A 11. Shapes of gods and goddess are worshipped by people. A. Images B. Reflections C. Clay shapes D. Clay toys E. No correction required Answer: Option A 12. In addition to enhanced their reputations through strategic use of philanthropy, companies are sponsoring social initiatives to open new markets. A. of enhancing their reputation B. to having enhance their reputation C. to enhancing their reputation D. to have their reputation enhancing E. No correction required Answer: Option C

13. The intruder stood quietly for few moments A. for few time B. for the few moments C. for moments D. for a few moments E. No correction required Answer: Option D

17. He found the gold coin as he cleans the floor. A. as he had cleaned B. while he cleans C. which he is cleaning D. while cleaning E. No correction required Answer: Option D

21. Acquisition of certain specific skills can be facilitated from general awareness, education to novel situations A. can be facilitated by B. may facilitate through C. can be felicitated with D. may be felicitated with 14. The police has so far 18. He admired the speed E. No correction required succeeded in recovering only with which he completed the Answer: Option A a part of the stolen property. work and appreciating the A. thus far succeeded for method adopted by him 22. He never has and ever recovery A. appreciate the method will take such strong B. so far succeeded in being adopted measures. recovery of B. appreciated the method A. had taken nor will ever C. as for as succeeded in adopted take recovery of C. appreciate the method of B. had taken and will ever D. so far succeeded to adoption take recover D. appreciated the method C. has and never will take E. No correction required adopting method D. had and ever will take Answer: Option E E. No correction required E. No correction required Answer: Option B Answer: Option A 15. He confidentially asked the crowd if they thought he 19. Maria unnecessarily 23. Technology must use to was right and the crowd picked up a quarrel with Rani feed the forces of change. shouted that they did. and left the party hurried. A. must be used to feed A. that he did A. has picked up B. must have been using to B. that they had B. picked on feed C. that he is C. picked C. must use having fed D. that he didn't D. picking up D. must be using to feed E. No correction required E. No correction required E. No correction required Answer: Option E Answer: Option C Answer: Option A 16. Why should the candidates be afraid of English Language is not clear. A. the candidates should be B. do the candidates be C. should be the candidates D. are the candidates E. No correction required Answer: Option A

20. She cooks, washes dishes, does her homework and then relaxing. A. relaxing then B. then is relaxing C. relaxing is then D. then relaxes E. No correction required Answer: Option D

197 | P a g e

24. Anyone interested in the use of computers can learn much if you have access to a personal computer. A. they have access B. access can be available C. he or she has access D. one of them have access E. No correction required Answer: Option C

25. They are not beware of all the facts A. are not aware for B. are not aware of C. are not to be aware D. must not to be aware for E. No correction required Answer: Option B 26. We can not always convey ourselves in simple sentences. A. cannot always convey B. can not always express C. cannot always express D. can not always communicate E. No correction required Answer: Option C

29. Despite of their differences on matters of principles, they all agree on the demand of hike is salary? A. Despite their B. Despite of the C. Despite for their D. Despite off their E. No correction required Answer: Option A

30. The man who has committed such a serious crime must get the mostly severe punishment. A. be getting the mostly severely B. get the most severe C. have got the most severely D. have been getting the 27. What does agonise me severe most most is not this criticism, but E. No correction required the trivial reason behind it. Answer: Option B A. most agonising me B. agonises me most C. agonising me most 31. For many centuries in D. I most agonised Indian History there was no E. No correction required city so famous like the city of Answer: Option B Ujjain. A. as 28. As there was no time, the B. such as remaining items were C. likewise deferred into the next D. so like meeting. E. No correction required A. are deferred till Answer: Option A B. were deferred till C. were deferred to 32. We don't know how did D. had deferred with the thief made an escape. E. No correction required A. how the thief did make Answer: Option B B. how the thief does make C. how the thief made D. how was the thief made E. No correction required Answer: Option C

198 | P a g e

33. He is a singer of repute, but his yesterday's performance was quite disappointing. A. performances of yesterday were B. yesterday performance was C. yesterday performance were D. performances about yesterday were E. No correction required Answer: Option E 34. Their earnings are such that they find it difficult to make both ends to meet. A. to makings both ends meet B. to make both ends for meeting C. to make both ends meet D. for making both ends to meet E. No correction required Answer: Option C 35. He has received no other message than an urgent telegram asking him to rush his village immediately. A. asked him to rush his village B. asking him to have rush his village C. asking him to rush to his village D. asking him rushing at his village E. No correction required Answer: Option C

36. One of the most significant phenomenons of our time has been the development of cinema. A. phenomenon B. phenomena C. phenomenonna D. phenomenonns E. No correction required Answer: Option B

40. If you are thinking about investigation overseas, isn't it makes sense to find an experience guide? A. it is not making B. doesn't it make C. does it make D. is it making E. No correction required Answer: Option B

37. Had you been told me about your problem, I would have helped you. A. If you would have told B. Had you have told C. had you told D. If you have told E. No correction required Answer: Option C

41. This is one of the most important inventions of this century. A. invention of this century B. invention of these century C. invention of centuries D. invention of the centuries E. No correction required Answer: Option E

38. It was until many years later that Gandhi became a rebel against authority. A. It was not until many years B. It was till many years C. It was not many years D. Until It was many years E. No correction required Answer: Option A 39. Anand has the guts to rise from the occasion and come out successfully. A. in rising from B. to raise with C. to rise to D. to rise against E. No correction required Answer: Option C

199 | P a g e

45. The research study is an eye-opener and attempts to acquaint us with the problems of poor nations. A. attempted to acquaint B. attempts at acquainting C. attempt to acquaint D. attempting to acquaint E. No correction required Answer: Option B

46. It was unanimously resolved that the parties should unitedly undertook launching of popular programmes. A. should be united undertook B. should be unitedly undertaken C. should be unitedly undertake 42. The orator had been left D. should unitedly undertake the auditorium before the E. No correction required audience stood up. Answer: Option D A. had been leaving B. was left 47. They continued to work C. had left in the field despite of the D. would leave heavy rains. E. No correction required A. even though there is Answer: Option C heavy rain B. although heavily rains 43. He dislikes the word C. in spite the heavy rains dislike, isn't he D. even though it rained A. didn't he B. doesn't he heavily C. hasn't he D. does he E. No correction required E. No correction required Answer: Option D Answer: Option B 48. It is always better to 44. We must take it granted make people realise the that Madhu will not come importance of discipline than for today's function. to impose them on it. A. take it for granted A. impose it with them B. impose them with it B. taking it granted C. imposing them on it C. took it as granted D. impose it on them D. have it granted E. No correction required E. No correction required Answer: Option D Answer: Option A

49. My doctor knew that I would eventually recover and do kind of work I would be doing before A. would have been doing B. would have done C. had been done D. had been doing E. No correction required Answer: Option D 50. Later he became unpopular because he tried to lord it on his followers. A. to lord it for B. to lord over C. to lord it over D. to lord it over on E. No correction required Answer: Option C

53. The drama had many scenes which were so humorous that it was hardly possible to keep a straight face. A. hardly possible for keeping B. hardly impossible for keeping C. hardly impossible for keep D. hardly possible keeping E. No correction required Answer: Option E 54. Hardly does the sun rise when the stars disappeared. A. have the sun rose B. had the sun risen C. did the sun rose D. the sun rose E. No correction required Answer: Option B

57. All the allegations levelled against him were found to be baseless. A. level against B. level with C. levelling with D. levelled for E. No correction required Answer: Option E 58. Ramesh is as tall if not, taller than Mahesh. A. not as tall but B. not so tall but as C. as tall as, if not D. as if not E. No correction required Answer: Option C

59. He hesitated to listen to what his brother was saying. 51. The crops are dying; it A. listened to hesitate must not had rained. B. hesitated listen to A. must had not 55. You will be late if you do C. hesitates to listening B. must not be not leave now D. is hesitated to listen to C. must not have A. did not leave E. No correction required D. must not have been B. left Answer: Option E E. No correction required C. will not leave Answer: Option C D. do not happen to leave 60. The prosecution failed in E. No correction required establish in every case today. 52. The courts are actively to Answer: Option E A. to safeguard the interests and B. on the rights of the poor. 56. The train will leave at C. as A. are actively to 8.30 pm, we have been ready D. upon safeguarding by 7.30pm so that, we can E. No correction required B. have been actively reach the station in time. Answer: Option A safeguarding A. were C. have to active in B. must be 61. One of my drawbacks is safeguarding C. are that I do not have D. are actively in D. should have totolerance of ambiguity. safeguarding E. No correction required A. do not have E. No correction required Answer: Option B B. cannot have Answer: Option B C. am not D. did not have to E. No correction required Answer: Option A 200 | P a g e

62. They should have calmly thought of the advantages that would accrue to them. A. should have been calm in thinking about B. should be calmly thought of C. shall have to calmly thought of D. should have calmly think of E. No correction required Answer: Option E

65. Because of his mastery in this field, his suggestions are wide accepted. A. are widely accepted B. widely acceptance C. have widely accepted D. have been wide accepted E. No correction required Answer: Option A

66. They felt humiliated because they realised that they had cheated. A. have been cheated 63. The easiest of the thing B. had been cheated to do is to ask the address to C. had been cheating the postman. D. were to be cheated A. of the things to do E. No correction required B. among the things did Answer: Option B C. of the thing to be done D. of all the things done 67. Tax evaders should E. No correction required heavily punished as they do it Answer: Option A intentionally. A. should be heavy fined 64. We demonstrated to B. should have heavily fined them how we were prepared C. shall have heavy fine the artistic patterns. D. should be heavily fined A. are prepared E. No correction required B. have prepared Answer: Option D C. are preparing D. had prepared E. No correction required Answer: Option D

201 | P a g e

68. We met him immediately after the session in which he had been given a nice speech. A. would be giving B. has been given C. will have given D. had given E. No correction required Answer: Option D 69. For some days the new professor lectured above the heads of his pupils. A. over the head of B. over the heads of C. on the heads of D. through the heds of E. No correction required Answer: Option B 70. The accused now flatly denies have admitted his guilt in his first statement. A. having admitted B. had admitted C. have been admitting D. has admitting E. No correction required Answer: Option A

Sentence Correction: Section 2 1. We were still standing in the queue when the film was beginning. A. film began B. film had begun C. beginning of the film was over D. film begins E. No correction required Answer: Option B

4. They failed in their attempt to repair the demolished portion of that building. A. for their attempt to repair B. in their attempting to repair C. with their attempt to repair D. in their attempt for 2. If I would have realised repairs the nature of job earlier, I E. No correction required would not have accepted it. Answer: Option E A. If I have had 5. I earnestly believe that you B. In case I would have will visit our relatives during C. Had I been your forthcoming trip to D. Had I E. No correction required Mumbai. A. had hardly believe that Answer: Option A B. sincerely would believe C. certainly believing that 3. The crime has growth D. could not believe rapidly in Russia since the E. No correction required disintegration of the Answer: Option E communist system. A. rapid crime has grown 6. By such time you finish B. crime has grown rapidly that chapter, I will write a C. crimes grow rapidly letter. D. crimes have been A. The time when rapidly grown B. By the time E. No correction required C. By that time D. The time Answer: Option B E. No correction required Answer: Option B

202 | P a g e

7. Though we have kept in mind to try and maintainmost facilities, we would like to request you to kindly bear with us any inconvenience that may be caused. A. must keep in mind to try and maintain B. have kept in mind trying and maintain C. would keep in mind to try and to maintain D. should have kept in mind to try and to maintain E. No correction required Answer: Option E 8. The tea-estate administration is in such mess thereis no leader to set the things right. A. in such a mess here B. in a such mess that here C. in such a mess that there D. with such a mess that there E. No correction required Answer: Option C 9. They examined both the samples very carefully but failed to detect any difference in them. A. some difference in B. some difference between C. any difference between D. any difference between E. any difference between Answer: Option C

10. "Friends and comrades, the light has gone away from our lives and there is darkness everywhere" A. off B. out of C. out from D. out off E. No correction required Answer: Option B

14. Making friends is more rewarding than to make enemies. A. to be unsociable B. to be sociable C. being unsociable D. making enemies E. No correction required Answer: Option D

11. Because of his ill health, the doctor has advised him not to refrain from smoking. A. to not refrain from B. to resort to C. to refrain from D. to be refrained from E. No correction required Answer: Option C

15. The moment they saw me, they were delight A. had delighted B. were delighted C. are delighted D. have been delighted E. No correction required Answer: Option B

12. They have a scheme of rewarding the best of the performers every year. A. a best performer B. the best among the performer C. a best among performer D. the best of the performer E. No correction required Answer: Option E

16. He should not had done it. A. had not B. should had not C. should not have D. should have E. No correction required Answer: Option C

13. What happens to all those travellers on the ship was not known? A. What happened of B. What happened in C. What is that happens to D. What happened to E. No correction required Answer: Option D

203 | P a g e

18. The moment the manager came to know fraudulent action of his assistant, he order immediately dismissed him. A. immediately ordered his dismissed B. ordered his immediate dismissal C. immediately order dismissal of his D. ordered for immediately dismissal of him E. No correction required Answer: Option B 19. The meeting was attended to by all invitees. A. all attended to by B. attended by all C. fully attended to by D. like attending to all E. No correction required Answer: Option B

20. If he has to spend five hours in the queue, it was really a wastage. A. is a really wastage B. is real a wastage 17. No sooner do the bells C. has really a wastage D. is really a wastage ring than the curtain rose. E. No correction required A. did the bell ring Answer: Option D B. did the bells ring C. had the bell rang 21. The world has seen small D. had the bell rung real attempt at population E. No correction required and resource planning. A. few Answer: Option B B. little C. less D. a few E. No correction required Answer: Option B

22. My hair stood off ends when I saw the horrible sight. A. stood at ends B. stood on ends C. stood to ends D. stands on ends E. No correction required Answer: Option B

25. The people generally try to curry favour with the corrupt but influential person. A. cook favour B. seek favour C. extract favour D. display favour E. No correction required Answer: Option E

28. Most of the Indian workers are as healthy as, if not healthier than, British workers. A. as if healthy as not healthier B. healthier but not as healthy C. as healthy, if not healthier D. so healthy, if not healthier 23. The long or short of it is E. No correction required that I do not want to deal 26. I have got some tea, but I Answer: Option E with that new firm. do not have a sugar. A. The long and short of it A. some B. The long and short for it B. got C. The long or short for it C. more D. The shot and long for it D. any E. No correction required E. No correction required Answer: Option A Answer: Option D 24. Can you tell me why did you not speak the truth? A. why did not you speak B. that why did you not speak C. why you did not speak D. why did you not spoke E. No correction required Answer: Option C

27. Had I realised how close I was to the edge of the valley, I would not have carried the bags there. A. Had I been realised B. If I would have realised C. When I realised D. Had I had realised E. No correction required Answer: Option E

204 | P a g e

Sentence Improvement: Section 1 1. The workers are hell bent at getting what is due to them. A. hell bent on getting B. hell bent for getting C. hell bent upon getting D. No improvement Answer: Option C 2. When it was feared that the serfs might go too far and gain their freedom from serfdom, the protestant leaders joined the princes at crushing them. A. into crushing B. in crushing C. without crushing D. No improvement Answer: Option B 3. If the room had been brighter, I would have been able to read for a while before bed time. A. If the room was brighter B. If the room are brighter C. Had the room been brighter D. No improvement Answer: Option C

4. The record for the biggest tiger hunt has not been met since 1911 when Lord Hardinge, the then Viceroy of India, shot a tiger than measured 11 feet and 6 inches. A. improved B. broken C. bettered D. No improvement Answer: Option B 5. His powerful desire brought about his downfall. A. His intense desire B. His desire for power C. His fatal desire D. No improvement

Answer: Option B 9. If he had time he will call you. A. would have B. would have had C. has D. No improvement Answer: Option C

10. Will you lend me few Answer: Option B rupees in this hour of need? 6. Will you kindly open the A. lend me any rupees B. borrow me a few rupees knot? C. lend me a few rupees A. untie D. No improvement B. break C. loose Answer: Option C D. No improvement Answer: Option A 7. He sent a word to me that he would be coming late. A. sent word B. had sent a word C. sent words D. No improvement Answer: Option A

205 | P a g e

8. John had told me that he hasn't done it yet. A. told B. tells C. was telling D. No improvement

11. During his long discourse, he did not touch that point. A. touch upon B. touch on C. touch of D. No improvement Answer: Option B

12. He found a wooden broken chair in the room. A. wooden and broken chair B. broken wooden chair C. broken and wooden chair D. No improvement

16. hoping not to be disturbed, I sat down in my easy chair to read the book. I won as a prize. A. I had won as a prize B. I have won as prize C. I had to win as a prize D. No improvement

Answer: Option B

Answer: Option A

20. I took the cycle which he bought yesterday. A. that he bought yesterday B. that which he had bought yesterday C. that he had bought yesterday D. No improvement Answer: Option C

17. More than one person was killed in accident. A. were killed B. are killed C. have been killed D. No improvement

21. Please make it a point to send you letter at my address. A. on my address B. to my address C. in my address D. No improvement

Answer: Option A

Answer: Option B

18. No one could explain how a calm and balanced person like him could penetrate such a mindless act on his friends. A. perpetuate B. perpetrate C. precipitate D. No improvement

22. If you are living near a market place you should be ready to bear the disturbances caused by traffic. A. to bear upon B. to bear with C. to bear away D. No improvement

Answer: Option B

Answer: Option B

15. While crossing the highway a five year old child was knocked out by a passing car. A. away B. up C. down D. No improvement

19. Five years ago today, I am sitting in a small Japanese car, driving across Poland towards Berlin. A. was sitting B. sat C. have been sitting D. No improvement

23. I hope you won't object to me watching while you work. A. against me watching B. me to watch C. to my watching D. No improvement

Answer: Option C

Answer: Option A

13. He could not look anything in the dark room. A. away B. up C. down D. No improvement Answer: Option C 14. The greatest thing in style is to have a use of metaphor. A. away B. up C. down D. No improvement Answer: Option C

206 | P a g e

Answer: Option C

24. You cannot forbid him leaving. A. he leaving B. his leaving C. him to leave D. No improvement Answer: Option C 25. You have come here with a view to insult me. A. to insulting me B. of insulting me C. for insulting me D. No improvement Answer: Option A 26. 20 kms are not a great distance in these days of fast moving vehicles. A. is not a great distance B. is no distance C. aren't a great distance D. No improvement

28. It became clear that the strangers were heading into a serious disaster. A. along B. towards C. for D. No improvement

32. Practically every part of the banana tree is used by man. A. each part B. any part C. most part D. No improvement

Answer: Option B

Answer: Option D

29. The dissidents hold a great problem in every political party. A. cause B. give C. pose D. No improvement Answer: Option A 30. I would have waited for you at the station if I knew that you would come. A. had known B. was knowing C. have known D. No improvement

Answer: Option A

Answer: Option A

27. The more they earn, more they spend. A. More they earn, more they spend B. More they earn, the more they spend C. The more they earn, the more they spend D. No improvement

31. They are social insects, living in communities, regulated by definite laws, each member of society bearing well-defined and separate part in the work of a colony. A. who are living in communities B. living among a community C. who lives with a community D. No improvement

Answer: Option C

Answer: Option D 207 | P a g e

33. My opinion for the film is that it will bag the national ward. A. opinion to B. opinion about C. opinion on D. No improvement Answer: Option B 34. The end of the examinations is (an) opportunity for celebrating. A. chance B. moment C. occasion D. No improvement Answer: Option B 35. We were not the wiser for all this effort to explain the case to us. A. none B. neither C. nevertheless D. No improvement Answer: Option A

36. Whenever my students come across new words, I ask them to look for them in the dictionary. A. to look it up B. to look them up C. to look at them D. No improvement

40. If you are not clear about the meaning of a word, it is wise to look to a dictionary. A. look for B. look at C. look up D. No improvement

Answer: Option B

Answer: Option C

37. We look forward to hear from you. A. hearing B. have heard C. listen D. No improvement Answer: Option A 38. It was indeed a shock for her, but she has later recovered from it. A. since B. then C. afterwards D. No improvement Answer: Option A 39. Realising is the significance of technical education for developing country, the government laid aside a large sum on it during the last plan-period. A. laid up B. set aside C. laid out D. No improvement Answer: Option B

208 | P a g e

44. Either he or I am going. A. he or I are going B. he is going or I am C. I or he is going D. No improvement Answer: Option D

45. I hope you vividly remember the premier of 41. You are warned against the film when I, my wife committing the same and you were present in mistake again. the hall. A. to commit A. my wife, I and you B. for committing B. you, I and my wife C. against to commit C. my wife, you and I D. No improvement D. No improvement Answer: Option D

Answer: Option C

42. No sooner he had returned home then his mother felt happy. A. had he returned home when B. he had returned home than C. did he return home than D. No improvement

46. To get one's name in the Rowland Ward's book of hunting records was the hot ambition of every serious hunter. A. extreme B. burning C. high D. No improvement

Answer: Option C

Answer: Option C

43. He should move on to the next point, and not harp one sting only. A. harp on string only B. harp only one string C. harp upon one string only D. No improvement

47. Taxpayers are to be conscious of their privileges. A. have to B. need C. ought to D. No improvement

Answer: Option C

Answer: Option A

48. As she was suffering from high fever, she could not face the examination. A. bear B. suffer C. take D. No improvement Answer: Option C

52. Mr. Smith arrived at India in June last year. A. to B. by C. in D. No improvement

56. If you cross the line you will be disqualified. A. cross upon the line B. cross on the line C. cross out the line D. No improvement

Answer: Option C

Answer: Option D

49. The demonstration passed off peacefully. A. passed out B. passed away C. passed on D. No improvement

53. But in all these cases conversion from scale have well-formulated. A. can be well-formulated B. are well-formulated C. well-formulated D. No improvement

Answer: Option D

Answer: Option B

57. Why the dinosaurs died out is not known. A. it is not known B. the reason is not known C. that is not known D. No improvement Answer: Option D

58. His father won't be able to leave for Varnasi 50. Every time I go in a lift 54. With a thundering roar until they have arrived. to my sixth floor the huge rocket soared up A. until they arrive B. until they will have apartment, I remember the from the launching pad. arrived calm and serenity of my A. flew up C. until they will arrive ancestral home in the B. went upwards D. No improvement village. C. took off A. move in a lift D. No improvement Answer: Option A B. ascend in a lift C. take a lift Answer: Option C D. No improvement 59. I will not go to school, if it shall rain tomorrow. Answer: Option C 55. There is dearth of woman doctor in our state. A. it would rain tomorrow B. it will rain tomorrow We shall have to recruit 51. In fact, if it hadn't been some from the other states. C. it rains tomorrow D. No improvement for his invaluable advice on A. women doctor so many occasions I B. woman doctors Answer: Option C wouldn't have achieved C. women doctors anything in life. D. No improvement A. remarkable advice B. valuable advices Answer: Option C C. priceless suggestion D. No improvement Answer: Option D

209 | P a g e

60. If I stood alone in defence of truth, and the whole world is banded against me and against truth. I would fight them all. A. will be banded B. were banded C. banded D. No improvement

64. The cloud of misfortunes appears to have blown out. A. over B. up C. away D. No improvement Answer: Option A

65. While we would like that all Indian Children to go to school, we need to ponder why they do not. 61. He has not and can never be in the good books A. that all the Indian of his employer because he children B. if all the children of lacks honesty. A. has not and cannot be India C. all Indian children B. has not and can never D. No improvement been C. has not been and can Answer: Option C never be D. No improvement 66. In India today many of Answer: Option C our intellectuals still talk in terms of the French Revolution and the Rights 62. When the examinations of Man, not appreciating were over Anil and me that much has happened went to our native town. since then. A. me and Anil A. much has been B. Anil and I happening C. I and Anil B. much had happened D. No improvement C. much might happen D. No improvement Answer: Option B Answer: Option D 63. Our office clock is not so correct as it should be it 67. I shall be grateful to is usually five minutes fast. you if you are of help to A. right me now. B. regular A. help C. accurate B. would help D. No improvement C. helped D. No improvement Answer: Option C Answer: Option B Answer: Option B

210 | P a g e

68. The logic of Berlin wall already had been undermined but when the news came through that the wall itself had been opened I jumped into a car. A. had been undetermined already B. had already been undetermined C. had been already undetermined D. No improvement Answer: Option B 69. Other countries have eradicated this disease ten years ago. A. eradicated B. had eradicated C. did eradicated D. No improvement Answer: Option A 70. Young men and women should get habituated to reading and writing about current affairs. A. used B. prepared C. trained D. No improvement Answer: Option D

Sentence Improvement: Section 2 1. The poor villagers have waited in the bitter cold for more than 4 hours now. A. have been waiting B. had waited C. has been waiting D. No improvement Answer: Option A 2. The old man felled some trees in the garden with hardly no effort at all. A. hard effort B. hardly any effort C. a hardly any effort D. No improvement Answer: Option B 3. The company goes to great length to ensure that employees can be comfortable in their work environment. A. are comfortable B. will be comfortable C. should be comfortable D. No improvement Answer: Option A 4. I want you to clearly understand that excuses won't do A. you clearly to understand B. you to understand clearly C. to clearly understand you D. No improvement Answer: Option D

211 | P a g e

5. He was fined for careless driving. A. got fined B. fined C. was to be fined D. No improvement Answer: Option D

8. As he is past his teens now, he can look for himself. A. after B. to C. around D. No improvement Answer: Option A

6. Although India is still by far a poor country, it can become rich if its natural and human resources are fully utilised. A. few and far between B. by and large C. by and by D. No improvement

9. There is no more room for you in this compartment. A. there is no more seat B. there is no more space C. there is no more accommodation D. No improvement

Answer: Option B

Answer: Option B

7. The reason why he wrote the letter was because he could not contact him over the phone. A. why he wrote the letter was since B. for which he wrote the letter because C. why he wrote the letter was that D. No improvement

10. Most donors would seriously protest any effort to extrapolate from such limited data. A. protest against B. protest at C. protest to D. No improvement

Answer: Option B

Answer: Option A 11. She says she's already paid me back, but I can't remember, so I'll have to take her word. A. to take her word true B. to take her at her word C. to take her word for it D. No improvement Answer: Option B

12. If you had attended the meeting, you would have benefited a great deal. A. could benefit B. would benefit C. benefited D. No improvement

16. Please remind me of posting these letters to my relatives. A. by posting B. to post C. for posting D. No improvement

Answer: Option D

Answer: Option B

13. This matter admits of no excuse. A. admits to B. admits from C. admits D. No improvement Answer: Option D 14. If he would have tried he would have succeeded. A. is tried B. was tried C. had tried D. No improvement Answer: Option C 15. It will be no good trying to find an excuse next time. A. to try to find B. to try finding C. trying finding D. No improvement Answer: Option A

Answer: Option B

21. My friend was in 17. Not a word they spoke hospital for a week after an to the unfortunate wife accident. about it. A. through A. did they speak B. following B. they will speak C. for C. they had spoken D. No improvement D. No improvement Answer: Option B Answer: Option A 22. All, but her, had made 18. Not long back, in an attempt. Japan, a mysterious nerve A. All, but she, gas affected a large number B. All, but herself, of people. C. All, but her, A. effected D. No improvement B. infected C. infested Answer: Option A D. No improvement Answer: Option B 19. We had nothing to eat since 8'o clock, this morning. A. have had nothing B. has had nothing C. did have nothing D. No improvement Answer: Option A

212 | P a g e

20. We did not see this movie yet. A. have seen B. have not seen C. have seen D. No improvement

23. Whatever to our other problems. we have no shortcoming to cheap labour in India. A. default B. deficit C. scarcity D. No improvement Answer: Option C

24. I have lived in Delhi since I was four. A. am living B. lived C. had lived D. No improvement

26. I shall not go untill I am invited. A. till I am invited B. Unless I am invited C. if not I am invited D. No improvement

Answer: Option D

Answer: Option B

25. This telephone number is not existing. A. has not existed B. does not exist C. has not been existing D. No improvement Answer: Option B

27. He died in the year 1960 at 11pm on 14 July. A. on 14 July in the year 1960 at 11pm B. in the year 1960 on 14 July at 11pm C. at 11pm on 14 July in the year 1960 D. No improvement Answer: Option C

213 | P a g e

28. Due to these reason we are all in favour of universal compulsory education. A. Out of these reasons B. For these reasons C. By these reasons D. No improvement Answer: Option B

Sentence Formation 1. 1. I 2. immediately 3. salary 4. my 5. want A. 43152 B. 15432 C. 25143 D. 42351 E. 45132 Answer: Option B

4. 1. medicine 2. a 3. Neeta 4. given 5. was A. 51423 B. 25431 C. 15423 D. 42531 E. 35421 Answer: Option E

7. 1. him 2. the 3. to 4. charge 5. handover A. 42531 B. 51342 C. 41352 D. 45231 E. 52431 Answer: Option E

2. 1. do 2. today 3. you 4. must 5. it A. 34152 B. 25413 C. 12543 D. 51324 E. 45213 Answer: Option A

5. 1. of 2. we 3. heard 4. him 5. had A. 42351 B. 52341 C. 25341 D. 25431 E. 25314 Answer: Option E

8. 1. seen 2. going 3. you 4. him 5. have A. 35214 B. 35142 C. 32514 D. 35124 E. 53124 Answer: Option B

3. 1. left 2. the 3. house 4. he 5. suddenly A. 12435 B. 21354 C. 45123 D. 52341 E. 24135 Answer: Option C

6. 1. at 2. it 3. take 4. once 5. away A. 23514 B. 14352 C. 32514 D. 53214 E. 42315 Answer: Option C

9. 1. bag? 2. you 3. seen 4. have 5. my A. 51432 B. 43512 C. 42351 D. 42153 E. 21435 Answer: Option C

214 | P a g e

10. 1. killed 2. a 3. Jaswant 4. bear 5. wild A. 31254 B. 53124 C. 23145 D. 43125 E. 15234 Answer: Option A

12. 1. tea 2. have 3. that 4. some 5. before A. 43251 B. 24315 C. 24153 D. 52431 E. 41325 Answer: Option C

14. 1. I 2. help 3. not 4. you 5. did A. 24351 B. 15324 C. 45231 D. 43152 E. 52431 Answer: Option B

11. 1. was 2. and 3. Suresh 4. kind 5. loving A. 31425 B. 54213 C. 34251 D. 15243 E. 12345 Answer: Option A

13. 1. not 2. hotel 3. comfortable 4. was 5. the A. 34521 B. 53412 C. 34152 D. 41352 E. 52413 Answer: Option E

15. 1. not 2. Hari 3. away 4. run 5. did A. 13542 B. 35412 C. 52431 D. 25143 E. 21345 Answer: Option D

215 | P a g e

Ordering Of Words 1. When he P : did not know Q : he was nervous and R : heard the hue and cry at midnight S : what to do The Proper sequence should be: A. RQPS B. QSPR C. SQPR D. PQRS Answer: Option A

3. Then P : it struck me Q : of course R : suitable it was S : how eminently The Proper sequence should be: A. SPQR B. QSRP C. PSRQ D. QPSR Answer: Option C

4. 2. I read an advertisement It has been established that that said P : Einstein was P : posh, air-conditioned Q : although a great Q : gentleman of taste scientist R : are available for R : weak in arithmetic S : fully furnished rooms S : right from his school days The Proper sequence should be: The Proper sequence A. PQRS should be: B. PSRQ A. SRPQ C. PSQR B. QPRS D. SRPQ C. QPSR D. RQPS Answer: Option B Answer: Option B

5. Since the beginning of history P : have managed to catch Q : the Eskimos and Red Indians R : by a very difficulty method S : a few specimens of this aquatic animal The Proper sequence should be: A. QRPS B. SQPR C. SQRP D. QPSR Answer: Option D 6. A French woman P : committed suicide Q : where she had put up R : who had come to Calcutta S : by jumping from the first floor balcony of the hotel The Proper sequence should be: A. PRQS B. QSRP C. RPSQ D. SRQP Answer: Option C

216 | P a g e

7. The national unity of a free people P : to make it impracticable Q : for there to be an arbitrary administration R : depends upon a sufficiently even balance of political power S : against a revolutionary opposition that is irreconcilably opposed to it The Proper sequence should be: A. QRPS B. QRSP C. RPQS D. RSPQ Answer: Option D

8. The grocer P : did not listen to the protests of customer Q : who was in the habit of weighing less R : whom he had cheated S : with great audacity The Proper sequence should be: A. PRSQ B. QSPR C. QPRS D. PQSR Answer: Option C 217 | P a g e

9. They felt safer P : to watch the mountain Q : of more than five miles R : as they settled down S : from a distance

11. If you need help P : promptly and politely Q : ask for attendants R : to help our customers S : who have instructions

The Proper sequence should be: A. RPSQ B. RSQP C. PQSR D. PRSQ

The Proper sequence should be: A. SQPR B. QPSR C. QSRP D. SQRP

Answer: Option A

Answer: Option C

10. People P : at his dispensary Q : went to him R : of all professions S : for medicine and treatment

12. It is easier P : to venture into space Q : for men R : beneath their feet S : than to explore

The Proper sequence should be: A. QPRS B. RPQS C. RQSP D. QRPS Answer: Option C

The Proper sequence should be: A. QRPS B. QPSR C. PSRQ D. PQSR Answer: Option B

13. It was to be P : before their school examination Q : which was due to start R : the last expedition S : in a month's

15. In the darkness P : the long, narrow beard Q : was clearly visible with R : the tall stooping figure of the doctor S : and the aquiline nose

The Proper sequence should be: A. SRQP B. RQSP C. RPQS D. SPRQ

The Proper sequence should be: A. RQPS B. PSQR C. RSQP D. QPRS

Answer: Option C

Answer: Option A

14. This time P : exactly what he had been told Q : the young man did R : beyond his dreams S : and the plan succeeded

16. We have to P : as we see it Q : speak the truth R : there is falsehood and weakness S : even if all around us

The Proper sequence should be: A. QPRS B. QPSR C. PQSR D. QSRP

The Proper sequence should be: A. RQSP B. QRPS C. RSQP D. QPSR

Answer: Option B

Answer: Option D

17. When it began to rain suddenly on the first of January P : to celebrate the new year Q : we ran for shelter R : to the neighbouring house S : where many people had gathered The Proper sequence should be: A. QRPS B. PSQR C. PRSQ D. QRSP Answer: Option D

18. He told us that P : and enjoyed it immensely Q : in a prose translation R : he had read Milton S : which he had borrowed from his teacher The Proper sequence should be: A. RSQP B. QRPS C. RQSP D. RQPS Answer: Option C

218 | P a g e

19. He knew that P : and then to save himself Q : was to save all the lives R : entrusted to his care S : the duty of a captain The Proper sequence should be: A. PQRS B. SQRP C. SPRQ D. QSRP Answer: Option B

21. Of many artists P : those who impressed me the most Q : I was a child R : but those with unique personalities off stage S : were not always the successful ones The Proper sequence should be: A. SRQP B. QRSP C. RSPQ D. QPSR

23. It is easy to excuse P : but it is hard Q : in a boy of fourteen R : the mischief of early childhood S : to tolerate even unavoidable faults The Proper sequence should be: A. RPQS B. QRSP C. QRPS D. RPSQ Answer: Option D

Answer: Option D 20. It is not, therefore P : that I pay a tribute Q : to conductors as a class R : with any feeling of unfriendliness S : to a particular member of that class The Proper sequence should be: A. PQRS B. RQPS C. RSPQ D. PSRQ Answer: Option C

24. 22. The majestic mahogany As a disease table P : and breaks up marriages P : belongs to an old prince Q : accidents and suicides Q : which has one leg R : alcoholism leads to missing S : affecting all ages R : who is no impoverished S : but not without some The Proper sequence pride should be: A. SRPQ The Proper sequence B. RPSQ should be: C. SRQP A. PQSR D. RQPS B. QRSP C. PRSQ Answer: Option C D. QPRS Answer: Option D

219 | P a g e

25. Education is P : of the proper sense of responsibilities Q : the first need R : in a citizen S : for the development

27. He was so kind and generous that P : he not only Q : made others do so R : but also S : helped them himself

The Proper sequence should be: A. SQPR B. QSRP C. QSPR D. PQRS

The Proper sequence should be: A. PSRQ B. SPQR C. PRSQ D. QPRS

Answer: Option C

Answer: Option A

29. There was P : needed for it everyday life Q : a time when each family R : for itself most of the things it S : actually produced The Proper sequence should be: A. QRSP B. RQPS C. RSPQ D. QSRP Answer: Option D

26. It would P : appear from his statement Q : about the policy of management R : in dealing with the strike S : that he was quite in the dark The Proper sequence should be: A. RPSQ B. PSQR C. RQPS D. PRQS Answer: Option B

220 | P a g e

28. We went P : along the railway line Q : and had a right to R : where other people not allowed to go S : but daddy belonged to the railway The Proper sequence should be: A. RPQS B. PRSQ C. RSQP D. PRQS Answer: Option B

30. Little P : that he had been let down Q : stood by all these years R : did he realise S : by a colleague whom he had The Proper sequence should be: A. RPSQ B. RSQP C. QSRP D. QSPR Answer: Option A

31. I saw that P : but seeing my host in this mood Q : I deemed it proper to take leave R : as I had frequently done before S : it had been my intention to pass the night there The Proper sequence should be: A. QPSR B. QRPS C. SPQR D. SRPQ

33. I was P : and stay fro few days in Delhi Q : when my father told me R : very excited S : that I could go with him

35. The appearance P : this dinosaurs were at their peak Q : of the first mammals on the earth R : at the time when S : went almost unnoticed

The Proper sequence should be: A. PQRS B. RQSP C. QRSP D. SPQR

The Proper sequence should be: A. SRPQ B. QSRP C. QRPS D. RPQS

Answer: Option B

Answer: Option C

34. work is the one thing P : and without it Q : that is necessary R : to keep the world going S : we should all die

36. By this time P : at the railway station Q : reported mass looting R : reports of violence were flooding in S : which police dispatches

Answer: Option D

32. It is very easy P : a great deal more than one realises Q : may mean R : that a phrase that one does not quite understand S : to persuade oneself The Proper sequence should be: A. RSQP B. SPQR C. SRQP D. RQPS Answer: Option B

221 | P a g e

The Proper sequence should be: A. QPSR B. RPQS C. SRPQ D. QRPS Answer: Option D

The Proper sequence should be: A. RSPQ B. SPRQ C. SQRP D. RSQP Answer: Option C

37. Then the women P : lamenting their evil desire Q : that had brought R : wept loudly S : this sorrow upon them The Proper sequence should be: A. RPQS B. RQPS C. PQSR D. PRQS Answer: Option A

39. Though he dialled frequently P : on telephone Q : my brother could not contact me R : and had left no information S : as I had gone out of office The Proper sequence should be: A. QPRS B. SQRP C. QPSR D. SPQR

41. He sat P : through the Town Hall Park Q : which flanked a path running R : under the boughs S : of a spreading tamarind tree The Proper sequence should be: A. PQSR B. RSQP C. RSPQ D. PRSQ Answer: Option B

38. Women P : till the other day Q : who were content being housewives R : about spending their time cooking S : now sound apologetic The Proper sequence should be: A. PQRS B. RSPQ C. QPSR D. RQPS Answer: Option C

222 | P a g e

Answer: Option C 42. 40. I don't remember He said P : I saw a man dying in P : here is the cat front of a hospital Q : throughout the night Q : but when I left R : which had been paying lucknow in 1984 havoc with things R : hit apparently by a fast S : that ate the rat moving car S : the exact date The Proper sequence should be: The Proper sequence A. PSQR should be: B. PSRQ A. SQRP C. QPRS B. SQPR D. QRPS C. QRPS D. SPRQ Answer: Option B Answer: Option B

43. It was P : in keeping with my mood Q : a soft summer evening R : as I walked sedately S : in the direction of new house The Proper sequence should be: A. SRPQ B. QRPS C. QPRS D. SQPR Answer: Option C

44. All such students P : get success Q : who are honest and hard-working R : and recognition S : in very walk of life The Proper sequence should be: A. SPRQ B. PRQS C. QPRS D. PSRQ Answer: Option C

223 | P a g e

45. The master P : who was very loyal to him Q : punished the servant R : without giving any valid reason S : when he left the work unfinished

47. P: We must read P : if we want to absorb the fruits of great literature Q : but with concentration R : them not as we do cricket stories S : undefined

The Proper sequence should be: A. RQPS B. RQSP C. QPSR D. QRPS

The Proper sequence should be: A. QPSR B. PSQR C. PRSQ D. PRQS

Answer: Option C

Answer: Option A

46. With her body P : dragging her unwilling feet Q : weak and infirm R : doubled with age S : she persisted in her mission

48. I enclose P : and the postage Q : a postal order R : the price of books S : which will cover

The Proper sequence should be: A. PQRS B. QPRS C. RQPS D. SRPQ Answer: Option C

The Proper sequence should be: A. RPSQ B. QSPR C. QSRP D. QPSR Answer: Option C

Ordering Of Sentences: Section 1 1. S1: Smoke oozed up between the planks. P : Passengers were told to be ready to quit the ship. Q : The rising gale fanned the smouldering fire. R : Everyone now knew there was fire on board. S : Flames broke out here and there. S6: Most people bore the shock bravely. The Proper sequence should be: A. RSQP B. PQRS C. SQPR D. QSRP Answer: Option D 2. S1: A force of exists between everybody in the universe. P : Normally it is very small but when the one of the bodies is a planet, like earth, the force is considerable. Q : It has been investigated by many scientists including Galileo and Newton. R : Everything on or near the surface of the earth is attracted by the mass of earth. S : This gravitational force depends on the mass of the bodies involved. S6: The greater the mass, the greater is the earth's force of attraction on it. We can call this force of attraction gravity.

3. S1: Calcutta unlike other cities kepts its trams. P : As a result there horrendous congestion. Q : It was going to be the first in South Asia. R : They run down the centre of the road. S : To ease in the city decided to build an underground railway line. S6: The foundation stone was laid in 1972. The Proper sequence should be: A. PRSQ B. PSQR C. SQRP D. RPSQ Answer: Option D 4. S1: For some time in his youth Abraham Lincoln was manager for a shop. P : Then a chance Customer would come. Q : Young Lincoln way of keeping shop was entirely unlike anyone else's. R : Lincoln would jump up and attend to his needs and then revert to his reading. S : He used to lie full length on the counter of the shop eagerly reading a book. S6: Never before had Lincoln had so much time for reading as had then.

The Proper sequence should be: A. PRQS B. PRSQ C. QSRP D. QSPR

The Proper sequence should be: A. SRQP B. QSPR C. SQRP D. QPSR

Answer: Option D

Answer: Option B

224 | P a g e

5. S1: All the land was covered by the ocean. P : The leading god fought the monster, killed it and chopped its body in to two halves. Q : A terrible monster prevented the gods from separating the land from the water. R : The god made the sky out of the upper part of the body and ornamented it with stars. S : The god created the earth from the lower part, grew plants on it and populated it with animals. S6: The god moulded the first people out of clay according to his own image and mind.

7. S1: You know my wife, Madhavi, always urged me to give up smoking. P : I really gave it up. Q : And so When I went to jail I said to myself I really must give it up, if for no other reason than of being self-reliant. R : When I emerged from jail, I wanted to tell her of my great triumph. S : But when I met her, there she was with a packet of cigarettes. S6: poor girl!.

The Proper sequence should be: A. PQRS B. PQSR C. QPSR D. QPRS

Answer: Option C

The Proper sequence should be: A. PSRQ B. SPQR C. QPRS D. RSPQ

8. S1: When a satellite is launched, the rocket begins by going slowly upwards Answer: Option D through the air. P : However, the higher it goes, the less air it meets. 6. Q : As the rocket goes higher, it travels S1: Smoke oozed up between the planks. faster. P : Passengers were told to be ready to R : For the atmosphere becomes thinner. quit the ship. S : As a result there is less friction. Q : The rising gale fanned the S6: Consequently, the rocket still does smouldering fire. not become too hot. R : Everyone now knew there was fire on board. The Proper sequence should be: S : Flames broke out here and there. A. QPRS S6: Most people bore the shock bravely. B. QSPR C. PQRS The Proper sequence should be: D. PQSR A. SRQP B. QPSR Answer: Option A C. RSPQ D. QSRP Answer: Option A 225 | P a g e

9. S1: A father having offered to take the baby out in a perambulator, was tempted by the sunny morning to slip into a pub for a glass of beer. P : Indignant at her husband's behaviour, she decided to teach him a lesson. Q : She wheeled away the pram. R : A little later, his wife came by, where to her horror, she discovered her sleeping baby. S : Leaving the pram outside, he disappeared inside the bar. S6: She waited for him, anticipating the white face and quivering lips which would soon appear with the news that the baby had been stolen. The Proper sequence should be: A. SRPQ B. RQPS C. SPQR D. PQSR

11. S1: Venice is a strange and beautiful city in the north of Italy. P : There are about four hundred old stone bridges joining the island of Venice. Q : In this city there are no motor cars, no horses, no buses. R : These small islands are near one another. S : It is not an island but a hundred and seventeen islands. S6: This is because Venice has no streets. The Proper sequence should be: A. PQRS B. PRQS C. SRPQ D. PQSR Answer: Option C

Answer: Option A 10. S1: The city is almost a slum and stinks most of time. P : The slush on the road did not deter them. Q : The occasional slips and falls were considered a small price to pay for the trip. R : They were excited, fascinated by the sight of fresh snow on the roads. S : Even so, it looked beautiful to tourists of various categories. S6: But some visitors came away with the unforgettable sight of young labours scantily clad.

12. S1: The Hound of Baskervilles was feared by the people of the area. P : Some people spoke of seeing a huge, shadowy form a Hound at midnight on the moor. Q : But they spoke of it in tones of horror. R : Nobody had actually seen the hound. S : This shadowy form did not reveal any details about the animal. S6: The Hound of Baskervilles remains an unsolved mystery.

The Proper sequence should be: A. RQPS B. QPRS C. RSQP D. SPQR

The Proper sequence should be: A. SPQR B. SPRQ C. PSRQ D. PQRS

Answer: Option D

Answer: Option A

226 | P a g e

13. S1: A gentleman who lived alone always had two plates placed on the table at dinner time. P : One day just as he sat down to dine, the cat rushed in to the room. Q : One plate was for himself and other was for his cat. R : she drooped a mouse into her own plate and another into her master plate. S : He used to give the cat a piece of meat from his own plate. S6: In this way the cat showed her gratitude to her master. The Proper sequence should be: A. QSPR B. PSRQ C. QRSP D. RPQS Answer: Option A 14. S1: Ants eat worms, centipedes and spiders. P : They are usually much quicker than the ant itself. Q : Nevertheless, these animals do not make easy game for ants. R : Besides, they have an extraordinary number of ways of escaping. S : They also eat larvae and insect adults such as flies, moths and spring tails. S6: Some jump, and some give out a pungent repellent substance. The Proper sequence should be: A. SQPR B. SPRQ C. SQRP D. SRQP Answer: Option A 227 | P a g e

15. S1: Satyajit Ray made several films for children. P : Later film makers have followed his lead. Q : Today other nations are making the children's film in a big way. R : This was at a time when no director considered children as potential audience. S : Ray was, thus, a pioneer in the field. S6: But today few think of Ray as a maker of children's films. The Proper sequence should be: A. PSRQ B. RSQP C. RSPQ D. SQRP Answer: Option C 16. S1: Hungary, with a population of about 10 million, lies between Czechoslovakia to the north and Yugoslavia to the south. P : Here a great deal of grain is grown. Q : In recent years, however, progress has been made also in the field of industrialisation. R : Most of this country consists of an extremely fertile plain, through which the river Danube flows. S : In addition to grain, the plain produces potatoes, sugar, wine and livestock. S6: The new industries derive mainly from agricultural production. The Proper sequence should be: A. QRSP B. RPSQ C. PRSQ D. RQSP Answer: Option B

17. S1: Palaeobotany is the study of fossil plants preserved in rocks dating back in millions of years. P : Records of the history of the world are contained in fossils. Q : Through the ages, plants have evolved from simple to more complex forms. R : First there were water plants then land plants appeared during the Paleozoic era. S : But since the fossil remains appear locked in rock layers, they are closely related to the geologist area of investigation. S6: The fossil plants indicate the age of the rock, and also point to facts regarding climate, temperature and topography. The Proper sequence should be: A. RQSP B. SQRP C. PSQR D. QRPS

19. S1: And then Gandhi came. P : Get off the backs of these peasants and workers, he told us, all you who live by their exploitation. Q : He was like a powerful current of fresh air, like a beam of light, like a whirlwind that upset many things. R : He spoke their language and constantly dre their attention to their appalling conditions. S : He didn't descent from the top, he seemed to emerge from the masses of India. S6: Political freedom took new shape and then acquired a new content. The Proper sequence should be: A. QSRP B. SRQP C. RSQP D. PRSQ Answer: Option B

20. S1: Biological evolution has not fitted man to any specific environment. Answer: Option D P : It is by no means a biological evolution, but it is a cultural one. Q : His imagination, his reason, his 18. emotional subtlety and toughness, makes S1: On vacation in Tangier, Morocco, my it possible for him not to accept the environment but to change. friend and I sat down at a street cafe. P : At one point, he bent over with a big R : And that series of inventions by which man from age by age has reshaped smile, showing me, a single gold tooth his environment is a different kind of and a dingy fez. Q : soon I felt the presence of someone evolution. S : Among the multitude of animals standing alongside me. which scamper, burrow swim around us R : But this one wouldn't budge. S : We had been cautioned about beggars he is in the only one who is not locked in to his environment. and were told to ignore them. S6: That brilliant sequence of cultural S6: Finally a man walked over to me and peaks can most appropriately be termed whispered, "Hey buddy this guy is your the ascent of man. waiter and he wants your order". The Proper sequence should be: The Proper sequence should be: A. QPRS B. SRQP A. SQRP B. SQPR C. QRSP D. SQRP C. QSRP D. QSPR Answer: Option C Answer: Option C 228 | P a g e

21. S1: The dictionary is the best friend of you task. P : That may not be possible always. Q : It is wise to look it up immediately. R : Then it must be firmly written on the memory and traced at the first opportunity. S : Never allow a strange word to pass unchallenged. S6: soon you will realize that this is an exciting task. The Proper sequence should be: A. PQRS B. SPQR C. QRPS D. SQPR Answer: Option D

23. S1: I usually sleep quite well in the train, but this time I slept only a little. P : Most people wanted it shut and I wanted it open. Q : As usual, I got angry about the window. R : The quarrel left me completely upset. S : There were too many people too much huge luggage all around. S6: It was shut all night, as usual. The Proper sequence should be: A. RSQP B. SQPR C. SQRP D. RSPQ Answer: Option B

24. 22. S1: In 1934, William Golding published a S1: The Bhagavadgita recognises the small volume of poems. nature of man and the needs of man. P : During the World War II(1939-45) he P : All these three aspects constitute the joined the Royal Navy and was present at nature of man. the sinking of the Bismarck. Q : It shows how the human being is Q : He returned to teaching in 1945 and rational one, an ethical one and a spiritual gave it up in 1962, and is now a full time one. writer. R : More than all, it must be a spiritual R : In 1939, he married and started experience. teaching at Vishop Wordsworth school in S : Nothing can give him fulfilment Salisbury. unless it satisfies his reason, his ethical S : At first his novels were not accepted. conscience. S6: But the Lord of the files which came S6: A man whom does not harmonise out in 1954 was welcomed as "a most them, is not truly human. absorbing and instructive tale". The Proper sequence should be: A. PSRQ B. RSPQ C. QPSR D. PSQR

The Proper sequence should be: A. RPQS B. RPSQ C. SRPQ D. SQPR

Answer: Option C

Answer: Option A

229 | P a g e

25. S1: The future beckons to us. P : In fact we have hard work ahead. Q : Where do we go and what shall be our endeavour? R : We shall also have to fight and end poverty, ignorance and disease. S : It will be to bring freedom and oppurtunity to the common man. S6: There is no resting for any one of us till we redeem our pledge in full. The Proper sequence should be: A. PSRQ B. QPSR C. QSRP D. SRPQ Answer: Option C 26. S1: Most of the universities in the country are now facing financial crisis. P : Cost benefit yardstick thus should not be applied in the case of universities. Q : The current state of affairs cannot be allowed to continue for long. R : Universities cannot be equated with commercial enterprises. S : Proper development of universities and colleges must be ensured. S6: The Government should realise this before it is too late. The Proper sequence should be: A. QRPS B. QSPR C. QRSP D. QPRS Answer: Option A

230 | P a g e

27. S1: While talking to a group, one should feel self-confident and courageous. P : Nor is it a gift bestowed by providence on only a few. Q : One should also learn how to think calmly and clearly. R : It is like the ability to play golf. S : It is not as difficult as most men imagine. S6: Any man can develop his capacity if he has the desire to do so. The Proper sequence should be: A. SQPR B. QSPR C. QRSP D. RSQP Answer: Option B 28. S1: A ceiling on urban property. P : No mill-owner could own factories or mills or plants. Q : And mass circulation papers. R : Would mean that. S : No press magnate could own printing presses. S6: since their value would exceed the ceiling fixed by the government. The Proper sequence should be: A. QSRP B. RPSQ C. SRPQ D. QPSR Answer: Option B

29. S1: The art of growing old is one which the passage of time has forced upon my attention. P : One of these is undue absorption in the past. Q : One's thought must be directed to the future and to things about which there is something to be done. R : Psychologically, there are two dangers to be guarded against in old age. S : It does not do to live in memories, in regrets for the good old days, or in sadness about friend who are dead. S6: This is not always easy one's own past is gradually increasing weight. The Proper sequence should be: A. QSRP B. RPQS C. RPSQ D. QPRS Answer: Option C

31. S1: Once King Shantnu met a young and beautiful fisher girl. P : He went to the fisherman and asked him for her asked him for her hand in marriage. Q : The King was extremely sad and returned to his palace. R : He fell in love with the fisher girl. S : The fisherman agreed to it condition that the son of his daughter should be heir to the throne of Hastinapur. S6: Devavrata, the King's son, asked him the reason of his sadness. The Proper sequence should be: A. PQRS B. RPSQ C. QSPR D. PSQR Answer: Option B

32. S1: Reliogion is not a matter of mere 30. dogmatic conformity. S1: I keep on flapping my big ears all day. P : It is not merely going through the P : They also fear that I will flip them all ritual prescribed to us. away. Q : It is not a question of ceremonial Q : But children wonder why I flap them piety. so. R : Unless that kind of transformation R : I flap them so to make sure they are occurs, you are not an authentically safely there on either side of my head. religious man. S : But I know what I am doing. S : It is the remarking of your own self, S6: Am I not a smart, intelligent the transformation of your nature. elephant? S6: A man of that character is free from fear, free from hatred. The Proper sequence should be: A. SRQP The Proper sequence should be: B. QPSR A. SPRQ C. QPRS B. QPSR D. PSRQ C. PSRQ D. SPQR Answer: Option B Answer: Option B 231 | P a g e

33. S1: We speak today of self-determination in politics. P : So long as one is conscious of a restraint, it is possible to resist it or to near it as a necessary evil and to keep free in spirit. Q : Slavery begins when one ceases to feel that restraint and it depends on if the evil is accepted as good. R : There is, however, a subtler domination exercised in the sphere of ideas by one culture to another. S : Political subjection primarily means restraint on the outer life of people. S6: Cultural subjection is ordinarily of an unconscious character and it implies slavery from the very start. The Proper sequence should be: A. SPRQ B. RSQP C. SPQR D. RSPQ

35. S1: The December dance and music season in Madras is like the annual tropical cyclone. P : A few among the new aspirants dazzle witht he colour of youth, like fresh saplings. Q : It rains an abundance of music for over a fortnight. R : Thick clouds expectation charge the atmosphere with voluminous advertisements. S : At the end of it one is left with the feeling that the music of only those artists seasoned by careful nurturing, stands tall like well-routed trees. S6: Many a hastily planed shrub gets washed away in the storm. The Proper sequence should be: A. RQPS B. QRPS C. RQSP D. QRSP Answer: Option B

Answer: Option C

36. S1: There is a touching story of Professor Hardy visiting Ramanujan as he lay 34. S1: Once upon a time an ant lived on the desperately ill in hospital at Putney. P : 'No Hardy, that is not a dull number bank of river. in the very least. P : The dove saw the ant struggling in Q : Hardy, who was a very shy man, water in a helpless condition. could not find the words for his distress. Q : All its efforts to come up is failed. R : It was 1729. R : One day it suddenly slipped in to S : The best he could do, as he got to the water. beside was "I say Ramanujan, I thought S : A dove lived in the tree on the bank the number of taxi I came down in was a not far from the spot. very dull number". S6: She was touched. S6: It is the lowest number that can be expressed in two different ways as the The Proper sequence should be: sum of two cubes. A. RQSP B. QRPS C. SRPQ D. PQRS The Proper sequence should be: A. RPQS B. RPSQ Answer: Option A C. PRQS D. QPRS Answer: Option B 232 | P a g e

37. S1: Politeness is not a quality possessed by only one nation or race. P : One may observe that a man of one nation will remove his hat or fold his hands by way of greetings when he meets someone he knows. Q : A man of another country will not to do so. R : It is a quality to be found among all peoples and nations in every corner of the earth. S : Obviously, each person follows the custom of his particular country. S6: In any case, we should not mock at others habits. The Proper sequence should be: A. RPQS B. RPSQ C. PRQS D. QPRS

39. S1: This weather-vane often tops a church spire, tower or high building. P : They are only wind-vanes. Q : Neither alone can tell us what the weather will be. R : They are designed to point to direction from which the wind is coming. S : Just as the barometer only tells us the pressure of air, the weather-vane tells us the direction of wind. S6: The weather-vane can, however give us some indication of other. The Proper sequence should be: A. PQRS B. PSRQ C. PRSQ D. SPQR Answer: Option D

Answer: Option B

40. S1: But how does a new word get into the dictionary? 38. P : When a new dictionary is being edited, S1: Throughout history man has used a lexicographer collects all the energy from the sun. alphabetically arranged citation slips for a P : Today, when we burn wood or use particular word. electric current we are drawing an energy. Q : The dictionary makers notice it and Q : However we now have a new supply make a note of it on a citation slip. of energy. R : The moment new word is coined, it R : All our ordinary life depends on sun. usually enter the spoken language. S : This has come from the sun. S : The word then passes from the realm S6: This energy comes from inside atoms. of hearing to the realm of writing. S6: He sorts them according to their The Proper sequence should be: grammatical function, and carefully writes A. SQPR B. RQPS a definition. C. QSRP D. PSRQ The Proper sequence should be: Answer: Option D A. PQRS B. PRSQ C. RQPS D. RSQP Answer: Option C

233 | P a g e

41. S1: Growing up means not only getting larger, but also using our sense and our brain is to become more aware of things around us. P : Not only does he have a memory but he is able to think and reason. Q : In this, man differs from all other animals. R : Before we spray our roadside plants or turn sewage in to our rivers, we should pause to think what the results of our action are likely to do. S : This is to say, he is able to plan what he is is going to do in the light of his experience before he does it. S6: In other words, we must develop and use our ability to reason, because the destruction or the preservation of the places in which we live depend on us. The Proper sequence should be: A. QRSP B. SPQR C. SPRQ D. QPSR

43. S1: Duryodhana was a wicked prince. P : one day Bhima made Duryodhana fall from a tree from which Duryodhana was stealing fruits. Q : He did not like that Pandavas should be loved and respected by the people of Hastinapur R : Duryodhana specially hated Bhima. S : Among the Pandavas, Bhima was extraordinary strong and powerful. S6: This enraged Duryodhana so much that he began to think of removing Bhima from his way. The Proper sequence should be: A. PSQR B. QPRS C. QSPR D. PSRQ Answer: Option C

Answer: Option D 42. S1: Jawaharlal Nehru was the greatest planenthusiast. P : Under Nehru's advice, the preIndependent congress set up National Planning Commission in 1938. Q : But he forgot that what could be achieved by force under the communist dictatorship of Russia was not possible under the democratic set up of India. R : He took the idea from Russia where Five year plans transformed a very backward country into a top power of the world. S : No free government can call for compulsory sacrifice and suffering from the whole people. S6: Nehru himself became the chairman of the commission. The Proper sequence should be: A. PSQR B. SRQP C. QPRS D. RQSP Answer: Option D 234 | P a g e

44. S1: Jawaharlal Nehru was born in Allahabad on 14 Nov 1889. P : Nehru meet Mahatma Gandhi in February 1920. Q : In 1905 he was sent to London to study at a school called Haroow. R : He became the first Prime Minister of Independent India on 15 August 1947. S : He married Kamla Kaul in 1915. S6: He died on 27 May 1964. The Proper sequence should be: A. QRPS B. QSPR C. RPQS D. SQRP Answer: Option B

45. S1: It was a dark moonless night. P : He turned over the pages, reading passages here and there. Q : He heard them on the floor. R : The poet took down his books of poems from his shelves. S : Some of them contained his earliest writings which he had almost forgotten. S6: They all seemed to him to be poor and ordinary mere childish words. The Proper sequence should be: A. RPQS B. RQSP C. RSPQ D. RPSQ Answer: Option D 46. S1: I had halted on the road. P : As soon as I saw the elephant I knew I should not shoot him. Q : It is a serious matter to shoot a working elephant. R : I knew that his 'must' was already passing off. S : The elephant was standing 8 yards from the road. S6: I decided to watch him for a while and then go home. The Proper sequence should be: A. SPQR B. PQSR C. RQPS D. SRPQ Answer: Option B

47. S1: There is only one monkey we can thoroughly recommend as an indoor pet. P : They quickly die from colds and coughs after the first winter fogs. Q : It is beautiful and intelligent Capuchin monkey. R : The lively little Capuchins, however, may be left for years in an English house without the least danger to their health. S : The Marmosets, it is true, are more beautiful than a Capuchins and just as pleasing, but they are too delicate for the English climate. S6: Finally let me say that no other monkey has a better temper or winning ways. The Proper sequence should be: A. PQRS B. QRPS C. QSPR D. RPSQ Answer: Option B 48. S1: A man can be physically confined within stone walls. P : But his mind and spirit will still be free. Q : Thus his freedom of action may be restricted. R : His hopes and aspiration still remain with him. S : Hence, he will be free spiritually if not physically. S6: No tyranny can intimidate a lover of liberty. The Proper sequence should be: A. PQRS B. SRQP C. QPRS D. QPSR Answer: Option A

235 | P a g e

49. S1: Once upon atime there lived three young men in a certain town of Hindustan. P : All the people of the neighbourhood were mortally afraid of them. Q : They were so powerful that they could catch growing lions and tear them to pieces. R : Someone told them that they would become immortal if they killed Death. S : The young men believed themselves to be very good friends. S6: All of them set out in search of their foe called Death. The Proper sequence should be: A. QPRS B. SQPR C. RSQP D. SRPQ Answer: Option B 50. S1: Metals are today being replaced by polymers in many applications. P : Above all, they are cheaper and easier to process making them a viable alternative to metals. Q : Polymers are essentially a long chains of hydrocarbon molecules. R : Today polymers as strong as metals have been developed. S : These have replaced the traditional chromium-plated metallic bumpers in cars. S6: Many Indian Institutes of science and Technology run special programmes on polymer science. The Proper sequence should be: A. QRSP B. RSQP C. RQSP D. QRPS Answer: Option A 236 | P a g e

51. S1: Since the sixties there has been an increasing interest in neurophysiology, which deals with the neural bases of mental activity and behaviour. P : It has format which is very similar to that of Brain and Language, a sister journal. Q : Since then, a number of journals devoted entirely to this area of research have appeared. R : Before the 1960's when this field was the concern of a small number of investigators, research articles were scattered in various neurological journals. S : Brain and cognition is one such journal. S6: So far the journal has published the mixture of articles including reports and investigations. The Proper sequence should be: A. RQSP B. QRSP C. QSPR D. RSPQ Answer: Option A 52. S1: An elderly lady suddenly became blind. P : The doctor called daily and every time he took away some of her furniture he liked. Q : At last she was cured and the doctor demanded his fee. R : She agreed to pay a large fee to the doctor who would cure her. S : On being refused, the doctor wanted to know the reason. S6: The lady said that she had not been properly cured because she could not see all his furniture. The Proper sequence should be: A. PQRS B. RPQS C. RSPQ D. RQPS Answer: Option B

53. S1: What are the causes of our chronic food shortage ? P : To find for these growing new millions is desperate task. Q : every year, we add more than a crore of persons to our population. R : Despite stupendous efforts by our government, the population is growing unabated. S : The chief cause is the population explosion. S6: This unprecedented growth can drag us to the doors of starvation very soon. The Proper sequence should be: A. QRSP B. SQPR C. QPRS D. PSQR Answer: Option B 54. S1: Our ancestors thought that anything which moved itself was alive. P : The philosopher Descartes thought that both men and animals were machines. Q : But a machine such as a motorcar or a steamship moves itself, as soon as machines which moved themselves had been made, people asked "Is man a machine?" R : And before the days of machinery that was a good definition. S : He also thought that the human machine was partly controlled by the soul action on a certain part of the brain, while animals had no souls. S6: Therefore some scientists think that life is just a very complicated mechanism. The Proper sequence should be: A. PRSQ B. RPQS C. PSQR D. RQPS Answer: Option C 237 | P a g e

55. S1: Payment for imports and exports is made through a system called foreign exchange. P : The value of the money of one country in relation to the money of other countries is agreed upon. Q : These rates of exchange vary from time to time. R : For instance, an American dollar or a British pound sterling is worth certain amounts in the money of other countries. S : Sometimes a United States dollar is worth 12 pesos in Mexico. S6: Another time it may be worth eight pesos. The Proper sequence should be: A. PQRS B. QPRS C. PRQS D. RPQS Answer: Option C 56. S1: Moncure Conway devoted his life to two great objects freedom of thought, and freedom of the individual. P : They threaten both kinds of freedom. Q : But something also has been lost. R : There are now dangers, somewhat different in form from those of the past ages. S : In regard to both these objects, something has been gained since his time. S6: Unless a vigorous and vigilant public opinion can be aroused in defence of them, there will be much less of both a hundred years hence then there is now. The Proper sequence should be: A. PQRS B. QSPR C. SQRP D. RSPQ Answer: Option C

57. S1: The study of speech disorders due to brain injury suggests that patients can think without having adequate control over their language. P : But they succeed in playing games of chess. Q : Some patients, for example fail to find the names of objects presented to them. R : They can even use the concepts needed for chess playing, though they are unable to express many of the concepts in ordinary language. S : They even find it difficult to interpret long written notices. S6: How they manage to do this we do not know.

59. S1: We now know that the oceans are very deep. P : For example, the Indian ocean has a range called the Indian Ridge. Q : Much of it is fairly flat. R : However, there are great mountain ranges as well. S : On average the bottom is 2.5 miles to 3.5 miles down. S6: This reaches from the India to the Antarctic.

The Proper sequence should be: A. PSQR B. RPSQ C. QSPR D. SRPQ

Answer: Option A

The Proper sequence should be: A. SQPR B. PQSR C. RSQP D. QPRS

60. Answer: Option C S1: Minnie went shopping one morning. P : Disappointed She turned around and returned to the parking lot. 58. Q : She got out and walked to the nearest S1: A black haired, young woman came shop. tripping along. R : She drove her car into the parking lot P : She was leading a young woman and stopped. wearing a hat. S : It was there that she realised that she'd Q : The woman swept it off and tossed it forgotten her purse at home. in the air. S6: She drove home with an empty R : The child jumped up to catch the hat. basket. S : The young man tossed his head to shake the hat back. The Proper sequence should be: S6: Both disappeared from view. A. RSQP B. RQSP The Proper sequence should be: C. PQRS A. PSQR B. RPSQ D. QPRS C. QRPS D. SQRP Answer: Option B Answer: Option A

238 | P a g e

61. S1: Far away in a little street there is a poor house. P : Her face is thin and worn and her hands are coarse, pricked by a needle, for she is a seam stress. Q : One of the windows is open and through it I can see a poor woman. R : He has a fever and asking for oranges. S : In a bed in a corner of the room her little boy is lying ill. S6: His mother has nothing to give but water, so he is crying. The Proper sequence should be: A. SRQP B. PQSR C. QPSR D. RSPQ Answer: Option C

63. S1: American private lies may seem shallow. P : Students would walk away with books they had not paid for. Q : A Chinese journalist commented on a curious institution: the library. R : Their public morality, however, impressed visitors. S : But in general they returned them. S6: This would not happen in china, he said. The Proper sequence should be: A. PSQR B. QPSR C. RQPS D. RPSQ Answer: Option B

64. S1: The path of Venus lies inside the path of the Earth. 62. P : When at its farthest from the Earth, S1: A noise started above their heads. Venus is 160 million away P : But people did not take it seriously. Q : With such a wide range between its Q : That was to show everyone that there greatest and leat distances it is natural was something wrong. that at sometimes Venus appears much R : It was a dangerous thing to do. brighter than at others. S : For, within minutes the ship began to R : No other body ever comes so near sink. the Earth, with the exception of the S6: Nearly 200 lives were lost on the Moon and occasional comet or asteroid. fateful day. S : When Venus is at its nearest to the earth it is only 26 million miles away. The Proper sequence should be: S6: When at its brightest, it is easily seen A. PQSR with the naked eye in broad daylight. B. PRQS C. QPRS The Proper sequence should be: D. QPSR A. SRPQ B. SQRP C. PSQR D. QPRS Answer: Option C Answer: Option A

239 | P a g e

65. S1: In India marriages are usually arranged by parents. P : Sometimes boys and girls do not like th idea of arranged marriages. Q : Most young people accepts the state of affairs. R : Shanta was like that. S : They assume their parents can make good choices. S6: She felt she was a modern girl and not subject for bargaining. The Proper sequence should be: A. SPRQ B. PSRQ C. QSPR D. RQPS Answer: Option C

67. S1: Several sub-cities have been planned around capital. P : Dwarga is the first among them. Q : They are expected to alleviate the problem of housing. R : It is coming up in the south west of capital. S : It will cater to over one million people when completed. S6: Hopefully the housing problem will not be as acute at present after these subcities are built. The Proper sequence should be: A. QPRS B. PRSQ C. PQRS D. QRSP Answer: Option A

68. 66. S1: Forecasting the weather has always S1: She said on the phone that she would been a difficult business. report for duty next day. P : During a period of drought, streams P : We waited for few days then we and rivers dried up, the cattle died from decided to go to her place. thirst and were ruined. Q : But she did not. Q : Many different things affect the R : we found it locked. weather and we have to study them S : Even after that we waited for her quite carefully to make accurate forecast. a few days. R : Ancient Egyptians had no need of S6: Eventually we reported to the police. weather in the Nile Valley hardly ever changes. The Proper sequence should be: S : In early times, when there were no A. PRSQ instruments, such as thermometer or the B. QPSR barometer, man looked for tell-tale signs C. QPRS in the sky. D. SQPR S6: He made his forecasts by watching flights of the birds or the way smoke rose Answer: Option C from fire. The Proper sequence should be: A. PRQS B. QPRS C. QRPS D. SPQR Answer: Option B 240 | P a g e

69. S1: As he passed beneath her he heard the swish of her wings. P : He was not falling head long now. Q : The monstrous terror seized him. R : But it only lasted a minute. S : He could hear nothing. S6: The next moment he felt his wings spread outwards. The Proper sequence should be: A. PSQR B. QSPR C. QSRP D. PRQS Answer: Option C

70. S1: Ms. Parasuram started a Petrol Pump in Madras. P : A total to 12 girls now work at the pump. Q : She advertised in newspapers for women staff. R : They operate in 2 shifts. S : The response was good. S6: Thus she has shown the way for many others. The Proper sequence should be: A. PQSR B. SQPR C. QSPR D. PQRS Answer: Option C

241 | P a g e

Ordering Of Sentences: Section 2 1. S1: Sun birds are among the smallest of Indian birds. P : Though they are functionally similar to the humming birds of the New World, they are totally unrelated. Q : They do eat insects too. R : They are also some of the most brilliantly coloured birds. S : sun birds feed on nectar mostly and helped in pollination. S6: Our common sun birds are the purple sun bird, the glossy black species and purple rumped sun bird, the yellow and maroon species. The Proper sequence should be: A. SQPR B. RPSQ C. QPRS D. PSRQ

3. S1: I took cigarettes from my case. P : But when the fit of coughing was over, he replaced it between his lips. Q : I lit one of them and placed it between the lips. R : Then with a feeble hand he removed the cigarette. S : Slowly he took a pull at it and coughed violently. S6: Then he continues to draw on it. The Proper sequence should be: A. PSQR B. QPSR C. QSRP D. SRPQ

Answer: Option A

Answer: Option C

2. S1: For decades, American society has been calling a melting pot. P : Differences remained - in appearence, mannerisms, customs, speech, religion and more. Q : The term has long been a cliche and halftruth. R : But homogenisation was never acheived. S : Yes, immigrants from diverse cultures and traditions did cast off vestiges of their native lands and become almost imperceptibly woven in to the American fabric. S6: In recent years, such differences accentuated by the arrival of immigrants from Asia and other parts of the world in the United States - have become something to celebrate and to nurture.

4. S1: There is difference between Gandhiji's concept of secularism and that of Nehru's. P : Nehru's idea of secularism was equal indifference to all religions and bothering about none of them. Q : According to Gandhiji, all religions are equally true and each scripture is worthy of respect. R : Such secularism which means the rejection of all religions is contrary to our culture and tradition. S : In Gandhiji's view, secularism stands for equal respect for all religions. S6: Instead of doing any good, such secularism can do harm instead of good. The Proper sequence should be: A. SQPR B. PSQR C. QSPR D. PRSQ

The Proper sequence should be: A. QRSP B. SQRP C. SQPR D. QSRP Answer: Option B 242 | P a g e

Answer: Option A

5. S1: As a dramatist Rabindranath was not what might be called a success. P : His dramas were moulded on the lines of the traditional Indian village dramas than the dramas of modern world. Q : His plays were more a catalogue of ideas than a vehicle of the expression of action. R : Actually the drama has always been the life of Indian people, as it deals with legends of gods and goddesses. S : Although in his short stories and novels he was able to create living and well defined characters, he did not seem to be able to do so in dramas. S6: Therefor, drama forms the essential part of the traditional Indian Culture. The Proper sequence should be: A. SRQP B. QPSR C. QSPR D. RSQP Answer: Option C 6. S1: It is regrettable that there is widespread corruption in the country at all levels. P : So there is hardly anything that the government can do about it now. Q : And there are graft and other malpractices too. R : The impression that corruption is universal phenomenon persists and the people not cooperate in checking this evil. S : Recently several offenders were brought to book, but they were not given deterrent punishment. S6: This is indeed a tragedy of great magnitude. The Proper sequence should be: A. QSRP B. SQRP C. RSQP D. PQSR Answer: Option A

243 | P a g e

7. S1: The heart is pump of life. P : They have even succeeded in heart transplants. Q : Nowadays surgeons are able to stop a patients heart and carry out complicated operations. R : A few years ago it was impossible to operate on a patient whose heart was not working properly. S : If heart stops we die in about five minutes. S6: All this was made possible by the invention of heart-lung machine. The Proper sequence should be: A. SRQP B. SPRQ C. SQPR D. SRPQ Answer: Option A 8. S1: Your letter was big relief. P : How did you exams go? Q : After your result, you must come here for a week. R : You hadn't written for over a month. S : I am sure you will come out with flying colours. S6: But don't forget to bring chocolate for Geetha. The Proper sequence should be: A. PSRQ B. QRPS C. RPSQ D. RSPQ Answer: Option C

Related Documents

Examveda English
January 2021 1
Examveda - English
January 2021 1
English Project.docx
January 2021 1
Anita English
January 2021 0
Reading-english
February 2021 1

More Documents from "Suman Rajagopal"

Examveda English
January 2021 1
Examveda - English
January 2021 1
Error Detection.pdf
January 2021 1
Algebra Handbook
February 2021 0